MBE Practice Questions Flashcards

You may prefer our related Brainscape-certified flashcards:
1
Q

real evidence admissibility - chain of custody

A

)roponent must show that the object has been held in a substantially unbroken chain of possession. It is not necessary to negate all possibilities of substitution or tampering; rather, what is required is to show adherence to some system of identification and custody.

How well did you know this?
1
Not at all
2
3
4
5
Perfectly
2
Q

Obscenity Test

A

Obscenity is unprotected speech. The Court has defined obscenity as a depiction of sexual conduct that, taken as a whole, by the average person, using contemporary community standards: (i) appeals to the prurient interest in sex; (ii) portrays sex in a patently offensive way; and (iii) using a national, reasonable person standard, does not have serious literary, artistic, political, or scientific value.

How well did you know this?
1
Not at all
2
3
4
5
Perfectly
3
Q

Admissibility of Learned Treatises

A

Can be used to impeach an expert witness by cross-examining the expert on statements contained in any scientific publication that is established as reliable authority; can also be used substantively - exception to the hearsay rule for learned treatises if: (i) the expert is on the stand and it is called to his attention, and (ii) it is established as reliable authority.

How well did you know this?
1
Not at all
2
3
4
5
Perfectly
4
Q

How to Establish Reliability of a Publication (e.g. Learned Treatises)

A

Reliability of a publication may be established by: (i) the direct testimony or cross-examination admission of the expert, (ii) the testimony of another expert, or (iii) judicial notice.

How well did you know this?
1
Not at all
2
3
4
5
Perfectly
5
Q

Admissibility of Prior Felony Convictions

A

The record of the conviction is hearsay; i.e., it is a statement, other than one made by the declarant while testifying at the trial or hearing, offered to prove the truth of the matter asserted. Under the Federal Rules, however, such judgments fall within the hearsay exception for records of felony convictions. Under the Federal Rules, judgments of felony convictions are admissible in both criminal and civil actions to prove any fact essential to the judgment, whether the judgment arose after trial or upon a plea of guilty. [Fed. R. Evid. 803(22)]

How well did you know this?
1
Not at all
2
3
4
5
Perfectly
6
Q

UCC - Proper Tender of Delivery

A

In a proper tender of delivery under UCC section 2-503, the seller must put and hold conforming goods at the buyer’s disposition for a time sufficient for the buyer to take possession. The seller must give the buyer notice reasonably necessary to enable him to take possession of the goods. Proper tender of delivery entitles the seller to acceptance of the goods and to payment according to the contract. [UCC §2-507]. Proper tender of delivery discharges the duty.

How well did you know this?
1
Not at all
2
3
4
5
Perfectly
7
Q

Unjust Restitution Under UCC Proper Tender of Delivery

A

Proper Tender of Delivery Discharges Duty Under Contract - When a party’s duty of performance is discharged, the other party is entitled to restitution of any benefits that he has transferred to the discharged party in an attempt to perform on his side. If one party would be unjustly enriched, to the detriment of the other, if he were permitted to keep the entire contract price, restitution is permissible in the amount the party was unjustly enriched.

How well did you know this?
1
Not at all
2
3
4
5
Perfectly
8
Q

UCC - “F.O.B”

A

The term “F.O.B.” is a delivery term under the UCC, which governs the contract when it is for the sale of goods. That term means “free on board,” and it obligates the seller to get the goods to the location indicated after the term. [UCC §2-319(1)] - doesn’t matter if parties are merchants

How well did you know this?
1
Not at all
2
3
4
5
Perfectly
9
Q

UCC - Breach of “Good Faith”

A

Uniform Commercial Code (“UCC”) imposes a duty of good faith on all parties, and failure to deliver under a contract simply because a better price can be obtained might violate this duty, the UCC does not provide for punitive damages for breach of this duty. When a seller fails to deliver goods, one remedy available to the buyer is cover damages—the difference between the contract price and the price of substitute goods.

How well did you know this?
1
Not at all
2
3
4
5
Perfectly
10
Q

Speech - Content Based Restriction

A

To justify content-based regulation of otherwise protected speech, the government must show that the regulation is necessary to achieve a compelling state interest that cannot be satisfied by less restrictive means.

How well did you know this?
1
Not at all
2
3
4
5
Perfectly
11
Q

During a trial, a motion for judgment as a matter of law (“JMOL”) may be filed:

A

During a trial, a motion for judgment as a matter of law (“JMOL”) may be filed at any time before submission of the case to the jury. The moving party must specify in its motion the judgment sought and the law and facts on which the party is entitled to judgment. Note, however, that the nonmoving party must have been heard on the issue.

How well did you know this?
1
Not at all
2
3
4
5
Perfectly
12
Q

To receive a jury trial, a party generally must

A

-TLDR: 14 days after last pleading; serve on all parties Under Rule 38, to receive a jury trial a party must file a written demand and serve it on all the parties within 14 days after the service of the last pleading directed to the jury-triable issue. Otherwise, the right to a jury trial generally is deemed waived. Note, however, that the Supreme Court has held that, in the absence of compelling reasons to the contrary, a court should grant relief from the waiver if the issue is normally tried by a jury

How well did you know this?
1
Not at all
2
3
4
5
Perfectly
13
Q

Woman commenced an action in federal court, properly based on diversity, seeking rescission of a contract. The company answered, denying the principal allegations of the woman’s complaint and asserting a counterclaim against the woman for breach of contract. In addition, the company timely served a demand for a jury trial. The woman did not.

What are the roles of the judge and jury as finders of fact in the trial of the parties’ claims?

A

Jury will first determine issues relating to breach of contract claims; judge will determine remaining issues of remission if they are not already resolved by the jury. If legal and equitable claims are joined in one action involving common fact issues, the legal claim is tried first before the jury, and then the equitable claim is tried to the court. The jury’s finding on fact issues will bind the court in the equitable claim.

How well did you know this?
1
Not at all
2
3
4
5
Perfectly
14
Q

In a diversity case, federal trial courts are required to apply a _______ when considering a motion for a new trial based on the excessiveness of the verdict.

A

n a diversity case, federal trial courts are required to apply a state law standard when considering a motion for a new trial based on the excessiveness of the verdict. Federal trial courts may neither use a federal law standard, nor choose a state or federal law standard in the court’s discretion.

How well did you know this?
1
Not at all
2
3
4
5
Perfectly
15
Q

Accomplice Liability at Common Law

A

Specific Intent Crime - An accomplice is one who (i) with the intent to assist the principal and the intent that the principal commit the substantive offense (ii) actually aids, counsels, or encourages the principal before or during the commission of the crime. Absent a statute - mere knowledge is not enough

How well did you know this?
1
Not at all
2
3
4
5
Perfectly
16
Q

A homeowner decided to destroy his home by fire in order to collect the insurance. A neighbor’s house was located a short distance from the homeowner’s home. The homeowner knew that there was a strong wind blowing towards the neighbor’s home; while he did not want to burn the neighbor’s home, he nevertheless set fire to his own home. Fire dept put out the fire moments before it spread to the neighbor’s home, which suffered damage from smoke and soot. The jurisdiction’s arson statute covers burning one’s own dwelling as well as the dwelling of another, but is otherwise unchanged from the common law. If the homeowner is charged with attempted arson of the neighbor’s home, is he likely to be found guilty?

A

The homeowner will be found not guilty because he did not have the requisite mental state. To convict a person for an attempted crime, the prosecution must establish that the defendant had an actual specific intent to cause the harm prohibited by the statute and committed an act beyond mere preparation in furtherance of that intent. Those elements-specific intent and act-are required regardless of the mental state required by the target offense. A person who took a substantial step towards commission of the crime but was only reckless with respect to the target offense could not be found guilty of attempt. The homeowner did not intend to burn the neighbor’s home. Therefore, he cannot be guilty of attempted arson of the neighbor’s home.

How well did you know this?
1
Not at all
2
3
4
5
Perfectly
17
Q

Malice Aforethought - define

A
  1. intent to kill 2. intent to inflict serious bodily injury (and someone dies) 3. intent to commit a serious dangerous felony (BARRK) that results in death - everyone that participated guilty (unintentional) 4. depraved heart murder - unintentional killing that results from defendant’s reckless conduct
How well did you know this?
1
Not at all
2
3
4
5
Perfectly
18
Q

Involuntary Manslaughter - types and define

A
  1. voluntary manslaughter - intentional killing in the heat of passion where the defendant acts with adequate provocation 2. involuntary manslaughter - unintentional killing caused by criminally negligent conduct 3. misdemeanor manslaughter - misdemeanor assault/battery resulting in the death of another
How well did you know this?
1
Not at all
2
3
4
5
Perfectly
19
Q

To what torts does the doctrine of transferred intent apply?

A

The transferred intent doctrine allows an intent to commit a tort against one person to be transferred to the committed tort or to the injured person. It applies to (i) assault, (ii) battery, (iii) false imprisonment, (iv) trespass to land, and (v) trespass to chattels

How well did you know this?
1
Not at all
2
3
4
5
Perfectly
20
Q

Elements for the tort of Battery

A

The elements that must be proved to establish a prima facie case for battery are: (i) An act by the defendant that brings about harmful or offensive contact to the plaintiff’s person; (ii) Intent by the defendant to bring about the harmful or offensive contact; and (iii) Causation. Contact is harmful if it causes actual injury, pain, or disfigurement. Contact is offensive if it would be considered offensive to a reasonable person.

How well did you know this?
1
Not at all
2
3
4
5
Perfectly
21
Q

Elements for the tort of Assault

A

The prima facie case for assault requires (i) an act by defendant causing a reasonable apprehension in plaintiff of immediate harmful or offensive contact to plaintiff’s person, (ii) intent by defendant to bring about in plaintiff apprehension of that contact, and (iii) causation. For there to be apprehension, plaintiff must be aware of defendant’s act at the time that it is occurring.

How well did you know this?
1
Not at all
2
3
4
5
Perfectly
22
Q

Who may use force? -Landowner who has been tortiously dispossessed? - owner of chattel trying to recapture the chattel? - citizen trying to effect a misdemeanor arrest? -property owner trying to defend the property from tortious interference?

A

A landowner may not use force to regain real property after being tortiously dispossessed. Most states today do not allow resort to “self-help”; one who has been wrongfully excluded from possession of real property may bring an ejectment action or other summary procedure to recover possession. Hence, the owner who uses force to retake possession is liable for whatever injury she inflicts. (In former years, under the common law, a landowner tortiously dispossessed of real property could use reasonable force to regain possession, if she acted promptly upon discovery of the dispossession.) An owner of chattel may use force to recapture the chattel. An owner may use reasonable force to recapture a chattel when in “hot pursuit” of the tortfeasor. A demand for return of the chattel must be made before force is used, unless the demand would be futile or dangerous. However, force can be used only against the tortfeasor or a third party who knows that the chattel was tortiously obtained. If an innocent third party has obtained the chattel, the owner is no longer privileged to use force to effect a recapture of the chattel. A citizen may use force to effect a misdemeanor arrest. However, the citizen is allowed to use only the amount of force necessary to effect the arrest and never deadly force. A property owner may use force to defend the property from tortious interference. Although a property owner may use reasonable force to defend property, she may not use force that will cause death or serious bodily harm. Furthermore, one may not use indirect deadly force such as a trap, spring gun, or vicious dog when such force could not lawfully be directly used, e.g., against a mere trespasser.

How well did you know this?
1
Not at all
2
3
4
5
Perfectly
23
Q

Shopkeeper’s Privilege - Elements

A

(1) must have a reasonable belief as to the fact of theft, (2) must conduct the detention in a reasonable manner and detain the suspect for a reasonable period of time for the privilege to apply. By statute in some states and case law in others, shopkeepers have been given a privilege to detain someone suspected of shoplifting and thus avoid liability for false imprisonment. The following conditions must be satisfied: 1. There must be a reasonable belief as to the fact of theft; 2. The detention must be conducted in a reasonable manner and only nondeadly force can be used; and 3. The detention must be only for a reasonable period of time and only for the purpose of making an investigation. A shopkeeper is not required to notify the police in a reasonable amount of time to avoid liability for false imprisonment when detaining a suspect for shoplifting.

How well did you know this?
1
Not at all
2
3
4
5
Perfectly
24
Q

Privilege of Private Necessity

A

A person may interfere with the real or personal property of another when the interference is reasonably and apparently necessary to avoid threatened injury from a natural or other force and the threatened injury is substantially more serious than the invasion that is undertaken to avert it.

How well did you know this?
1
Not at all
2
3
4
5
Perfectly
25
Q

Using Force to Recapture Chattels

A

The defense of recapture of chattels is limited by the circumstances of the original dispossession. When another’s possession of the owner’s chattel began lawfully, the owner may use only peaceful means to recover the chattel. Force may be used to recapture a chattel only when in “hot pursuit” of one who has obtained possession wrongfully (e.g., by theft).

How well did you know this?
1
Not at all
2
3
4
5
Perfectly
26
Q

The defendant robbed a bank and fled in a getaway car driven by an accomplice, not realizing that one of the bundles of money he took had the serial numbers recorded and had a tiny tracking device attached to the wrapper. The bank’s security consultant obtained portable tracking equipment and was able to trace the bundle of money to the defendant’s house. The police were notified and they arrived at the defendant’s house a few hours after the robbery. They knocked on the door, announced their presence, and saw someone matching the description of the robber in the hallway. They entered and arrested the defendant, and then conducted a protective sweep of the house for the accomplice, who they believed had a gun. They did not find him, but while checking a closet, they discovered several of the bundles of money from the bank and a gun the defendant had used in the robbery. The police also discovered two clear plastic bags of what appeared to be marijuana sitting on top of a dresser. They seized the money, the gun, and the two bags. Later testing confirmed that the substance in the bags was marijuana. The defendant was charged with the bank robbery and with possession of the marijuana. At a preliminary hearing, he moves to suppress introduction of the money, gun, and marijuana. How should the court rule?

A

The court should suppress all of the evidence because it was the fruit of an unconstitutional arrest. As a general rule, the police must have an arrest warrant to effect an arrest of an individual in his own home. There is no general “emergency” exception to the warrant requirement. While police officers in hot pursuit of a fleeing felon or trying to prevent the destruction of evidence may sometimes make a warrantless search and seizure, the burden is on the government to show that one of those exceptions applies. Here, the police did not arrive at the defendant’s house in hot pursuit of the defendant, and there was no indication that the defendant might be destroying the money or other evidence; i.e., there were no circumstances precluding them from keeping the house under surveillance while they obtained a warrant. Hence, the arrest was unconstitutional. Because an arrest constitutes a seizure under the Fourth Amendment, the exclusionary rule applies, and evidence that is the fruit of the unconstitutional arrest may not be used against the defendant at trial. Here, all of the evidence was seized without a warrant, and none of the other exceptions to the warrant requirement are applicable. While the protective sweep that turned up the money and gun probably would have been within the bounds of a search incident to an arrest because the police had reason to believe an armed accomplice was present, the arrest in violation of the Fourth Amendment makes the search unlawful. Similarly, while the bags of marijuana were discovered in plain view, the police have to be legitimately on the premises for that exception to apply.

How well did you know this?
1
Not at all
2
3
4
5
Perfectly
27
Q

A fee simple owner of a restaurant provided in his will that the property should go on his death “in fee simple to my friend, but if during my friend’s lifetime my son has children and those children are alive when my friend dies, then to said living children.” When the owner died, the friend took over the restaurant. If the son has children and one or more of them are alive when the friend dies, who will take title to the restaurant at that time?

A

The son’s children, because their interest will vest, if at all, within a life in being plus 21 years. The interest given to the son’s children does not violate the Rule Against Perpetuities because the interest will vest, if at all, within 21 years after the life of the friend. Pursuant to the Rule Against Perpetuities, no interest in property is valid unless it must vest, if at all, not later than 21 years after one or more lives in being at the creation of the interest. In the case of a will, the perpetuities period begins to run on the date of the testator’s death, and measuring lives used to show the validity of an interest must be in existence at that time. Here, the interest given to any of the son’s children who are born during the friend’s lifetime and who survive the friend must vest, if at all, on the death of the friend (who is a life in being at the time of the owner’s death). Thus, this interest will vest, if it does vest, within 21 years after the friend’s life, and is therefore not in violation of the Rule Against Perpetuities. (A) is therefore incorrect; if one or more of the son’s children is alive at the time of the friend’s death, the friend’s heirs will get nothing because their fee simple will be divested. (B) incorrectly characterizes the interest of the son’s children as a possibility of reverter. A possibility of reverter is the future interest left in a grantor who conveys a fee simple determinable estate. Although under different circumstances the son’s children could acquire a possibility of reverter as heirs of the grantor (the owner), their interest in this case was conveyed directly to them in the owner’s will. (C) is incorrect because the interest of the son’s children is not vested. Their interest is a shifting executory interest rather than a remainder because it divests the fee simple estate of the friend and his heirs. The friend has a fee simple subject to an executory interest because the estate will remain with his heirs if none of the son’s children are alive when the friend dies. The friend’s death while the son’s children are alive divests the interest of the friend’s heirs; it is therefore a shifting executory interest rather than a remainder.

How well did you know this?
1
Not at all
2
3
4
5
Perfectly
28
Q

A pedestrian was injured in an auto accident caused by a driver. The pedestrian’s injuries included a broken nose and a broken toe. Not sure of the strength of her case, she sued the driver only for the injuries to her nose. She was awarded $15,000 in damages. Encouraged by this success, she now wishes to sue the driver for the injuries to her toe. May she sue the driver again?

A

The pedestrian may not sue the driver again because of merger. Merger occurs when the plaintiff wins; her cause of action is said to “merge” into the judgment such that she cannot relitigate the cause of action later. The court would hold that all of the personal injuries received by the pedestrian in one accident constitute a single cause of action, and that claim preclusion principles forbid relitigation.

How well did you know this?
1
Not at all
2
3
4
5
Perfectly
29
Q

A man and a woman were arrested and charged with a series of armed robberies. Each suspect was given Miranda warnings, and different interrogation teams questioned each suspect separately. Upon being questioned, the man told the police, “I’m not going to talk until I see a lawyer.” An officer responded, “You might want to reconsider, because your partner has already confessed, and she’s implicated you in the crimes.” The man then told the police that he wanted to talk to the woman privately. The police escorted the man to the woman’s cell, locked him in with her, and left. Unbeknownst to either of them, the police had bugged the woman’s cell and recorded both the man and the woman making self-incriminating statements during their meeting. The man made no further statements to the police on advice of counsel, whom he called immediately after his conversation with the woman. The man was put on trial first, and the prosecution sought to introduce into evidence tapes of the bugged conversation between the man and the woman. The defense made a motion to suppress the evidence. Should the court grant the motion to suppress?

A

The conversation should be suppressed because the police conduct violated the man’s Sixth Amendment right to counsel. The Sixth Amendment provides that in all criminal prosecutions a defendant has a right to the assistance of counsel at all critical stages after formal proceedings have begun. For Sixth Amendment purposes, a criminal prosecution begins when adversary judicial proceedings have commenced, such as the filing of formal charges in this case. Because custodial interrogation is a critical stage of prosecution, the Sixth Amendment is violated by post-charge interrogation unless the defendant has waived his right to counsel. Interrogation includes not only direct questioning, but also any other conduct by the police intended to elicit a response. The police conduct here (telling the man that the woman had implicated him and then bugging the conversation) constitutes prohibited interrogation. [See Maine v. Moulton (1985)] (A) is incorrect because the wiretap was not an illegal search under the Fourth Amendment. Wiretapping and other forms of electronic surveillance are subject to the Fourth Amendment prohibition of unreasonable searches and seizures. However, to have a Fourth Amendment right, a person must have a reasonable expectation of privacy with respect to the place searched or the item seized. In a different context, the Supreme Court has held that prisoners have no reasonable expectation of privacy in their cells or in any personal property that they have in their cells. [Hudson v. Palmer (1984)] Hence, neither defendant can assert a Fourth Amendment claim based on the wiretap, because they had no reasonable expectation of privacy in the jail cell. The fact that there was no expectation of privacy does not make choice (C) correct, however. Even though he probably cannot claim that the bugging was an unreasonable search under the Fourth Amendment, the man can claim that it was an interrogation in violation of his Sixth Amendment right to counsel, as discussed above. (D) is incorrect because it is irrelevant. The facts probably would not give rise to a Miranda violation in light of the Court’s ruling in Illinois v. Perkins (1990) that Miranda does not apply unless interrogation is by someone known to be a police officer (on the rationale that Miranda is merely a prophylactic rule designed to offset the coercive nature of a custodial interrogation by a police officer). In any case, Miranda rights and Sixth Amendment rights to counsel can only be waived knowingly, and so the man’s ignorance of the fact that the cell was bugged precludes a finding of waiver here.

How well did you know this?
1
Not at all
2
3
4
5
Perfectly
30
Q

Several members of a small terrorist group are on trial in federal court for conspiring to bomb a military installation. The prosecution would like to introduce the testimony of a military guard at one of the installation’s gates. The guard had been present when a bomb that was being planted by a member of the group had exploded prematurely. The guard will testify that she ran over to administer first aid to the member, who in great pain told her that his group was in the process of planting three other bombs in other areas of the military installation and was going to detonate them all at the same time to get publicity for their cause. The guard will also testify that the member disclosed the locations of the other bombs and the names of two other members of the group. The authorities were able to prevent the other bombings and arrest the other members of the group. The member died from his injuries. What is the best basis for allowing the guard to testify as to the member’s statements?

A

The member’s statements are admissible as a statement against interest. Under the Federal Rules, statements of a person, now unavailable as a witness, against that person’s pecuniary, proprietary, or penal interest when made are admissible as an exception to the hearsay rule. Here, the member’s statements implicating himself in the bombing conspiracy were against his penal interest when he made them; hence, they are probably admissible under that exception. (A) is wrong because for a co-conspirator’s statement to qualify as a vicarious statement of an opposing party, the statement must have been in furtherance of the conspiracy by a participant in it. Here, the member’s statements were not made in furtherance of the conspiracy but instead served to thwart its success. (C) is wrong because the member’s statements are not being used to show his then-existing state of mind but rather the scope of the conspiracy and the defendants’ participation in it. (D) is wrong because, even assuming that the member made the statements while believing his death was imminent (which the facts do not clearly establish), dying declarations are admissible under the Federal Rules only in a prosecution for homicide or in a civil action, and this case was neither of those.

How well did you know this?
1
Not at all
2
3
4
5
Perfectly
31
Q

A father executed a deed to his art gallery “to my daughter for her life, and on my daughter’s death to her children; provided, however, that if my daughter stops painting, to my brother.” The daughter has two children and is still painting. At the time of the grant, what is the best description of the interest of the daughter’s two children?

A

The daughter’s two children have a vested remainder subject to open and subject to complete divestment. A remainder is a future interest created in a transferee that is capable of taking in possession on the natural termination of the preceding estate. A remainder is vested if the beneficiaries are ascertainable and their taking in possession is not subject to a condition precedent. A vested remainder created in a class of persons that is certain to take but is subject to diminution by reason of others becoming entitled to take is a vested remainder subject to open. Vested remainders may be subject to total divestment if possession is subject to being defeated by the happening of a condition subsequent. Here, the daughter’s two children have a remainder because, on the expiration of the daughter’s life estate, they will be entitled to possession of the property. The remainder is not subject to a condition precedent and the beneficiaries are in existence and ascertained, so the remainder is vested, not contingent. The remainder is subject to open because the daughter may have more children. Finally, the remainder is subject to total divestment because the daughter’s children’s right to possession is subject to being defeated by the daughter’s ceasing to paint.

How well did you know this?
1
Not at all
2
3
4
5
Perfectly
32
Q

A vintner divided his vineyard into two parcels, drawing the boundaries so that the single well that had irrigated the entire vineyard fell on the border of the two properties. The vintner then conveyed the eastern parcel to his friend by a deed that contained the following covenant: “If the well located on the boundary of the eastern and western parcels continues to be used for irrigation purposes and becomes in need of repair or replacement, the grantee, his heirs, and assigns and the grantor, his heirs, and assigns each promise to pay one-half of the cost of such repair or replacement. This covenant shall run with the land.” The deed from the vintner to the friend was not recorded, and the vintner did not record a copy of the deed with the records for the western parcel. The friend later sold the eastern parcel to a farmer. The farmer’s deed did not contain the covenant about the well. After 15 years of use by the owners of both the eastern and western parcels, the well began to fail. The farmer took it upon himself to have the well repaired at a cost of $30,000. About two weeks later, the farmer discovered the deed from the vintner to the friend in some old files. By this time, the western parcel had passed to the vintner’s son by inheritance and again to the son’s daughter by inheritance from the now-deceased son. The daughter knew nothing of the covenant concerning the well. The farmer presented the daughter with the bill for the well repair with a copy of the vintner/friend deed and a note that said he expected to be reimbursed for $15,000. The daughter refuses to pay, and the farmer sues. The jurisdiction has a 10-year statute of limitations for acquiring property by adverse possession, and the following recording statute: “Any conveyance of an interest in land shall not be valid against any subsequent purchaser for value, without notice thereof, unless the conveyance is recorded.” For whom is the court most likely to rule?

A

The farmer will most likely prevail in his suit for one-half the cost of the well repairs because the covenant runs with the land. When a covenant runs with the land, subsequent owners of the land may enforce or be burdened by the covenant. If all of the requirements for the burden to run are met, the successor in interest to the burdened estate will be bound by the arrangement as effectively as if he had himself expressly agreed to be bound. To be bound: (i) the parties must have intended that the covenant run with the land; (ii) the original parties must have been in horizontal privity; (iii) the succeeding party must be in vertical privity with the original promisor; (iv) the covenant must touch and concern the land; and (v) generally, the burdened party must have actual or constructive notice of the covenant. Here, the intent is shown by the express language of the covenant, which says that it is intended to run with the land. Even without that language, the use of the words “heirs” and “assigns” would show the intent for the covenant to run. The original parties were in horizontal privity because at the time the vintner entered into the covenant, he and the friend shared an interest in the land independent of the covenant—as grantor and grantee. The daughter is in vertical privity with the vintner because she holds the entire interest in the western parcel held by the vintner. The covenant touches and concerns the land because promises to pay money to be used in a way connected with the land are held to touch and concern the property. Because the daughter was unaware of the covenant, the required notice seems to be missing. While it is generally true that the owner of the burdened land must have notice, it should be remembered that the requirement is a function of the recording statute. (At common law, the covenant was enforceable in an action for damages regardless of notice; this was changed by the recording statutes.) However, because the daughter is a donee (an heir) and not a bona fide purchaser, she is not protected by the recording statute and thus is subject to the covenant even without notice. For that reason, (A) is wrong. (B) is wrong because the farmer’s possession does not satisfy several of the requirements for adverse possession. Because the farmer had a legal right to use the well, his use was not adverse or hostile to the rights of the vintner’s son and the son’s daughter, but was rather permissive. The farmer’s possession also fails the exclusivity requirement because the facts state that the well was used to irrigate both parcels for most of the statutory period. (D) is wrong because the farmer’s status as a bona fide purchaser has no effect on his ability to enforce the covenant. A successor in interest to the original promisee may enforce the covenant (enjoy the benefit) if there was intent and vertical privity, and the covenant touches and concerns the land. Notice is not required for the benefit to run. Thus, because the above requirements are met here, the farmer may enforce the covenant regardless of his status as a bona fide purchaser. Had the farmer taken the property as a donee, the above analysis would be the same.

How well did you know this?
1
Not at all
2
3
4
5
Perfectly
33
Q

A plumber working for a company providing plumbing services to commercial and industrial establishments was required to be “on call” for emergency plumbing services 24 hours a day, and was required to drive his company van home each night so he would have all of his tools and equipment at hand for any calls. However, he was not permitted to use the company van for personal errands. On his way home one afternoon, he took a detour toward a supermarket a few blocks away to pick up some items for dinner. While entering the supermarket parking lot, he drove negligently and struck a pedestrian, seriously injuring him. The pedestrian filed suit against the plumber’s company in a jurisdiction that maintains traditional common law rules regarding contribution and indemnity, and the jury awarded him $100,000 in damages, which the company paid. If the company sues the plumber to recoup its loss in the lawsuit, which party will prevail?

A

The company can recover 100% of the judgment under common law indemnity rules. The principle of indemnity permits a shifting between the tortfeasors of the entire loss (i.e., the payment made to satisfy plaintiff’s judgment). This is in contrast to contribution, which apportions the loss among those who are at fault. Indemnity is available in vicarious liability situations, where one party is held liable for damages caused by another simply because of his relationship to that person. Hence, an employer such as the plumber’s company that has been held vicariously liable under the doctrine of respondeat superior can obtain indemnification from the employee (the plumber) whose conduct actually caused the damage. (B) is incorrect because the company need not show that the plumber breached a company rule before it can obtain indemnity. The fact that the plumber’s negligence caused the injury and that the company was liable for the judgment solely because of its relationship to the plumber permits indemnification here. (C) is incorrect because vicarious liability is one of the most common areas where indemnity is available. (D) is incorrect because the company’s requirement that the plumber be on call 24 hours a day merely establishes that the company will be vicariously liable for the plumber’s negligence; it does not bar the company from recovering from the plumber because the plumber’s negligence actually caused the damage.

How well did you know this?
1
Not at all
2
3
4
5
Perfectly
34
Q

What will a mistake (both parties) do to a contract?

A

When both parties entering into a contract are mistaken about existing facts relating to the agreement, the contract may be voidable by the adversely affected party if (i) the mistake concerns a basic assumption on which the contract is made; (ii) the mistake has a material effect on the agreed-upon exchange; and (iii) the party seeking avoidance did not assume the risk of the mistake.

How well did you know this?
1
Not at all
2
3
4
5
Perfectly
35
Q

What happens at a Rule 26(f) conference? When is it?

A

At a Rule 26(f) conference, the parties must confer to consider their claims and defenses, the possibility of settlement, initial disclosures, and a discovery plan. The parties then must submit a proposed discovery plan to the court, and the plan must address the timing and form of required disclosures, the subjects on which discovery may be needed, the timing of and limitations on discovery, and relevant orders that may be required of the court. At the start of the case, before discovery requests are sent. (CANNOT SERVE DISCOVERY UNTIL AFTER 26(f) (only requests for production))  Unless court order says otherwise, at least 21 days before scheduling order, parties “meet and confer.”

How well did you know this?
1
Not at all
2
3
4
5
Perfectly
36
Q

The governor of an arid western state owned a vacation home and permitted his son to have a party there. At the end of the night the son failed to properly extinguish a bonfire that he and his friends had built, and within a few hours, wind-blown cinders had spread the fire to the trees east of the lodge. At the same time several miles away, a worker at a lumber mill was making emergency repairs to a pipe running between two mill buildings. He did not notice some of the sparks from his welding torch land in a pile of dried lumber and catch fire, and he failed to check the area after he was finished. By the time the fire was noticed by another employee, it was out of control. The wind blew both fires toward a landowner’s hunting lodge. They merged a mile away and shortly thereafter totally consumed the lodge. For political reasons, the landowner did not bring a lawsuit against the governor or his son. He did, however, file a lawsuit against the lumber mill, alleging that its employee’s negligence caused the destruction of his lodge. Evidence at trial established that either fire alone would have destroyed the lodge as well. Can the landowner recover from the lumber mill?

A

The landowner can recover the full amount of his damages from the lumber mill because the negligence of its employee caused the destruction of the lodge. Before a defendant will be liable for a breach of duty to the plaintiff, it must be shown that the breach was the actual and proximate cause of the injury. The general test for determining whether an act or omission is the actual cause of the injury is the “but for” test, i.e., whether the injury would not have occurred but for the act or omission. Under certain circumstances, however, the “but for” test is inadequate to determine actual cause. Where several causes combine to bring about an injury—and any one alone would have been sufficient to cause the injury—the actual cause requirement is satisfied if defendant’s conduct was a substantial factor in causing the injury. Under this analysis, the fire started by the lumber mill employee was an actual cause of the destruction of the landowner’s lodge because it was a substantial factor in causing the harm. It was also a proximate cause of the harm because no intervening forces broke the causal connection between the act and the harm. Because its employee was acting within the scope of his employment when he caused the fire to start, the lumber mill is vicariously liable for the injury that resulted. (A) is incorrect because the “but for” test is not applicable to these facts. Under that test, neither fire would be the actual cause of the harm because, looking at either fire alone, the harm would have occurred even without it. However, under the substantial factor test, both fires are actual causes of the injury. (B) is incorrect even though it is true that the damage is indivisible. The landowner can still recover from the lumber mill even if he does not sue the other tortfeasor. (D) is incorrect because joint and several liability rules allow the landowner to recover his full damages from the lumber mill. Where two or more tortious acts combine to proximately cause an indivisible injury to plaintiff, each tortfeasor will be jointly and severally liable for that injury, even though each defendant acted entirely independently. The effect of joint and several liability is that the plaintiff may recover the entire amount of the damages from any tortfeasor, who then may have a right of contribution from the other tortfeasor. Hence, even though the negligence of another tortfeasor was also an actual cause of the destruction of the landowner’s lodge, the landowner is entitled to recover all of his damages from the lumber mill.

How well did you know this?
1
Not at all
2
3
4
5
Perfectly
37
Q

A landowner validly conveyed a small office building to the Green Party “as long as they use it for operating quarters until the next presidential election.” After the next presidential election, which was in three years, the building would go to a private organization that monitors and prepares comprehensive listings of gas prices throughout the country. A year after the conveyance, the landowner died, validly devising all of her property to her son. Although this jurisdiction is a common law jurisdiction with respect to all real property considerations, the state’s probate laws provide that future interests or estates in real property may be passed by will or descent in the same manner as present or possessory interests. Last week, the Green Party and the gas monitoring organization joined together to sell the office building in fee simple absolute to a developer. The son filed suit to prevent the sale of the property to the developer. In this action, who should prevail?

A

The son may enjoin the sale because he has an interest in the property. A fee simple determinable is an estate that automatically terminates on the happening of a stated event. The Green Party’s interest in the office building is a fee simple determinable because it lasts as long as the Party is using the building for operating quarters. However, the grant does not provide for the contingency of the Green Party ceasing to use the building as operating quarters before the next presidential election. This gap would be filled by a possibility of reverter retained by the landowner. Because the landowner passed that interest to her son in her will, there can be no contract to sell the property without his signature. Note: Although the gas monitoring organization appears to have an indefeasibly vested remainder (i.e., it is created in an ascertained company, is certain to become possessory, and is not subject to being defeated, divested, or diminished in size), its interest is not capable of taking on the natural termination of the preceding estate and so is characterized as a springing executory interest. (A) is wrong because the son also has an interest in the land. (B) is wrong because the interest in the office building will pass to the gas monitoring organization, if at all, within 21 years. (C) is wrong because the Green Party is not prohibited from transferring any interest; it could pass a defeasible fee.

How well did you know this?
1
Not at all
2
3
4
5
Perfectly
38
Q

A state statute defines all murders as second degree murders unless deliberation and premeditation can be shown, in which case the crime is elevated to first degree murder. Manslaughter is defined as at common law. The defendant, just having been served with divorce papers, decided to drown his sorrows at the local pub. After drinking heavily and becoming very intoxicated, the defendant became enraged when another patron spilled a drink on him. He took a nearby ashtray and smashed it over the patron’s head, killing him instantly. The crimes below are listed in descending order of seriousness. What is the most serious crime of which the defendant could be convicted? Press Enter or Space to submit the answer A Murder in the first degree. Correct B Murder in the second degree. Incorrect C Voluntary manslaughter. D Involuntary manslaughter.

A

The most serious crime that the defendant would be convicted of is murder in the second degree. The jurisdiction defines murder in the first degree as deliberate premeditated murder, whereas all other types of killings are defined as at common law. Deliberation and premeditation requires some time of cool reflection on the idea of killing. In the instant case, there are no facts indicating that the defendant coolly reflected on the idea of killing. The facts indicate an impulsive killing rather than any type of deliberate, premeditated killing. Furthermore, the facts indicate that the defendant was very intoxicated, which would serve as a basis for reducing the crime to second degree murder. Thus, (A) is incorrect. The state defines murder in the second degree just like common law murder. At common law, murder required malice; i.e., (i) the intent to kill, (ii) the intent to inflict great bodily injury, (iii) reckless indifference to an unjustifiably high risk to human life, or (iv) the intent to commit a felony. Here, the reckless indifference element arguably could be satisfied. By smashing a heavy ashtray over the other patron’s head, the defendant unjustifiably disregarded that the blow could be a killing blow. Neither would intoxication be a defense, as there would be no specific intent-to-kill requirement under this type of analysis. (D) is incorrect. Involuntary manslaughter is a killing committed with criminal negligence or during the perpetration of some unlawful act not encompassing a felony for felony murder. Certainly, striking another with a heavy object would constitute criminal negligence sufficient for conviction. However, involuntary manslaughter is a lesser crime than murder in the second degree, and the call of the question asks for the most serious crime of which the defendant could be convicted, making (B) a better choice than (D). (C) is also incorrect. Voluntary manslaughter is a killing committed under the duress of an adequate provocation, and it requires (i) a provocation sufficient to arouse the sudden and intense passion in the mind of an ordinary person such as to cause him to lose self-control; (ii) the defendant to be in fact provoked; (iii) an insufficient time to cool off; and (iv) the defendant did not in fact cool off. It is unlikely that having a drink spilled on him would cause an ordinary person to commit murder. As a result, such a provocation is not adequate to reduce the killing to voluntary manslaughter.

How well did you know this?
1
Not at all
2
3
4
5
Perfectly
39
Q

A plaintiff sued an auto manufacturer for negligence after a car accident involving the plaintiff’s car that was made by the auto manufacturer. Sixty days after service of the complaint and 40 days after service of the manufacturer’s answer that contained no counterclaim, the plaintiff filed a motion seeking to file an amended complaint adding a claim for strict products liability against the auto manufacturer stemming from the same incident. The statute of limitations for strict products liability claims expired one week before the motion was filed. How should the court rule on the plaintiff’s motion?

A

The court should grant the motion. Federal Rule of Civil Procedure 15 states that leave of court (to grant motions to amend) is to be “freely given when justice so requires.” The rule does not provide any clear date when amendments are no longer permissible, although later amendments obviously would be less fair and less likely to be considered in the interest of justice. Additionally, for statute of limitations purposes, proposed claims may be considered to “relate back” to the date of the original pleading in which the claim was made under Rule 15(c). That is critical here because, although the statute of limitations for the proposed products liability claim had expired at the time the motion was made, the amended complaint asserting a products liability claim relates back to the original filing because that claim stems from the same facts alleged in the original complaint, and the statute of limitations had not expired at the time of the original filing. (A) is incorrect because a plaintiff may amend the complaint once as a matter of course (i.e., without court intervention) not later than 21 days after service of the auto manufacturer’s answer. Here, 40 days have passed since the auto manufacturer filed its answer, so amending as a matter of course is not permitted. (C) is incorrect. Although the proposed claim would be futile if filed in an original complaint, here, because the claim can relate back to a complaint filed at a time when the proposed claim was still viable, the proposed claim is not considered futile. (D) is incorrect by suggesting that the motion is not timely. Although plaintiff can no longer amend as a matter of course, there is no absolute date pursuant to which a motion to amend is untimely.

How well did you know this?
1
Not at all
2
3
4
5
Perfectly
40
Q

Using his cellphone, a witness recorded a speeding driver hitting a pedestrian. The witness sold the recording to the driver. The driver then gave the recording to his attorney. After the pedestrian filed suit against the driver, the pedestrian sent a discovery request to the driver requesting that he produce “all items that show or describe the accident.” Does he have to provide the video?

A

The driver must provide the video. Parties are entitled to discovery that fits under Rule 26(b) (1), which includes “any nonprivileged matter that is relevant to any party’s claim or defense.” In other words, even if the matter is only relevant to the opposing party, it would still be covered. Additionally, Rule 34 requires a party to produce relevant physical material, including electronically stored information, such as the recording here. There is no exception to relevance for matter that is damaging to a party, so (B) is incorrect. (Being damaging to the defendant’s defense might be reason not to disclose it as an initial disclosure because the defendant would not use the recording to support his defense, but the recording would have to be disclosed on a proper request.) (C) is also incorrect. There is no exception from the scope of discovery for items that were purchased; discovery encompasses all items in a party’s possession or control. Furthermore, although the recording is in the attorney’s possession, this should be interpreted as under the party’s control. (D) is incorrect because, while it states the rule for when work product must be provided, there is no work product involved. Mere possession by an attorney of factual information does not constitute work product that is exempt from discovery. Rather, work product is material created by a party or a representative (such as an attorney) prepared in anticipation of litigation. This recording was not created by the party or any representative of the party, and therefore it is not work product.

How well did you know this?
1
Not at all
2
3
4
5
Perfectly
41
Q

A minor league ballplayer hit a fly ball over the wall and out of the park during a game and struck a woman riding along the adjacent street on a bicycle. The woman sued the ballplayer for negligence. The woman alleged that the ballplayer had often hit balls out of the park and was aware that he had previously struck a car driving down the street. Assuming the woman’s allegations are correct, is she likely to prevail?

A

The woman will not prevail because the ballplayer’s conduct did not breach any duty owed to the woman. The prima facie case for negligence requires the plaintiff to show: (i) a duty on the part of the defendant to conform to a specific standard of conduct for the protection of the plaintiff against an unreasonable risk of injury; (ii) breach of that duty by the defendant; (iii) that the breach was the actual and proximate cause of the plaintiff’s injury; and (iv) damage to the plaintiff’s person or property. Whenever a person engages in an activity, he is under a legal duty to act as an ordinary, prudent, reasonable person engaged in the same or similar activity. If a defendant’s conduct creates an unreasonable risk of injury to persons in the position of the plaintiff, the general duty of care extends from the defendant to the plaintiff. Here, there is some risk of injury to those outside the park based on previous occurrences. However, there are no precautions that the ballplayer could have undertaken that would not directly conflict with the requirements of his job and harm his career. On balance, the burden on the ballplayer to avoid any risk of injury far outweighs the likelihood of a ball hit by him clearing the wall and causing injury to someone. Hence, the ballplayer’s conduct did not create an unreasonable risk of injury to the woman and he did not breach a duty of care owed to her. Thus, she is not likely to prevail.

How well did you know this?
1
Not at all
2
3
4
5
Perfectly
42
Q

MPC Insanity Test

A

Pursuant to the Model Penal Code, a defendant is entitled to acquittal if he suffered from a mental disease or defect and as a result lacked substantial capacity to either: (i) appreciate the criminality of his conduct; or (ii) conform his conduct to the requirements of law. Note that the Model Penal Code test combines the M’Naghten and irresistible impulse tests.

How well did you know this?
1
Not at all
2
3
4
5
Perfectly
43
Q

Durham Insanity Test

A

if the jurisdiction followed the Durham insanity test = a defendant is entitled to acquittal if his crime was the product of mental disease or defect.

How well did you know this?
1
Not at all
2
3
4
5
Perfectly
44
Q

M’Naughten Insanity Rule

A

presents a valid defense under the M’Naghten rule, which provides for acquittal if a disease of the mind caused a defect of reason, such that the defendant lacked the ability at the time of his actions to either: (i) know the wrongfulness of his actions; or (ii) understand the nature and quality of his actions.

How well did you know this?
1
Not at all
2
3
4
5
Perfectly
45
Q

Irresistible Impulses Insanity Test

A

irresistible impulse test provides for acquittal if, because of mental illness, the defendant was unable to control his actions or to conform his conduct to the law.

How well did you know this?
1
Not at all
2
3
4
5
Perfectly
46
Q

MBE’s Definition of Assault

A

For purposes of the MBE, an assault is either (i) an attempt to commit a battery, or (ii) the intentional creation, other than by mere words, of a reasonable apprehension in the mind of the victim of imminent bodily harm.

How well did you know this?
1
Not at all
2
3
4
5
Perfectly
47
Q

a son executed and delivered to the creditor a warranty deed to a large tract of undeveloped land. The creditor promptly recorded the deed. Shortly thereafter, she built a house on the property and has lived there ever since. The son never actually owned the land. It belonged to his father, but the father had promised to leave the property to the son. Later, the father died and his will devised the property to the son. Pressed for money, the son then sold the land to an investor by warranty deed, which the investor promptly recorded. Although the investor paid full value for the property, he purchased it strictly for investment and never visited the site. He therefore did not realize that the creditor was living there, and knew nothing of the son’s earlier deed to the creditor. The jurisdiction in which the land is located has the following statute: “A conveyance of an estate in land (other than a lease for less than one year) shall not be valid against any subsequent purchaser for value without notice thereof unless the conveyance is recorded.” Which of the following is the most likely outcome of a quiet title action brought by the creditor against the investor? A The creditor prevails, because the son had no title to convey to the investor. B The creditor prevails, because the investor was not a purchaser for value without notice of the creditor’s interest. C The investor prevails, because under the doctrine of estoppel by deed, title inures to the benefit of the original grantee only as against the grantor. D The investor prevails, because under the recording acts, the deed from the son to the creditor was not in the chain of title and hence did not constitute notice to the investor.

A

The creditor will prevail in a suit to quiet title because the investor had notice of the creditor’s interest in the property and, thus, is not a bona fide purchaser for value. When a grantor purports to convey property that he does not own, his subsequent acquisition of title to that property vests in the grantee under the doctrine of estoppel by deed. Most courts, however, hold that this is personal estoppel, which means that title inures to the grantee’s benefit only as against the grantor, not a subsequent bona fide purchaser. If the grantor transfers his after-acquired title to an innocent purchaser for value, the bona fide purchaser gets good title. There is a split of authority as to whether the original grantee’s recordation of the deed imparts sufficient notice to prevent a subsequent purchaser from being a bona fide purchaser, but the majority view is that it does not because it is not in his chain of title. Thus, it is not the fact that the creditor recorded that prevents the investor from being a bona fide purchaser. The fact that the creditor built a home and was living on the property gave the investor constructive notice of her interest. A title search is not complete without an examination of possession. If the possession is unexplained by the record, the subsequent purchaser is charged with knowledge of whatever an inspection of the property would have disclosed and anything that would have been disclosed by inquiring of the possessor. Therefore, the investor is charged with knowledge of the creditor’s possession and with what the creditor would have told him about her possession; i.e., that the property was conveyed to her by the son prior to his conveyance to the investor. Consequently, the investor does not qualify as a bona fide purchaser, and (C) is an incorrect choice. (A) is incorrect because, although the son is estopped to deny that he acquired title for the benefit of the creditor, he could have conveyed valid title to a subsequent purchaser for value who had no notice of the creditor’s interest. Therefore, it is not exactly correct to say that the son had no title to convey. (D) is incorrect because the investor will not prevail. It is true that under the recording acts the creditor’s deed was not in the chain of title, but the investor still does not qualify as a bona fide purchaser. The investor is on inquiry notice arising from the creditor’s possession of the property.

How well did you know this?
1
Not at all
2
3
4
5
Perfectly
48
Q

A boy mowing his lawn noticed a strong vibration from the engine but continued to mow. The engine housing suddenly broke apart and pieces flew off the lawnmower. One piece struck the boy in the head, seriously injuring him. The boy’s mother was inside the house and heard yelling from the backyard. She went to the window and saw her son lying on the ground by the lawnmower and a friend of his kneeling over him. She became very upset and fainted. Subsequent investigation showed that a negligent repair by a local mechanic caused the engine housing to shatter. The mother brought a lawsuit against the mechanic, seeking recovery for her son’s injury and the emotional distress she suffered. Can the mother recover damages for her emotional distress? A No, because her son’s continuing to mow after noticing the vibration was a superseding cause of the harm. B No, because the mother was not within the zone of danger from the mechanic’s negligence. C Yes, because the mother was closely related to someone in the zone of danger from the mechanic’s negligence. D Yes, because her son was injured by the mechanic’s negligence.

A

The mother cannot recover damages for her emotional distress. A duty to avoid negligent infliction of emotional distress may be breached when the defendant creates a foreseeable risk of physical injury to the plaintiff. The mother might try to assert two theories in support of her emotional distress claim, but she is unlikely to prevail on either one. First, she could claim distress flowing from fear for her own safety, but she may prevail only if the defendant’s negligence placed her in a zone of danger. Here, because she was safely inside her home and quite distant from the mower when it exploded, she was not in a zone of danger. Alternatively, she could claim distress flowing from her anguish at seeing her son injured. However, for a bystander who is outside the zone of danger from the risk of physical injury but who suffers emotional distress from seeing the defendant negligently injure another, most states allow recovery only if: (i) the plaintiff and the person injured by the defendant are closely related; (ii) the plaintiff was present at the scene of the injury; and (iii) the plaintiff personally observed or perceived the event. Here, while she is related to her son, who was injured by the mechanic’s negligence, she was not present at the scene of the injury and did not personally observe or perceive the event. Hence, she cannot recover damages for negligent infliction of emotional distress. (A) is incorrect. Under proximate cause rules, a third party’s negligence that contributes to the plaintiff’s harm will not be considered a superseding force where it is within the foreseeable risk created by the defendant’s negligence. While the son may have been negligent in continuing to use the mower, this conduct would not cut off the defendant’s liability to the mother if she otherwise could recover for her distress. (C) is incorrect because the plaintiff’s close relationship with the injured person is only one of the requirements for a plaintiff outside the zone of danger to recover emotional distress damages. Because the mother was not present at the scene and did not observe the event, she cannot recover. (D) is incorrect. While the mother can recover on her son’s behalf for his injury, she cannot recover for her emotional distress, as discussed above.

How well did you know this?
1
Not at all
2
3
4
5
Perfectly
49
Q

A hockey player who was playing in the final game of the season before a hostile crowd in the opponent’s packed stadium had an opportunity to get his team into the playoffs, but he missed a shot into an open net as the horn sounded, ending the game. As the crowd cheered and jeered, the puck bounced back to him and he shot it in anger toward the stands. A fan who had been looking the other way turned back toward the rink just in time to be struck in the face by the puck. He suffered a broken nose and a severe gash under his eye. After the game, the league commissioner fined the player for violating league rules by intentionally directing the puck out of the playing area. If the fan sues the player for battery, will the fan likely prevail? A No, because by attending a hockey game, the fan assumed the risk of pucks being shot into the stands. B No, because the player did not have the intent to strike the fan with the puck. C Yes, because the player knew that it was substantially certain that a fan would be hit by the puck. Incorrect D Yes, because the player violated league rules by intentionally shooting the puck out of the playing area.

A

Answer Discussion - Incorrect The fan will prevail in his battery action because the player had the requisite intent for battery. A prima facie case for battery requires plaintiff to prove (i) an act by defendant that brings about a harmful or offensive contact to the plaintiff’s person, (ii) intent on defendant’s part to bring about harmful or offensive contact, and (iii) causation. The intent element is satisfied as long as the defendant knew with substantial certainty that the harmful or offensive contact would result. Here, the player’s conduct caused a harmful contact to the fan, because the player set into motion the force that caused injury to the fan. His intentionally shooting the puck into the crowded stands is enough to establish that he knew with substantial certainty that the puck would strike a spectator. (Note that even if he only intended to cause apprehension of contact, which is the intent for assault, this intent would suffice for liability for battery under the doctrine of transferred intent.) (A) is wrong because assumption of risk is not a defense to intentional torts. The fan may have assumed a risk of injury from a hockey puck’s being accidentally or even negligently shot into the stands, but he did not assume the risk of a player’s intentionally shooting the puck at a spectator. (B) is wrong because the player need not have intended to strike that fan to be liable. As long as he knew with substantial certainty that a fan would be struck, he is liable even if he did not single out the fan as the target. (D) is wrong. The fact that the player violated league rules when he shot the puck into the stands tends to establish only that a spectator does not impliedly consent to a puck’s intentionally being shot at him, thus negating the defense of consent in a battery action. It does nothing to establish that the player did have the intent to commit a battery.

How well did you know this?
1
Not at all
2
3
4
5
Perfectly
50
Q

A dog owner lived next door to a day care center. Because he had a large yard and there were no applicable zoning restrictions, he installed a kennel and began training attack dogs to sell to businesses. As soon as he opened the business and posted signs in front advertising the exceptional ferocity of the dogs, some parents who had children enrolled in the day care center became alarmed at the prospect of the dogs right next to the yard where the children played, especially because the children could see and hear the dogs being taught to attack people. Within a few months of the dogs’ arrival next door, the owner of the day care lost 10% of her enrollment. If the day care owner brings a nuisance action against the dog owner, what will be the most critical factual issue that the trier of fact must resolve to determine who should prevail? A Whether the day care owner suffered other damages in addition to her economic losses. B Whether the day care owner’s use of her property makes her business abnormally sensitive to the presence of the dogs. C Whether the dog owner conducted his business with reasonable care. D Whether the dog owner was apprised of the day care owner’s concerns and did nothing to alleviate them.

A

The determining factor for the day care owner in prevailing will be whether her use of the property is abnormally sensitive to the presence of the dogs. Nuisance is an invasion of private property rights by conduct that is either intentional, negligent, or subject to strict liability. Strict liability will be the basis for a nuisance action (sometimes called an “absolute” nuisance or a “nuisance per se”) when wild animals or abnormally dangerous domestic animals are involved, or when defendant is engaged in an abnormally dangerous activity. Thus, dogs known by their owner to be vicious may create a private nuisance when they interfere with the use and enjoyment of the land next door, and the owner may be subject to strict liability because of his knowledge of the dogs’ dangerous propensities. [See Restatement (Second) of Torts §822, comment j] For the presence of the dogs to be an actionable nuisance, however, they must result in a substantial interference with the day care owner’s use of her land. The interference will not be characterized as substantial if it is merely the result of plaintiff’s specialized use of her own property. [See Foster v. Preston Mill Co., 268 P.2d 645 (1954)-D not strictly liable for blasting operations that caused female mink on P’s ranch to kill their young in reaction to the vibrations] Hence, (B) states the most critical factual issue. (A) is incorrect because the day care owner does not need to establish other types of damages to recover once she has established that the dog owner’s activity is an actionable interference with the use and enjoyment of her land. (C) is incorrect because the exercise of reasonable care by the dog owner is irrelevant; the day care owner’s nuisance action arises from an activity for which the dog owner is strictly liable. (D) is incorrect because the dog owner’s knowledge of his interference with the day care owner’s use of her property would only establish that his conduct might also be an intentional nuisance, which would require the day care owner to show unreasonableness, i.e., that her injury outweighs the utility of his conduct. She does not need to make that showing for a nuisance action based on strict liability.

How well did you know this?
1
Not at all
2
3
4
5
Perfectly
51
Q

A landowner included in his will a provision giving “all of my property, both real and personal, wherever situated, to my widow for life, and after her death to any of our children who may survive her.” What is the gift to the children?

A

A contingent remainder. A remainder is a future interest created in a transferee that is capable of taking in present possession on the natural termination of the preceding estate created in the same disposition. Note that, as a rule of thumb, remainders always follow life estates. A remainder will be classified as contingent if its taking is subject to a condition precedent, or it is created in favor of unborn or unascertained persons. Here, the interest in the children follows a life estate and is a remainder because it is capable of taking in possession on the natural termination of the preceding estate. It is subject to the condition precedent of surviving the landowner’s widow and, additionally, is in favor of unascertained persons (the children who survive the landowner’s widow will not be ascertained until her death). Thus, the interest is a contingent remainder.

How well did you know this?
1
Not at all
2
3
4
5
Perfectly
52
Q

When should the court grant Motion for Summary Judgment

A

FRCP Rule 56: Summary judgment may (DISCRETIONARY) be granted if, from the pleadings, affidavits, and discovery materials, it appears that there is NO genuine dispute of material fact and the moving party is entitled to judgment as a matter of law. The court may not decide disputed fact issues on a motion for summary judgment; if there is a genuinely disputed material fact (meaning a dispute backed by evidence on both sides of the issue), the case must go to trial. For example, should be granted if the plaintiff’s evidence for the PF case not been contradicted by any admissible evidence. – in the DISCRETION of court  Party must show • No genuine dispute of material fact • Entitled to judgment as a matter of law o Any party can move for this no later than 30 days after close of discovery. o Can the motion be for “partial” judgment—e.g., as to liability, but allow the case to go to trial on damages • Court can look at evidence - The court views that evidence in the light most favorable to the nonmoving party. o The parties proffer the evidence, usually (1) affidavits or (2) declarations or (3) deposition testimony or (4) interrogatory answers – ALL SIGNED UNDER OATH; cannot be second-hand knowledge/hearsay; judge does not assess credibility on summary judgment

How well did you know this?
1
Not at all
2
3
4
5
Perfectly
53
Q

A man and a woman were in a two-car traffic accident. Immediately after the accident, the man was treated by a physician at a nearby hospital’s emergency room. The man later filed a negligence action against the woman in federal district court, seeking compensatory damages. The woman now seeks discovery regarding the emergency room physician’s observations, opinions, and treatment of the man. Is the woman entitled to discovery regarding that information?

A

The woman is entitled to discovery regarding the physician’s observations, opinions, and treatment of the man because the physician developed opinions about the man’s injuries for purposes other than litigation or trial.

How well did you know this?
1
Not at all
2
3
4
5
Perfectly
54
Q

After picking up a load of hazardous chemical waste, a truck driver for a waste management company set out on the road to his next stop. However, he had failed to secure the latch on the back panel of the truck. Consequently, the panel opened while the truck was on the road, and a metal canister full of chemical waste fell onto the road. A car struck the canister, causing the car to veer off the road and injure the driver. The driver filed suit against the company for his injuries. The jurisdiction in which the above events took place has adopted a rule of partial comparative negligence. At trial, the driver of the car admitted that he had momentarily taken his eyes off the road to look at his speedometer. When he had looked up again, the canister was there and he could not stop in time. The jury found that the company, through its truck driver, had acted willfully and wantonly and was 90% at fault, while the driver of the car was 10% at fault. The driver filed a motion for judgment notwithstanding the verdict, seeking recovery for 100% of his damages. If the judge grants the motion, what is the most likely reason? A A plaintiff’s comparative negligence is not taken into account in cases of willful and wanton conduct by the defendant. B A state ordinance mandating motorists to stay within the posted speed limit requires as a matter of law an occasional glance at the speedometer. C The company was more than 50% at fault. D The company was engaged in an abnormally dangerous activity.

A

If the driver was effectively required by statute to take an occasional quick look at his speedometer to make sure that he was complying with appropriate speed limits, then his momentary glance at the speedometer in the instant case would, as a matter of law, not constitute negligent conduct. Because this is a matter of law, the judge would be authorized to correct this aspect of the jury’s verdict. If the driver is thus found to be not negligent in this matter, his recovery will not be reduced. (A) is incorrect because in most states that have adopted comparative negligence, the plaintiff’s negligence will be considered even in cases where the defendant has acted willfully and wantonly. (C) is incorrect because the fact that the defendant is more than 50% at fault does not mean that the plaintiff is entitled to receive 100% of his damages from the defendant in a partial comparative negligence jurisdiction. It only means that the plaintiff’s recovery is not totally defeated. (D) is incorrect because, although the transportation of chemical waste would probably be considered an abnormally dangerous activity, liability for conducting an abnormally dangerous activity attaches only if the harm results from the kind of danger to be anticipated from such activity; i.e., the injury must flow from the normally dangerous propensity of the activity. The canister falling from the truck is not the “normally dangerous propensity” of transporting chemical waste.

How well did you know this?
1
Not at all
2
3
4
5
Perfectly
55
Q

Claim Preclusion - Res Judicata Elements

A

Claim preclusion (res judicata) requires that (i) a valid, final judgment on the merits was entered in the first case; (ii) the cases were brought by the same claimant against the same defendant; and (iii) the same cause of action is involved in the later lawsuit. Generally, a claimant is required to assert all causes of action arising out of the same transaction or occurrence that is the subject matter of the claim

How well did you know this?
1
Not at all
2
3
4
5
Perfectly
56
Q

When may a person use deadly force in self-defense?

A

A person may use deadly force in self-defense if he: (i) is without fault; (ii) is confronted with unlawful force; and (iii) reasonably believes that he is threatened with imminent death or great bodily harm. Generally, one who is at fault for starting a confrontation has no right to use force in his own defense during that confrontation. However, if the victim of the initial aggression suddenly escalates a relatively minor fight into one involving deadly force and does not give the aggressor a chance to withdraw or retreat, the aggressor may use deadly force in his own defense.

How well did you know this?
1
Not at all
2
3
4
5
Perfectly
57
Q

When may a non-party to an action intervene as a matter of right under Rule 24?

A

Under Rule 24, a nonparty may intervene in an action as a matter of right in two situations. First, a nonparty may intervene when it has an unconditional right to do so by a federal statute. Second, a nonparty may intervene if (i) it has an interest in the property or transaction the is the subject matter of the action; (ii) the disposition or resolution of the action may as a practical matter impair the nonparty’s ability to protect its interest; and (iii) the nonparty’s interest is not adequately protected by an existing party in the action.

How well did you know this?
1
Not at all
2
3
4
5
Perfectly
58
Q

When will an injunction be granted to enforce a restrictive covenant?

A

When the covenant runs with the land. A covenant will be enforceable as an equitable servitude-allowing a covenantee, covenantor, or successor to enforce the covenant in equity by way of injunction-when there is (i) a covenant in a writing satisfying the Statute of Frauds, that (ii) touches and concerns the land (i.e., the effect of the covenant makes the land more useful or valuable to the benefited party) and that (iii) indicates an intention that the servitude exists, and (iv) notice is given to future owners of the burdened land.

How well did you know this?
1
Not at all
2
3
4
5
Perfectly
59
Q

The United States was involved in a dispute with a small island nation over the ownership of an archipelago. On discovering that the archipelago was rich in oil, the President announced that he would appoint an ambassador to negotiate a treaty with the island nation to jointly exploit the oil reserve. A majority of Senators believed that the island clearly belonged to the United States and did not want to negotiate with the island nation. They passed a resolution requiring the President to include a Senator in his diplomatic mission to ensure that the Senate’s view was presented in any negotiation with the island nation. What is the strongest constitutional ground for the President’s refusal to do so? A As commander in chief, the President has the exclusive power to determine how to protect our national interest abroad. B The resolution is unreasonable because it includes a Senator and not any Representatives. C The President has the exclusive power to select diplomatic representatives of the United States. D The Senate, if it does not like the President’s actions, can refuse to appropriate the necessary monies for the President to implement his policies.

A

The President’s strongest argument is that the power to select ambassadors is vested by the Constitution in the President, and the Senate’s only power in this respect is to advise and give (or withhold) its consent. The Senate is not given the power to force ambassadors on the President. (A) is not a strong argument because the President’s power as commander in chief is not involved here. That power involves the President’s role as the supreme military leader, and military issues are not involved under the facts. (B) is not a strong argument because as far as foreign relations are concerned, the Senate does have more powers than the House. As stated above, ambassadors may be selected only with the advice and consent of the Senate, and the President’s treaty power is also similarly limited. Thus, but for the fact that the resolution is not within the Senate’s power to enforce, it would be appropriate to exclude the House from participating, because foreign affairs are involved. (D) may be a true statement, because Congress controls appropriations, but it is not a strong argument because it merely states that the Senate has another remedy (i.e., besides forcing an ambassador on the President), and the fact that the Senate has another method for achieving its goals has no bearing on whether its action here is permissible.

How well did you know this?
1
Not at all
2
3
4
5
Perfectly
60
Q

A defendant was convicted in federal court of possession of one kilogram of heroin with intent to distribute. She was sentenced to a prison term. Subsequently, the defendant was indicted by a federal grand jury for conspiracy to distribute the same kilogram of heroin. She moved to dismiss the indictment. Should her motion be granted? A No, because the Double Jeopardy Clause does not apply when the second prosecution is for violation of a separate statute. B No, because each prosecution requires proof of an element that the other does not. C Yes, because the Double Jeopardy Clause protects her against a second prosecution for the same criminal conduct. D Yes, because the Due Process Clause protects her against double punishment for the same criminal conduct.

A

The defendant’s motion should be denied because a prosecution for conspiracy is distinct from a prosecution for any substantive offense involving the same conduct as the conspiracy. The Fifth Amendment provides that no person shall be twice put in jeopardy for the same offense. The general rule is that two crimes do not constitute the same offense if each crime requires proof of an additional element that the other crime does not require, even though some of the same facts may be necessary to prove both crimes. [Blockburger v. United States (1932)] Furthermore, a prosecution for conspiracy is not barred merely because some of the alleged overt acts of that conspiracy have already been prosecuted. [United States v. Felix (1992)] Here, both the conspiracy charge and the possession charge require proof of an element that the other charge does not; hence, there is no double jeopardy problem with the indictment. (A) is incorrect because it is too broad a statement. The fact that separate statutes are involved does not establish that these are not the “same offense” for purposes of double jeopardy. (C) is incorrect because the “same conduct” test is not currently used by the Supreme Court to evaluate a double jeopardy claim. (D) is incorrect because the question involves the defendant’s motion to quash an indictment and not her ultimate punishment.

How well did you know this?
1
Not at all
2
3
4
5
Perfectly
61
Q

A driver and his passenger were involved in an automobile accident when the driver ran a red light and crashed into another car. Due to a manufacturing defect in the automobile’s airbag system, the passenger side airbag did not deploy. The passenger was killed on impact. The passenger’s estate brought suit against the driver and the airbag’s manufacturer. At trial it is established that the driver was negligent in running the red light. What effect would such proof have on the claim of the passenger’s estate against the airbag manufacturer? A It would reduce recovery by the estate if the action against the manufacturer is based on negligence. B It would bar recovery by the estate if the trier of fact finds that the driver was the sole legal cause of the passenger’s death. C It would bar recovery by the estate if it is shown that the driver is the sole legal heir of the passenger’s estate. D It would have no effect on recovery by the estate as long as the action against the manufacturer is based on strict liability.

A

The driver’s negligence would bar recovery if it was the sole legal cause of the passenger’s death. Regardless of the theory that the plaintiff is using in a products liability action, actual and proximate cause must be established. If the driver’s negligence is the sole legal or proximate cause of the passenger’s death, it would preclude the estate’s suit against the airbag manufacturer because the defect was not a legal cause of the passenger’s death. (A) is incorrect because the driver’s contributory negligence will not be imputed to the passenger; hence, it will not reduce the estate’s recovery under comparative negligence rules. (C) is incorrect. A potential beneficiary who was negligent will be subject to the jurisdiction’s fault rules. Hence, his negligence will reduce his recovery under pure comparative negligence rules but will not bar it, even if he is the sole heir. (D) is incorrect. In most pure comparative negligence jurisdictions, the same comparative fault rules will apply whether the action against the manufacturer is based on negligence or strict liability. As discussed above, whether the estate can recover, and the extent of its recovery, depends on causation issues and the driver’s status as a beneficiary of the estate.

How well did you know this?
1
Not at all
2
3
4
5
Perfectly
62
Q

What is conversion? What is the remedy?

A

Conversion is the intentional interference with the plaintiff’s right of possession in the chattel that is serious enough to warrant that the defendant pay the full value of the chattel. Conversion will be found if the defendant was using the chattel without permission and it was accidentally damaged, as in this case. The remedy for conversion is the fair market value of the chattel at the time and place of conversion.

63
Q

Parole Evidence Rule - UCC - May you provide Extrinsic Evidence of Additional Consistent Oral Terms that are customary within the business?

A

Under UCC section 2-202, a party cannot offer consistent additional terms if the writing was intended as a complete statement of the terms of the agreement. (In contrast, evidence of custom in the trade can be offered regardless of the completeness of the written agreement.) One of the general rules of contract construction, including contracts for goods under the UCC, is that courts will look to see what custom and usage are in the particular business and in the particular locale where the contract is either made or to be performed.

64
Q

In an attempt to induce her niece to improve her grades, an aunt told her niece that she would give her $50 for every “A” she earned in law school. Inspired by the opportunity to earn extra spending money, the niece studied like never before and ended up earning three “A’s” the following semester. Upon contacting her aunt with the news, the niece learned that her aunt was “only kidding” about paying her for every “A” earned in law school. If the aunt’s offer to her niece was held to be valid, it would be an offer for what type of contract? A A bilateral contract. B A unilateral contract. C A bilateral contract or a unilateral contract, according to the offeree’s intentions. D A unilateral contract that became a bilateral contract when the niece began to perform.

A

The aunt’s offer to her niece was for a unilateral contract. In a unilateral contract, acceptance of an offer is possible only by performing a stipulated act, whereas in a bilateral contract, acceptance of an offer is accomplished by promising to do the stipulated act. The aunt’s offer could be accepted by the niece only by completion of performance (i.e., obtaining an “A” in the class), and when the niece received her three “A’s,” a unilateral contract was formed. Thus, (B) is correct and (A) and (C) are incorrect. (D) is incorrect because acceptance of an offer by performance of the stipulated act creates a unilateral contract, and such performance does not convert the contract into a bilateral contract, which is created by a promise to perform rather than by performance itself.

65
Q

A child was severely injured at an amusement park when she was ejected from a ride that went slightly off its track. The ride malfunctioned as a result of a manufacturer’s defect, but had the child been properly secured in the ride’s seatbelt by one of the ride operators, she would not have been injured. The child was unable to identify which ride operator improperly buckled her in. In the child’s suit against the amusement park, who will win? A The child will win, because a ride operator failed to use reasonable care in securing the seatbelt. B The child will win, because operators of the ride did not discover the ride’s defect through a reasonable inspection. C The amusement park will win, because the child cannot specifically identify which employee was negligent. D The amusement park will win, because the child’s injury was caused by a defect that was present in the ride at the time of its purchase from the manufacturer.

A

Because the ride operator was negligent in improperly securing the child, the amusement park is vicariously liable under the doctrine of respondeat superior. This doctrine imposes liability on an employer for the tortious conduct of its employee occurring within the scope of the employment relationship. Here, securing the children in the ride was one of the ride operator’s tasks. Since this task was performed negligently and this negligence was one of the causes of the child’s injuries, the amusement park will be liable. (B) is incorrect because the failure to make a reasonable inspection of the ride is not the negligent conduct suggested by the facts; rather, it was the failure to belt the child in securely that was negligent. Nothing in the facts indicates that a reasonable inspection would have disclosed the defect. (C) is incorrect because the amusement park will be liable even if the child cannot identify which specific employee was negligent. Through the doctrine of res ipsa loquitur, the child can establish breach of duty just by the fact that she was ejected from the ride after she had been strapped in; in other words, an inference of negligence is established because the accident causing her injury is the type that would not normally occur unless someone was negligent. The other two elements required for res ipsa loquitur are that the plaintiff was free of fault, which is easy to show in this case, and that the negligence was attributable to the defendant; i.e., that this type of accident ordinarily happens because of the negligence of someone in the defendant’s position. If the plaintiff were suing a ride operator individually, this requirement would prevent her from using res ipsa loquitur in most jurisdictions because she could not establish that that particular operator was negligent. However, the amusement park will be liable under the doctrine of respondeat superior regardless of which of its employees was negligent, because the amusement park, through its employees, did have exclusive control over the ride; therefore, the negligence is attributable to the amusement park. (D) is incorrect because the amusement park will lose even though the injury to the child was caused by a defect present at the time the ride was purchased. The negligence of one of the park’s employees was also a cause of the child’s injuries.

66
Q

A local entertainment section of a newspaper published a story on the town’s business district, accompanied by photos of various businesses in the district. A minister who happened to be walking on the sidewalk in front of an adult bookstore when a photo was taken for the story became very upset when he saw it in the newspaper, because the camera angle made it appear that he was exiting the bookstore. If the minister sues the newspaper for invasion of privacy and establishes the above facts, is he likely to prevail? A Yes, because the photo made it appear as if he was exiting an adult bookstore. B Yes, because the newspaper made a public disclosure of a private fact. C No, because he was on a public sidewalk when the photo was taken. D No, because he has not alleged any economic or pecuniary damages.

A

The minister likely will prevail because unauthorized use of his picture that falsely makes him appear to be exiting the adult bookstore would be highly offensive to a reasonable person under the circumstances and constitute a false light invasion of privacy. To establish a prima facie case for invasion of privacy based on publication by defendant of facts placing plaintiff in a false light, the following elements must be proved: (i) publication of facts about plaintiff by defendant placing plaintiff in a false light in the public eye; and (ii) the “false light” must be something that would be highly offensive to a reasonable person under the circumstances. Here, the photo created the false impression that the minister was exiting an adult bookstore. Publication of the photo conveying this false impression of the minister’s conduct would be highly offensive to a reasonable person under the circumstances. (B) cannot be correct because it is couched in terms of disclosure of private facts about the plaintiff. To establish a prima facie case for invasion of privacy based on public disclosure of private facts about plaintiff, the following elements must be proved: (i) publication or public disclosure by defendant of private information about the plaintiff; and (ii) the matter made public is such that its disclosure would be highly offensive to a reasonable person of ordinary sensibilities. Here, no private facts were disclosed, and therefore an action based on public disclosure of private facts will not succeed. (C) is incorrect. Because the minister was placed in a false light, it makes no difference that the picture was taken on a public sidewalk. An invasion of privacy based on false light can occur on public property as well as private property. (D) is incorrect. The absence of economic harm to the minister has no bearing on a privacy action. In an action for invasion of right to privacy, the plaintiff need not plead and prove special damages, providing the elements of a prima facie case are present. In other words, emotional distress and mental anguish are sufficient damages.

67
Q

As a result of a personal injury lawsuit, a victim obtained a judgment against a tortfeasor for $100,000. The tortfeasor, who had few assets, did not pay the judgment. On April 1 of the following year, the tortfeasor inherited a parcel of land from her uncle. On May 1, the tortfeasor entered into a contract with a buyer to sell the land for $120,000. The contract was not recorded. The buyer immediately applied to a bank for a loan of $100,000. The bank approved the buyer’s loan, and on May 15, a closing was held. The tortfeasor deeded the land to the buyer, and the buyer executed a mortgage for $100,000 to the bank. Due to an error by the title company, the deed from the tortfeasor to the buyer was not recorded, although the mortgage to the bank was recorded. Neither the buyer nor the bank had any knowledge of the victim’s judgment. On May 20, the victim recorded his judgment in the county recorder’s office where the land was located. At that time, he had no knowledge of the buyer’s or the bank’s rights. When he learned about them, he immediately brought a proceeding to foreclose his judgment lien, naming the tortfeasor, the buyer, and the bank as parties. The jurisdiction has a typical grantor/grantee recording index, and has enacted the following statute: “Any judgment properly filed in the county recorder’s office shall, for 10 years from filing, be a lien on the real property then owned or subsequently acquired by any person against whom the judgment is rendered. No conveyance or mortgage of real property shall be good against subsequent bona fide purchasers for value and without notice unless the same be recorded according to law.” As between the victim and the bank, which party’s interest in the land will be given priority? A The bank, because the bank recorded its mortgage before the victim recorded his judgment lien. B The bank, because the victim is not protected by the recording statute. C The victim, because the victim’s judgment was filed in the recorder’s office before the buyer’s deed was recorded. D The victim, because the judgment lien extends to after-acquired property.

A

The victim will not likely prevail against the bank because a majority of courts hold that the judgment lienor is not protected by the recording statute. If the statute here, which is a notice statute, were applicable to protect the victim, he would have priority over the bank because his judgment lien was recorded before the buyer’s deed was recorded. Under this view, the bank’s mortgage would have been considered “wild” and would be deemed unrecorded because the preceding conveyance, the buyer’s deed, was actually unrecorded. A searcher in the public records would therefore have been unable to find the mortgage. Hence, if the statute were applicable to protect the victim, he would have priority over the bank. However, most courts reason that either (i) a judgment creditor is not a bona fide purchaser because he did not pay contemporaneous value for the judgment, or (ii) the judgment attaches only to property “owned” by the debtor, and not to property previously conveyed away, even if that conveyance was not recorded. Under the statute in the present question, a judgment does not attach until it is recorded. Here, the victim’s judgment did not attach to the land until after the bank obtained a mortgage on it, and the recording statute does not change that result. The failure of the buyer to record, and the resultant treatment of the bank as unrecorded, is irrelevant. Thus, the bank’s mortgage is superior to the victim’s lien. (A) is wrong because it does not matter whether the bank’s mortgage was recorded, as against a subsequent judgment lien creditor. The judgment lien creditor is not protected by the recording statute, so the bank prevails even though its mortgage would be deemed unrecorded, as discussed above. (C) is wrong because, as discussed above, a majority of courts hold that the judgment lienor is not protected by the recording statute. (D) is wrong because the land was not after-acquired property, because the judgment lien was not filed until the tortfeasor had obtained-and conveyed away-an interest in the property. However, if the victim had in fact recorded his lien before the tortfeasor inherited the land, the after-acquired property provision of the statute would have applied, the victim would have had a recorded lien on the land as soon as the tortfeasor acquired it, and the victim would have gained priority over the bank.

68
Q

A defendant was involved in an accident in which her car struck the rear end of the car driven by the plaintiff. The police issued tickets to the defendant, charging her with reckless driving and speeding. When the defendant’s case came before the traffic court, her attorney entered into a plea bargain with the prosecutor. Under the plea bargain, the defendant agreed to plead guilty to speeding and to pay a fine of $100, and the prosecution agreed to drop the reckless driving charge. Accordingly, the defendant pleaded guilty and the court fined her $100. In the later civil suit, where the plaintiff is seeking damages from the defendant for personal injuries, is the guilty plea before the traffic court admissible? A. Yes, because it is a statement by an opposing party. B Yes, because it is a statement against interest. C No, because there is a public policy in favor of plea bargaining to promote court efficiency. D No, because no felony was involved.

A

The defendant’s guilty plea is a statement by an opposing party and thus is admissible. Under the Federal Rules, a statement by an opposing party (commonly called an admission) is not hearsay. [Fed. R. Evid. 801(d)(2)] A plea of guilty to a traffic infraction is a formal judicial statement. The statement is conclusive in a prosecution for that infraction, but if the plea is used in another proceeding, it is merely an evidentiary statement (i.e., it is not conclusive and can be explained). Here, the defendant has acknowledged by her guilty plea that she was speeding at the time of the accident. This fact is relevant to the plaintiff’s suit for personal injuries because it increases the likelihood that the defendant was at fault in the accident that caused those injuries. Therefore, the defendant’s guilty plea is admissible in the current civil action as an evidentiary statement. (B) is wrong because there is no indication that the defendant is unavailable. Statements of a person, now unavailable as a witness, against that person’s pecuniary, proprietary, or penal interest when made are admissible under the statement against interest exception to the hearsay rule. [Fed. R. Evid. 804(b)(3)] A declarant is unavailable if: (i) she is exempt from testifying because the court rules that a privilege applies, (ii) she refuses to testify despite a court order to do so, (iii) she testifies to not remembering the subject matter, (iv) she is dead or ill and unable to testify, or (v) she is absent and the statement’s proponent has been unable to procure her attendance or testimony by process or other reasonable means. [Fed. R. Evid. 804(a)(1) - (5)] Since the defendant apparently is available as a witness in the suit, the statement against interest exception is inapplicable. Although it may be true that public policy favors plea bargaining, (C) is wrong because there is no attempt here to offer a statement made during the plea bargaining process. Under the Federal Rules, withdrawn guilty pleas, pleas of nolo contendere, offers to plead guilty, and evidence of statements made in negotiating such pleas are not admissible in any proceeding. [Fed. R. Evid. 410] However, there is no prohibition against admitting the guilty plea itself. This question asks whether the guilty plea is admissible, not whether statements made in negotiation thereof are admissible. Thus, the policy favoring plea bargains is irrelevant. (D) is wrong because the question asks about the admissibility of the plea rather than a copy of the conviction. Convictions may be introduced to prove any fact essential to the case only if they are felony convictions. [Fed. R. Evid. 803(22)] Because the conviction is not being offered, the fact that a felony is not involved is of no consequence. The defendant’s plea is admissible as an opposing party’s statement even though a felony was not involved.

69
Q

A comprehensive federal health-care reform statute created a Federal Health Policy Board, which was directed to monitor the fees charged for various medical procedures covered by insurance. The board also had the power to subpoena records to determine whether fee increases were a true reflection of cost increases. Nothing in the statute provided for caps on fee increases. Because of the continuing escalation of health-care costs while the statute was being debated, several states had passed health-care legislation on their own. One state passed legislation that prohibited most fee increases of 10% or more per year for specified health-care services covered by insurance, and created a health-care review board to regulate these costs and impose monetary penalties on health-care providers or insurers that tried to circumvent the cap. Which of the following would be the best basis for finding the state provision unconstitutional? A The federal legislation was passed after the state legislation and therefore supersedes it. B The Federal Health Policy Board was constituted with many of the same powers as the state board but was not given the power to impose sanctions. C The state provision impairs existing contracts between health-care providers and insurers in violation of the Contract Clause. D Health-care fee caps create an undue burden on interstate commerce even in the absence of federal regulation.

A

The fact that the federal board was similar to the state board but was not given the power to restrict fee increases and impose sanctions in an otherwise comprehensive bill suggests that such provisions in the state law violate the Supremacy Clause. A state law may fail under the Supremacy Clause even if it does not directly conflict with a federal statute or regulation if it interferes with the achievement of a federal objective or the federal regulations occupy the entire field. Where the federal laws are comprehensive or a federal agency is created to oversee the field, preemption will often be found. The fact that the health-care legislation was comprehensive but the federal board was not given regulatory or enforcement power suggests that Congress did not want specific restrictions in these areas and may have wanted free-market principles to determine fee increases at the outset. The state board’s power to impose these restrictions may violate the Supremacy Clause under these circumstances. (A) is incorrect because the fact that the federal legislation was passed later does not automatically mean that the state legislation has been superseded. In areas of concurrent legislative power, a state regulation will be upheld if it does not conflict with and is not preempted by federal legislation. (C) is incorrect because the Contract Clause prevents only substantial impairments of existing contracts by state legislation, and only if the legislation does not serve an important and legitimate public interest or is not a reasonable and narrowly tailored means of promoting that interest. Here, the law has a prospective effect only, and even if existing contracts between health-care providers and insurers are affected by the legislation, the other requirements for the Contract Clause to apply are not likely to be satisfied. (D) is incorrect because states may regulate local aspects of interstate commerce in the absence of federal regulation as long as the regulation is nondiscriminatory and does not unduly burden interstate commerce, which is a case-by-case balancing test. Here, the legislation appears to be nondiscriminatory and there are insufficient facts to establish that it would constitute an undue burden; hence, (B) presents a stronger argument than (D).

70
Q

A jogger not paying attention to where he was going collided with an elderly woman in a wheelchair, injuring her. The woman, who was mentally incapacitated, was being pushed down the sidewalk by her daughter, who was also her legal guardian. After the accident, the daughter did not take the woman to the doctor for treatment because the daughter had an unreasonable fear of catching a virus at the doctor’s office. As a result, the woman’s wounds became infected and took longer to heal. What argument provides the woman with her best chance to recover against the jogger for all of her injuries? A The doctrine of mitigation of damages does not bar recovery for the original injuries caused by the jogger. B Any unreasonable conduct on her daughter’s part is not to be imputed to the woman. C Neither the woman nor her daughter were contributorily negligent in causing the collision. D Defendants must take their victims as they find them, including the unreasonable behavior of their legal guardians.

A

The woman’s best argument that her daughter’s refusal to take her to a physician, if deemed to be negligent, is not imputed to her. A plaintiff has a duty to take reasonable steps to mitigate damages. Thus, in personal injury cases, there is a duty to seek appropriate treatment to effect healing and to prevent aggravation. Failure to do so will preclude recovery for any particular item of injury that occurs or is aggravated due to the failure to mitigate. Thus, the woman’s not consulting a doctor could limit her recovery to the damages for the original injury only. However, she is incapacitated and in the care and control of a legal guardian (her daughter), who decided not to bring her to a doctor. In actions against a third party, a legal guardian’s negligence is not imputed to the person under her care. Thus, using the argument in choice (B) that any negligence on the part of the woman’s daughter will not be imputed to her, she should receive a full recovery for all of her injuries if she prevails in an action against the jogger. (A) is incorrect even though it is a true statement. The doctrine of mitigation of damages requires a plaintiff to take reasonable steps to mitigate damages, which in personal injury cases means seeking appropriate treatment to effect a cure or healing and to prevent aggravation. Hence, it does not bar recovery for the plaintiff’s original injuries. However, choice (B) gives her a chance to recover for all of her injuries, and choice (A) does not. (C) is incorrect because a plaintiff’s contributory negligence as to the accident itself is relevant only as to the amount of damages recoverable for the original injury; it does not affect the rule that a plaintiff must take all reasonable measures to mitigate damages after the original injury is inflicted. (D) is incorrect because it misstates the concept of “taking your victim as you find her.” This concept refers to the physical or mental condition of the victim at the time of the injury (e.g., the “eggshell skull plaintiff”); it does not cover the victim’s relationship to others and their attitudes or actions. Thus, the attitude of a victim’s legal guardian toward medical treatment is not included in “taking your victim as you find her.”

71
Q

A company that was the leading supplier of home water filtration systems had a network of sales promoters who were under contract for two- or three-year terms and were compensated solely by commissions earned from sales and by occasional bonuses. Veteran promoters also earned commissions by recruiting other promoters for the company. One of the company’s veteran promoters was contacted by a former top sales representative for another manufacturer who was looking for similar sales opportunities in the region. The sales rep knew that the promoter might be able to get her a position with his company, which was looking for additional promoters. At the time he met with the sales rep, the promoter’s contract with the company had one more month to run. When the promoter’s contract with the company expired, he announced that he was forming his own business to market a different line of water filtration systems manufactured by a competitor of the company, and that the sales rep would be in charge of his promotional network. The company brought an action against the promoter for interference with business relations for hiring the sales rep. At a preliminary hearing, the parties stipulated to the above facts and that the promoter was an independent contractor rather than an employee of the company. The promoter then filed a motion for a summary judgment in his favor. Should the court grant the promoter’s motion? A Yes, because the sales rep had no business relationship with the company at the time the promoter’s alleged interference occurred. B Yes, because the promoter was an independent contractor rather than an employee of the company. C No, because the jury could find that the means the promoter used to obtain the sales rep were not privileged. D No, because the jury could find that the promoter breached his contract with the company by meeting with the sales rep.

A

The court should not grant the promoter’s motion because the jury could find that the promoter used improper means, while working for the company, to divert the sales rep for his own purposes. To establish a prima facie case for interference with business relations, the following elements must be proved: (i) existence of a valid contractual relationship between plaintiff and a third party or a valid business expectancy of plaintiff; (ii) defendant’s knowledge of the relationship or expectancy; (iii) intentional interference by defendant that induces a breach or termination of the relationship or expectancy; and (iv) damage to plaintiff. Thus, a plaintiff has a cause of action for interference with probable future business relationships for which the plaintiff has a reasonable expectation of financial benefit. On the other hand, an interferer’s conduct may be privileged where it is a proper attempt to obtain business for the interferer, particularly if the interference is only with a prospective business relationship rather than with an existing contract. What is proper depends on various factors, including the means of persuasion used. Here, the promoter’s conduct would not be privileged if the jury were to find that he improperly used his position with the company to develop a relationship with the sales rep. (A) is incorrect because even though the company did not have an existing contractual relationship with the sales rep, it could very well show that it had a reasonable expectation of signing a contract with the sales rep that the promoter knew of and intentionally interfered with. (Note that courts do not permit recovery for negligent interference with business relations.) Whether the company could prove its expectancy to a sufficient degree to establish actual damages would be a question for the trier of fact; hence, summary judgment would not be appropriate on this basis. (B) is incorrect because the promoter can be liable for interference with business relations regardless of whether he was an independent contractor or an employee of the company, as long as he used improper means for steering the sales rep away from the company. (D) is incorrect because a defendant’s breach of his own contract with the plaintiff is not a basis for the tort of interference with business relations. If the promoter breached his contract with the company, the company’s cause of action would be in contract and its remedy would be governed by contract rules. Here, the tort action that the company is suing on does not require establishing a breach of the promoter’s contract with the company.

72
Q

A gang member threatened to kill the defendant unless he robbed a convenience store and gave the proceeds to the gang member. The gang member also demanded at gunpoint that the defendant kill the clerk to prevent identification. In abject fear of his life, the defendant did everything that the gang member requested. If the defendant is arrested and charged with murder and robbery in a common law jurisdiction, what result? A The defendant should be convicted of murder and robbery. B The defendant should be acquitted of the robbery and convicted of murder. C The defendant should be convicted of robbery, and the killing will be reduced to voluntary manslaughter. D The defendant should be acquitted of the robbery, and the killing should be reduced to voluntary manslaughter.

A

The defendant should be convicted of common law murder, but acquitted of the robbery. At common law, murder is the unlawful killing of a human being with malice aforethought. “Malice aforethought” exists if the defendant has any of the following states of mind: (i) the intent to kill (express malice); (ii) the intent to inflict great bodily injury; (iii) a reckless indifference to an unjustifiably high risk to human life (“abandoned and malignant heart”); or (iv) the intent to commit a felony. In the instant case, malice could be found either by the intent to kill (because the clerk was intentionally killed to prevent identification) or by the intent to commit a felony (the killing was committed during the course of a robbery). Robbery is an aggravated form of larceny and consists of the following elements: (i) a taking; (ii) of the personal property of another; (iii) from the other’s person or presence; (iv) by force or intimidation; (v) with the intent to permanently deprive him of it. Clearly, the elements for robbery are met here. Thus, at first glance, the defendant has committed both murder and robbery. However, the fact pattern also raises the defense of duress. A person is not guilty of an offense, other than intentional homicide, if he performs an otherwise criminal act under the reasonable belief that another will imminently inflict death or great bodily harm on him or an immediate family member if he does not commit the criminal act. In the instant case, the defendant committed the robbery under duress and thus should be acquitted of that charge, making (A) incorrect. However, duress would not be effective against a murder charge based on an intent-to-kill theory, and here the defendant intentionally killed the store clerk under instructions from the gang member to prevent identification. Thus, because the defendant could be convicted of an intent-to-kill murder, but acquitted of the robbery charge based on duress, (B) is the correct answer. (C) and (D) are incorrect. An argument could be raised that the killing should be reduced to voluntary manslaughter from murder, given that the defendant was acting under the provocation of a threat of deadly force. At common law, provocation would reduce a killing to voluntary manslaughter if (i) the provocation must have been one that would arouse the sudden and intense passion in the mind of an ordinary person such as to cause him to lose self-control; (ii) the defendant must have in fact been provoked; (iii) there must not have been sufficient time to cool off; and (iv) the defendant did not in fact cool off. Provocation includes being subjected to a serious battery or a threat of deadly force. That said, the reduction to voluntary manslaughter occurs only as to the person who provoked the defendant (or the killing of a third person under the transferred intent doctrine). Thus, had the defendant killed the gang member, he might have been able to claim “adequate provocation” to have the killing reduced to voluntary manslaughter (assuming that a straight self-defense issue could not have been raised). However, as discussed above in terms of a duress defense, it would not justify the killing of a Third Party

73
Q

A homeowner hired a contractor to make some improvements on his house. They entered into a written contract providing that the contractor would do the improvements for $5,000. Shortly after the contract was signed, the contractor told the homeowner to give the money to his (the contractor’s) daughter when the job was finished, adding, “She is getting married soon and I want her to have a nice wedding present from me.” The daughter was aware that her father made this statement to the homeowner. She married, but soon thereafter the contractor told the homeowner to pay him the $5,000, and not the daughter, because his son-in-law had a gambling problem and would probably use the money to bet at the racetrack. What is the best argument in favor of the daughter’s being able to enforce a contract for $5,000 in her favor? A Statute of Frauds. B Parol evidence rule. C The daughter was an intended third-party beneficiary. D The daughter married in reliance on the promise. Highlight

A

The daughter’s best argument to enforce the contract in her favor is that she married in reliance on the contract (detrimental reliance), although she will probably be unsuccessful. Here, the daughter was a gratuitous assignee (because she gave no consideration) and her rights under the contract were revoked. Thus, her strongest argument will be one that nullifies the revocation. Under the doctrine of detrimental reliance, a promise will be enforced to the extent necessary to prevent injustice if it was made with a reasonable expectation that it would induce reliance, and such reliance was in fact induced. The problem with this argument here is that it is not clear that the daughter relied on the promise to give her $5,000, because she already had planned to get married. However, none of the other choices is a possible argument, so (D) is her best choice. (A) would not help her because the Statute of Frauds is a defense to enforcement of certain contracts when there is no writing, including contracts where the consideration is marriage. Here the daughter is seeking to enforce the contract, not prevent its enforcement. (B) is incorrect because the parol evidence rule only prevents the introduction of prior or contemporaneous oral statements to contradict the terms of an integrated written contract. Here, the statement that gave the daughter her rights and the one that took them away were both subsequent oral statements. (C) does not help her because she was not an intended third-party beneficiary. If a contract between two parties contemplates performance to a third party, that third party may have rights to enforce the contract. To do so, the third party must be an intended beneficiary at the time the contract was made (e.g., designated in the contract). An assignment, on the other hand, is a contract that does not contemplate performance to a third party when the contract is made. Rather, later one of the parties transfers his rights to another. Here, the contractor and the homeowner signed their contract and later the contractor assigned his rights to his daughter. Thus, the daughter was not an intended third-party beneficiary who could enforce the agreement, but merely an assignee who gave no consideration for the assignment. As such, the contractor was free to revoke the assignment, and his daughter cannot recover the $5,000.

74
Q

A homeowner returned home from work one day to find a robber in her living room. After a brief physical altercation, the homeowner ran to a bedroom, hid in a closet, and called 911 on her cell phone. Police officers arrived in less than two minutes and were able to apprehend the robber as he tried to run out the front door. Once they made sure he was locked in the police car, one of the officers went to speak with the homeowner about what had happened. She was still crying and shaking when the officer found her, and she said, “Thank you for catching him! He punched me in the head as I was running away!” The robber was charged with robbery and assault. Traumatized, the homeowner left the country and cannot be traced, despite the efforts of the prosecutor. The prosecutor intends to call the officer to testify as to the homeowner’s statement. Should the court allow the officer’s testimony? Press Enter or Space to submit the answer A. Yes because the homeowner’s statement is an excited utterance. B Yes, because the homeowner’s statement is not testimonial. C No, because admitting the homeowner’s statement would violate the defendant’s constitutional rights. D No, because the homeowner’s statement is hearsay not within any exception.

A

Confrontation Clause Issue The testimony should not be admitted. Under the Confrontation Clause, an accused has the right to be confronted by the witnesses against him. A hearsay statement will not be admitted-even if it falls within a hearsay exception-when: (i) the statement is offered against the accused in a criminal case; (ii) the declarant is unavailable; (iii) the statement was testimonial in nature; and (iv) the accused had no opportunity to cross-examine the declarant’s “testimonial” statement prior to trial. The Supreme Court has established that if the primary purpose of police interrogation is to enable the police to help in an ongoing emergency, statements made in the course of the interrogation are nontestimonial. When the primary purpose of the interrogation is to establish or prove past events potentially relevant to a later criminal prosecution, statements are testimonial. Here, the homeowner’s statement was hearsay because it was made out of court and is being offered for its truth-that the robber punched her in the head. It appears that the emergency had already resolved by the time the statement was made. Although only a few minutes had passed since the physical altercation and the homeowner was still upset, the robber no longer posed any danger because he was locked in the police car and the homeowner was aware of this (“Thank you for catching him!”). Therefore, the homeowner’s statements to the officer were testimonial. Because the homeowner is unavailable to testify at trial and the robber has had no opportunity to cross-examine the statements, admitting them at trial through the testimony of the officer would violate the Confrontation Clause. (A) is incorrect. Under the excited utterance exception, a declaration made by a declarant during or soon after a startling event is admissible if it relates to the startling occurrence and was made under the stress of excitement produced by the startling event. Here, finding a robber in one’s home is a startling event, and the homeowner was still under the stress of the excitement when she made the statement-she was crying and shaking. The statement likely qualifies as an excited utterance. However, even a hearsay statement that falls within an exception may be barred by the Confrontation Clause, as is the case here. (B) is incorrect. As stated above, when the primary purpose of the interrogation is to establish or prove past events potentially relevant to a later criminal prosecution, statements are testimonial. Because the homeowner’s statement was made for this purpose and not to address an ongoing emergency, it was testimonial in nature and its admission would violate by the Confrontation Clause. (D) is incorrect. As stated above, the homeowner’s statement falls within the exception for excited utterances, so it will not be excluded because of the hearsay rule. The testimony will be excluded because of the robber’s constitutional right to be confronted by the witnesses against him

75
Q

A plaintiff sued a defendant and his employer for personal injuries. The plaintiff claimed that she was struck on the head by a wrench dropped by the defendant from a high scaffold, on which the defendant was working in the course of a construction project. To prove that it was the defendant who dropped the wrench, the plaintiff offers the wrench itself as evidence: The wrench bears the brand name “Craftsman” on the handle, and other evidence shows that the wrenches used by the defendant on the job are “Craftsman” brand wrenches. Is the wrench admissible? A No, because but for the word “Craftsman” the wrench would be irrelevant, and the word “Craftsman” is inadmissible hearsay. B No, because the wrench is irrelevant as it fails to show that it is more likely than not that the defendant was the person who dropped it. C Yes, because the wrench is relevant direct evidence that it was the defendant who dropped the wrench and is not hearsay. D Yes, because the wrench is relevant circumstantial evidence that it was the defendant who dropped the wrench and is not hearsay.

A

The word “Craftsman” is not hearsay, and the wrench is relevant circumstantial evidence on the issue of whether the defendant dropped the wrench that struck the plaintiff. Evidence is relevant if it tends to make the existence of any fact of consequence to the action more probable than it would be without the evidence. [Fed. R. Evid. 401] If the defendant uses “Craftsman” wrenches on the job, and the wrench that struck the plaintiff bears the brand name “Craftsman,” it is more probable than would otherwise be the case that the wrench that struck the plaintiff was dropped by the defendant. Thus, the wrench is relevant to prove that the defendant dropped the wrench. The wrench is circumstantial, rather than direct, evidence because a fact about it is being proved as a basis for an inference that another fact is true; i.e., the fact that the wrench bears the name “Craftsman” is proved to form a basis for inferring that the defendant dropped the wrench. Direct evidence is offered to prove a fact about the object as an end in itself. Here, the wrench bearing the name “Craftsman” is not being offered as a means of proving, e.g., that the wrench is in fact a “Craftsman” brand. Consequently, the wrench constitutes circumstantial evidence. In addition, when offered for the stated purpose, the wrench is not hearsay. Hearsay is a statement, other than one made by the declarant while testifying at the trial or hearing, offered in evidence to prove the truth of the matter asserted. [Fed. R. Evid. 801(c)] The wrench is not being offered to prove the truth of the matter asserted (i.e., that the wrench is actually a “Craftsman”). It is of no significance whether the wrench being offered is a genuine “Craftsman.” What is important is that it bears the same name as those wrenches used by the defendant on the job. Thus, introduction of the wrench into evidence will not violate the rule against hearsay. (A) is incorrect because it states that the word “Craftsman” on the wrench creates a hearsay problem. As noted, the wrench is not being offered to prove its genuineness as a “Craftsman,” but rather to form the basis for an inference that it was dropped by the defendant. Thus, there is no hearsay problem. (B) reaches the incorrect conclusion that the wrench is not relevant. As has been explained above, the fact that the wrench that struck the plaintiff bears the name “Craftsman” tends to make more probable the material fact that the defendant is the person who dropped the wrench. Therefore, the wrench is relevant. Also, (B) states an incorrect test for relevance. To be relevant, the wrench need not show that it is more likely than not that the defendant dropped it; rather, the wrench must have some tendency to make it more probable than it would be without this evidence that the defendant dropped it. (C) is incorrect because the wrench is circumstantial, rather than direct, evidence that the defendant dropped the wrench that struck the plaintiff.

76
Q

Congress created a seven-member safety commission to investigate and make recommendations to Congress for new fireworks safety laws, to make further rules for establishing safety and performance standards, and to prosecute violations of these safety standards. The chairman of the commission was appointed by the President. Three members were selected by the Speaker of the House of Representatives, and three members were selected by the President pro tempore of the Senate. An organization with proper standing seeks to enjoin enforcement of the commission’s rules. Which of the following presents the strongest constitutional argument that the organization can make against the validity of the commission? A The commerce power does not extend to activities occurring solely within a state. B Legislative power may not be delegated by Congress to an agency in the absence of clear guidelines. C The organization is denied due process of law because it is not represented on the commission. D The commission lacks authority to enforce its standards because some of its members were appointed by Congress.

A

The strongest argument is that the commission lacks authority to enforce its standards. The Appointments Clause of the Constitution permits Congress to vest appointments of inferior officers only in the President, the courts, or the heads of departments. Enforcement is an executive act; therefore, Congress cannot appoint members of a commission that exercises enforcement powers. In these facts, the safety commission consists of some members appointed by Congress. This commission therefore violates the Appointments Clause of the Constitution because it has enforcement powers, since it can prosecute violations. Legislative power can be delegated even under vague guidelines without creating an unconstitutional delegation. Thus, (B) is incorrect. (A) is incorrect because the commerce power extends to many activities occurring solely within a particular state, if, taken as a whole, the activities in question have a substantial economic effect on interstate commerce. There is no constitutional provision requiring that a party or group to be regulated be represented on the commission that regulates it. (C) is therefore incorrect.

77
Q

A woman was crossing the street at a crosswalk, but did not look both ways. The woman was hit by a truck, and immediately afterwards, she was struck by a car. As a result of these collisions with the vehicles, the woman suffered severe injuries. Although it was impossible to determine which portion of the woman’s injuries was caused by the driver of the car and which was caused by the truck driver, at the trial of the woman’s suit, the jury determined that the driver of the car was 20% negligent, that the truck driver was 40% negligent, and that the woman was 40% negligent. It was further determined that the woman had suffered $100,000 in damages. The woman had already received $10,000 from her group medical insurance plan. The driver of the car had a $500,000 auto liability insurance policy, and the truck driver is now insolvent. How much will the woman recover in damages from the driver of the car? A $90,000. B $60,000. C $50,000. D $20,000.

A

Answer Discussion - Correct The woman will recover $60,000. Under pure comparative negligence, which you should assume is the rule unless the question specifies otherwise, the plaintiff may recover no matter how great her negligence. In this case, the woman has suffered damages of $100,000. Because she was 40% negligent, she may recover only $60,000 ($100,000 less $40,000). Therefore, (B) is correct, and (A) is wrong. Absent a statute, damages are not reduced or mitigated because of benefits received from collateral sources (e.g., health insurance). Thus, the woman’s receipt of $10,000 under her insurance plan does not diminish her recovery. (C) is therefore wrong. The driver of the car and the truck driver are jointly and severally liable for the woman’s injuries because their negligent acts combined to proximately cause an indivisible injury to the woman. Because the driver of the car and the truck driver are jointly and severally liable, the woman may recover the entire $60,000 from the driver of the car. Thus, (D) is wrong.

78
Q

The owner of a small computer consulting firm was attending the annual trade meeting of the computer industry and spoke with the owner of a second consulting firm about doing joint projects. The owner of the second firm replied by rejecting the idea immediately, stating that she believed that the first owner was incompetent. A sales representative of a computer supply firm overheard the remark. The owner of the first firm sued the owner of the second firm for defamation. If the first owner does not prevail in this lawsuit, what will be the likely reason? A It was not reasonably foreseeable that the second owner’s remark would be overheard. B The second owner did not know that her remark would be overheard. C There was no publication. D The sales representative was not a party to the conversation.

A

If the plaintiff does not prevail, it will be because it was not reasonably foreseeable that the defendant’s remark would be overheard, and therefore the fault requirement for the publication element would not be satisfied. To establish a prima facie case for defamation, the following elements must be proved: (i) defamatory language on the part of the defendant; (ii) the defamatory language must be “of or concerning” the plaintiff (i.e., it must identify the plaintiff to a reasonable reader, listener, or viewer); (iii) publication of the defamatory language by the defendant to a third person; and (iv) damage to the reputation of the plaintiff. The second owner’s statement constitutes defamatory language because it adversely affects the first owner’s reputation by attacking his competency. The publication requirement is satisfied when there is a communication to a third person who understands it. However, the communication to the third person must be made either intentionally or negligently; if it was not reasonably foreseeable that the defamatory statement would be overheard by the sales representative, the fault requirement for the publication element is not satisfied. (B) is incorrect. The plaintiff could prevail even if the defendant did not know that her remark would be overheard as long as it was reasonably foreseeable that it could be overheard. (C) is incorrect because there in fact was a publication, i.e., there was a communication, albeit not intentionally made, to the sales representative, who would reasonably have understood it to be defamatory. (D) is incorrect because there is no requirement that the third party be a party to the conversation-the third party need only be a reader, listener, or viewer.

79
Q

In lieu of a snack vending machine, a small company put candy bars into a box in the office kitchen. Using the honor system, employees were asked to put a dollar into an envelope in the box if they took out a candy bar. The office manager would periodically take the money collected in the envelope and use it to purchase new candy. One night while cleaning the kitchen, the company’s night janitor impulsively took all the money from the envelope. His actions were recorded on the company’s newly installed security camera. The janitor later admitted that he took the money to fund illegal dogfights. Which of the following crimes did the janitor commit? A Embezzlement, because the janitor took money that rightfully belonged to his employer. B Larceny, because he did not intend to purchase replacement candy with the money. C Robbery, because the theft was motivated by an underlying felony. D No crime, because the money in the envelope was voluntarily abandoned by his co-workers.

A

The janitor is guilty of larceny, which consists of the taking and carrying away of the tangible personal property of another by trespass, with intent to permanently deprive the person of his interest in the property. The janitor’s actions meet this definition. (A) is wrong because the janitor was not the employee responsible for the candy money. Embezzlement requires the fraudulent conversion of property of another by a person in lawful possession of that property. Here the janitor was never in lawful possession of the money so it was not an act of embezzlement. (C) is also wrong. Motive is not what distinguishes larceny from robbery. Robbery is defined as a taking of personal property of another from the other’s person or presence, by force or intimidation, with the intent to permanently deprive her of it. Robbery is essentially an aggravated form of larceny in which the taking is accomplished by force or threat of force. Here there was no force involved. (D) is wrong because, even though the employees were on their honor to contribute money for the candy, the money was not just abandoned by them. The money was exchanged for candy, originally purchased by the company, and thus belonged to the company at the time the janitor took it.

80
Q

A wholesale seller sent a fax to a manufacturer with whom he had done business before: “Send 500 ‘Madewell’ chairs at your usual price.” The manufacturer responded, also by fax, that the line was being discontinued, but he would ship his last 500 chairs at $75 per chair, his usual price. The manufacturer immediately began the paperwork for processing the order and started preparing and packing the chairs for shipment. Before the chairs could be delivered, the wholesaler canceled his order, noting that the price was too high. The day after receiving the wholesaler’s cancellation, the manufacturer sold the chairs to another buyer for $75 each. If the manufacturer sues the wholesaler for damages, how much should he recover? A Nothing, because this was a contract between merchants and the wholesaler canceled within a reasonable time. B Nothing, because the manufacturer was able to cover by selling the chairs at the same price he would have received from the wholesaler. C $37,500, the full contract price, because the wholesaler breached the contract and $75 per chair was a fair price. D The incidental costs of preparing the paperwork and other office costs connected with preparing and packing the chairs for shipment to the wholesaler.

A

The manufacturer will recover only his incidental damages, i.e., the costs of preparing to ship the chairs. An offer calling for shipment of goods, such as the offer here, may be accepted by prompt shipment with notice or by a promise to ship. Acceptance forms a contract. Here, the manufacturer accepted the wholesaler’s offer by promising to ship, and a contract was formed. The wholesaler breached the contract by canceling his order. When a buyer breaches by repudiating his offer, as the wholesaler did here, the seller has a right to recover his incidental damages plus either the difference between the contract price and the market price or the difference between the contract price and the resale price of the goods, reduced in either case by any expenses saved as a result of the breach. Here, the manufacturer made what he would have if the sale with the wholesaler had gone through-there was no difference between the contract price and the resale price. Thus, the manufacturer would be limited to his incidental damages. Lost profits would not be available because the chairs were the last ones that the manufacturer had and would have because the line was being discontinued. Therefore, the manufacturer would not have been able to sell another set of 500 chairs to another potential buyer. (A) is incorrect because there is no rule under the UCC, which governs the contract here, that makes contracts between merchants cancelable within a reasonable time. (B) is incorrect because, as indicated above, the UCC allows the seller to recover incidental damages. (C) is incorrect because the UCC seeks only to put the nonbreaching party in as good a position as it would have been in had the other party performed. Here, awarding the manufacturer the full contract price would put him in a better position than performance would have, because it would give him a double recovery for selling the same goods. (The result would be different, however, if the manufacturer had had more chairs to sell, because in that case, the breach would have cost the manufacturer additional sales-i.e., he could have sold to the wholesale seller and to the second buyer.)

81
Q

Suspecting criminal activity, a police officer acting without a warrant peeked through a small opening in the shutters of an apartment. The officer observed the apartment’s tenant and the defendant making methamphetamine. The officer immediately entered the apartment and arrested the tenant and the defendant, and he confiscated the ingredients for the methamphetamine, the tools used for methamphetamine production, and any completed methamphetamine for evidence. The search is later ruled invalid at a suppression hearing. May the defendant now claim that her Fourth Amendment rights have been violated by the seizure of the ingredients, tools, and methamphetamine from the apartment? A Yes, because the items will be used in evidence against her. B Yes, if she was an overnight guest of the tenant. C No, because she was not the owner or occupier of the apartment. D No, unless she admits to ownership of the items.

A

The defendant can claim a reasonable expectation of privacy for Fourth Amendment purposes if she was an overnight guest of the owner of the place searched. To raise a Fourth Amendment claim of an unreasonable search or seizure, a person must have a reasonable expectation of privacy with respect to the place searched or the item seized. It is not enough merely that someone has an expectation of privacy in the place searched. The Supreme Court has imposed a standing requirement so that a person can complain about an evidentiary search or seizure only if it violates her own reasonable expectations of privacy. The Court has held that a person has a reasonable expectation of privacy any time (i) she owned or had a right to possession of the place searched, (ii) the place searched was in fact her own home, whether or not she owned or had a right to possession of it, or (iii) she was an overnight guest of the owner of the place searched. Thus, the defendant would have standing to challenge the search of the tenant’s apartment if she was an overnight guest of the tenant. (A) is incorrect because standing to raise a Fourth Amendment claim does not exist merely because a person will be harmed by introduction of evidence seized during an illegal search of a third person’s property. The person must establish that her own reasonable expectation of privacy has been violated. (C) is wrong because the fact that the defendant was not the owner or occupier of the apartment does not preclude her from challenging the search. As discussed above, an overnight guest may also have a reasonable expectation of privacy in the premises for purposes of the Fourth Amendment. (D) is incorrect. Although the defendant may have standing to object to the seizure of items if she claims ownership of them, that is not the only basis for raising a Fourth Amendment claim; she will have standing to object to the search of the apartment under the circumstances in (B) regardless of whether she claims ownership of the methamphetamine.

82
Q

Pursuant to a contract, a landscaper performed $30,000 of landscape work for a homeowner. By coincidence, the homeowner and the landscaper were involved in an automobile accident that was unrelated to the landscape work. The homeowner was injured in the accident and sued the landscaper in federal district court for negligence, seeking $100,000 in damages. The homeowner and the landscaper are citizens of different states. May the landscaper assert and maintain a counterclaim against the homeowner for breach of contract, seeking the $30,000 due under the landscape contract? Press Enter or Space to submit the answer A No, because the Federal Rules of Civil Procedure permit a counterclaim only if the counterclaim arises from the same transaction or occurrence as the plaintiff’s initial claim. Correct B No, because, while the court has diversity of citizenship jurisdiction over the homeowner’s negligence claim and the Federal Rules of Civil Procedure permit the counterclaim, the federal court does not have subject matter jurisdiction over the landscaper’s contract claim. C Yes, because the Federal Rules of Civil Procedure permit the counterclaim, and the court has diversity of citizenship jurisdiction over both claims. D Yes, because the court has diversity of citizenship jurisdiction over the homeowner’s negligence claim, and it has supplemental jurisdiction over the landscaper’s contract claim because it is a compulsory counterclaim.

A

The landscaper cannot assert and maintain a counterclaim against the homeowner for breach of contract because the federal court does not have subject matter jurisdiction over the contract claim. The court has diversity of citizenship jurisdiction over the negligence claim because there is complete diversity and the amount in controversy exceeds $75,000. A compulsory counterclaim (a claim that arises out of the same transaction or occurrence as the plaintiff’s claim) does not need to meet the jurisdictional amount requirement for diversity jurisdiction. However, a permissive counterclaim (a claim that arises out of an unrelated transaction) must meet the jurisdictional amount requirement. Here, the landscaper’s contract claim would be a permissive counterclaim because it is unrelated to the negligence claim arising from the accident. Therefore, the claim’s amount in controversy would have to exceed $75,000. The claim is only for $30,000; therefore, the court does not have subject matter jurisdiction over the contract claim. (A) is incorrect because a defendant may assert claims against a plaintiff that are unrelated to the plaintiff’s claims, provided there is subject matter jurisdiction. (C) is incorrect because the court does not have diversity jurisdiction over the contract claim. (D) is incorrect because the contract claim is not a compulsory counterclaim.

83
Q

A landlord entered into a written lease of a bakery for a term of 25 years with a baker. The parties agreed to a right of first refusal if the bakery was offered for sale during the term of the lease. The lease also permitted assignments and subleases on notice to the landlord. Three years later, the baker retired and, after notifying the landlord, transferred the lease to a chocolatier. Twenty-one years later, the landlord entered into a contract with a buyer for the sale of the bakery for $100,000. The landlord had informed the buyer of the lease but had forgotten about the right of first refusal. When the chocolatier learned of the sale to the buyer, she informed both the landlord and the buyer that she wanted to exercise her option and was prepared to purchase the bakery for the contract price. The jurisdiction’s Rule Against Perpetuities is unmodified by statute. Can the chocolatier enforce the option? A Yes, because an option held by a tenant on leased property cannot be separated from the leasehold interest. B Yes, because the option touches and concerns the leasehold estate. C No, because the transfer to the chocolatier made the option void under the Rule Against Perpetuities. D No, because the option was not specifically included when the lease was transferred to the chocolatier.

A

The chocolatier can enforce the option to purchase because it is a covenant that runs with the land. When a tenant makes a complete transfer of the entire remaining term of his leasehold interest, it constitutes an assignment. The assignee and the landlord are then in privity of estate, and each is liable to the other on all covenants in the lease that run with the land. The covenant runs with the land if the original parties so intend and the covenant “touches and concerns” the leased land, i.e., burdens the landlord and benefits the tenant with respect to their interests in the property. Here, the transfer of the lease to the chocolatier was an assignment, making all covenants in the lease that run with the land enforceable by the assignee. The right of first refusal burdens the landlord’s power of alienation over the bakery, and there is nothing to indicate that the parties intended the option to be personal to the baker. Hence, the chocolatier can enforce the option and purchase the property. (A) is incorrect because most courts do not bar an option from being separated from the leasehold interest if that is the parties’ intent. The tenant may transfer the leasehold interest while retaining the option to purchase, or vice versa. Whether the option in this case stayed with the leasehold interest depends on whether it was a covenant that runs with the land. (C) is incorrect because options and rights of first refusal are not subject to the Rule Against Perpetuities when connected to leaseholds. If a tenant assigns the leasehold, the option generally is considered a running covenant, exercisable by the assignee absent a contrary intent. If the option had been separated from the leasehold estate, so that it was no longer exercisable by the tenant, the Rule would have become applicable to the option (and it would have invalidated the option here because it could have been exercised more than 21 years after a life in being). Here, however, the option was not severed from the leasehold; the entire interest was transferred to the chocolatier as the new tenant. Hence, the Rule Against Perpetuities is not applicable. (D) is incorrect because, as discussed above, the option is a covenant that runs with the land regardless of whether it was specified in the assignment to the chocolatier. The chocolatier, as the assignee of the leasehold, can enforce the option on privity of estate grounds.

84
Q

A landowner owned two adjoining parcels of land. The landowner sold the western parcel to a buyer by a deed that contained the following clause: “Grantee promises for himself, his heirs, successors, and assigns to not erect a structure over two stories on the land.” The buyer recorded the deed and built a two-story house on the property and lived there for 30 years, after which he sold the land to a movie star by a deed that did not contain the structure height restriction. The movie star decided to tear down the existing house on the land and erect a three-story house. Her neighbor, who purchased the eastern parcel from the landowner 10 years earlier, discovers that the movie star’s house will be larger than his and files suit to enforce the covenant at law. Who will prevail? A The neighbor, because the restrictive covenant runs with the land. B The neighbor, because privity is not required to enforce a covenant at law. C The movie star, because her deed did not contain the restrictive covenant. D The movie star, because she had no notice of the structure height restriction.

A

The neighbor will prevail because the restrictive covenant runs with the neighbor’s land, which is benefited, and with the movie star’s land, which is burdened. A covenant at law will run with the land and be enforceable against subsequent grantees if: (i) the contracting parties intended it to run; (ii) there is privity of estate between the original promisor and promisee (horizontal privity), as well as between the promisor and his successor (vertical privity); (iii) the covenant touches and concerns the property; and (iv) the burdened party has notice of the covenant. Here, the use of the words “heirs, successors, and assigns” in the covenant shows the intent for the covenant to run. The original parties were in horizontal privity because at the time the buyer entered into the covenant, he and the landowner shared an interest in the land independent of the covenant-as grantor and grantee. Because the neighbor and the movie star hold the entire interest in the parcels held by the landowner and the buyer, respectively, there is vertical privity. The covenant touches and concerns the land because it diminishes the movie star’s rights in connection with her enjoyment of the western parcel. Finally, the movie star has notice of the covenant because it was recorded in her chain of title (in the landowner-buyer deed). Thus, the covenant runs with the land. (B) is incorrect because privity is required to enforce a covenant at law. Moreover, privity does exist under these facts, as explained above. (C) is incorrect. The fact that the structure height restriction was not contained in the movie star’s deed does not shield her from enforcement of the covenant because it was contained in a previous deed relating to the property (record notice). (D) is incorrect because the movie star had at least record notice of the restriction.

85
Q

In an action by the plaintiff against the defendant, one of the issues is whether the defendant is a licensed physical therapist. Normally, the names of all licensed physical therapists are registered with the office of the state Department of Professional Registrations. The plaintiff wishes to introduce a certified document, signed by the chief registrar of the department (who cannot be located), stating that an examination of the department’s rolls does not disclose the defendant’s name. Should the document be admitted? A Yes, because a statement of absence from public record is admissible. B Yes, because the chief registrar is unavailable. C No, because the document is hearsay not within an exception. D No, because the document is not self-authenticating.

A

The document should be admitted. Related to the exception to the hearsay rule for public records and other official writings, Federal Rule of Evidence 803(10) provides that evidence in the form of a certification or testimony from the custodian of public records that she has diligently searched and failed to find a record is admissible to prove that a matter was not recorded, or, inferentially, that a matter did not occur. Here, the defendant’s status as a licensed physical therapist would normally be revealed in the records of the department. The document here at issue is admissible, under the foregoing hearsay exception, as a means of proving that the defendant is in fact not licensed. (B) is incorrect because this hearsay exception does not require unavailability of the declarant. (C) is incorrect because, as explained above, the statement of absence from public record forms an exception to the hearsay rule. (D) is incorrect because a public document that has been signed and certified is self-authenticating under Rule 902; hence, no testimonial sponsorship for the document is required.

86
Q

A pedestrian sued a driver after the driver’s car hit the pedestrian at an intersection. The pedestrian claims that the driver ran a stop sign and the driver contends that the pedestrian darted out into traffic. The pedestrian seeks to call her friend to the stand to testify that the pedestrian is an extremely cautious person who invariably obeys traffic laws. Should the judge admit the friend’s testimony? A Yes, because it is proper habit evidence. B Yes, because it tends to show that the pedestrian is less likely to have darted out into traffic. C No, because the pedestrian’s character is not directly in issue. D No, because the friend is a biased witness.

A

This is character evidence because it describes the pedestrian’s general tendency to act cautiously. Evidence of a person’s character is generally irrelevant and inadmissible in a civil case except in rare circumstances where character is directly in issue (e.g., defamation or negligent hiring cases). Here, because the pedestrian’s character is not directly in issue in this civil personal injury lawsuit, the friend’s testimony is inadmissible. (A) is incorrect. Under Federal Rule 406, evidence of a person’s habit may be admitted to prove that on a particular occasion the person acted in accordance with the habit. Habit describes one’s regular response to a specific set of circumstances, while character describes one’s disposition in respect to general traits. Since habits are more specific and particularized, evidence of habit is relevant and can be introduced in circumstances when it is not permissible to introduce evidence of character. Here, however, the friend’s testimony is character evidence, not habit evidence. The fact that the pedestrian is cautious and obeys traffic laws is not nearly specific enough to qualify as habit evidence-despite the use of the term “invariably.” (B) is incorrect. The answer choice recites the general rule of relevance-that is, evidence is relevant if it makes the existence of any fact of consequence to the determination of the action more likely or less likely than it would be without the evidence. However, despite meeting the low threshold for relevance, character evidence is generally inadmissible in civil cases to prove that a person acted in conformity with the trait during the events of the case. (D) is incorrect. Although the credibility of a witness may be impeached by showing that the witness is biased, that does not mean the testimony itself is inadmissible. Indeed, most witnesses testifying as to the habit or character of a person will have some sort of personal bias in favor of or against that person.QUESTION ID: ED06

87
Q

Congress created the National Agency for Burglar Alarms (“NABA”), giving it the power to regulate both burglar alarm hardware and installation personnel. NABA adopted a regulation requiring that all burglar alarm installation companies be licensed, and providing that anyone installing an alarm without a license could be fined. The regulation also provided that any company in the installation business on the day the regulation was adopted automatically would receive a license, but to obtain a license thereafter, an applicant would have to show that he has worked as an installer at a licensed company for at least three years. A man who has been installing alarm systems for eight years sold his installation business a few months before the NABA regulation was adopted and went to work for the purchaser servicing his old accounts. A few months after the NABA regulation was adopted, a representative from a national department store chain approached the man with an offer to hire him as an independent contractor to revamp the chain’s alarm systems. The man quit his job and applied for a NABA installer’s license. His application was denied because he was not in business on the day the NABA regulation was adopted and had worked for a licensed installer for only a few months. The man decided to install the alarm systems anyway. Can the man properly be fined for installing alarms? A Yes, because the NABA was established under Congress’s power to legislate for the general welfare, and Congress may take whatever steps are necessary and proper to enforce its laws. B Yes, because the regulation falls within the scope of Congress’s commerce power, and Congress may delegate its authority to regulate as it has done here. C No, because the regulation interferes with the man’s fundamental right to earn a living without a substantial justification and so violates the Privileges and Immunities Clause of Article IV, Section 2. D No, because a government agency cannot itself levy fines for a violation of its regulations.

A

The man can be fined. Congress has the power to regulate alarm installation companies under the Commerce Clause because the clause permits Congress to regulate any local or interstate activity that, either in itself or in combination with other activities, has an effect on interstate commerce. Burglar alarm companies use instrumentalities of interstate commerce such as phone lines and have a cumulative effect on interstate commerce even though some may only do business locally. Hence, their activities can be regulated by Congress. The delegation to the NABA is valid because Congress has broad discretion to delegate its legislative power; the Supreme Court will uphold almost any delegation of congressional power. Therefore, the man can be fined. (A) is incorrect because it improperly mixes two concepts. Congress does not have the power to legislate for the general welfare-there is no federal police power-but rather Congress has the power to spend for the general welfare. (C) is incorrect because, even assuming that the regulation here interferes with the man’s right to make a living, it would not violate the interstate Privileges and Immunities Clause of Article IV because the clause restricts states, not the federal government. (D) is incorrect. Congress can provide that violation of an agency’s regulations is a criminal offense that can be enforced through the imposition of fines. Furthermore, an agency has the power to impose civil fines and penalties for a violation of its regulations.

88
Q

A boy planned to rob the local currency exchange, but he needed a getaway car and somebody to be a lookout. He asked his sister, who immediately refused and warned him against carrying it out. The boy showed his sister a gun, threatening to shoot her if she did not go along with the plan. The sister consented to help, and the pair left for the currency exchange, with the sister driving. When they arrived, the sister left the car running so that she could get away and contact the police while her brother was inside, but the boy ordered her to go inside with him. During the robbery attempt, the clerk pulled out a gun, and the boy and the clerk shot and killed each other. If the sister is charged with the killing of the store clerk, should the jury find her criminally liable if it accepts the sister’s version of the facts? A Yes, because duress is not a defense to murder. B Yes, because the clerk’s death occurred in furtherance of and during the perpetration of a robbery in which the sister was participating. C No, because the sister lacked the requisite intent. D No, because the sister withdrew from the conspiracy to commit robbery and thus is not liable for any acts committed in furtherance of the conspiracy.

A

The sister is not criminally liable for the store clerk’s death. A defendant may, by virtue of her participation in a conspiracy, be liable for the crimes of all other conspirators if the crimes were committed in furtherance of the objectives of the conspiracy and were a natural and probable consequence of the conspiracy (i.e., were foreseeable). Furthermore, liability for felony murder would attach if the killing were committed during the course of a felony. Thus, the sister would be criminally liable for the killing of the clerk if she were a co-conspirator or if she were a principal to the underlying felony (the robbery). However, under the facts, the sister is neither because she lacked the requisite intent. Given the threat to her life, the sister did not freely enter into a conspiracy to commit robbery. (A) is incorrect because, while duress is not a defense to murder, it is a defense to the underlying felony, and thus is a defense to the felony murder charge. Here, the sister was acting under duress during the robbery, and thus she has a defense to the robbery charge. (B) is incorrect because, as stated above, under these facts duress is a good defense to the robbery charge and thus would be a good defense to the felony murder charge. (D) is also incorrect. Although a valid withdrawal would be a good defense to subsequent crimes of any co-conspirator, it is doubtful whether the sister in fact entered into a conspiracy in the first place. Because she never had the intent required for participation in the conspiracy, the jury would not be required to consider her defense of withdrawal.

89
Q

The plaintiff was exiting from a parking garage owned and operated by the city when he discovered that the exit ramp was blocked by construction barricades and a pile of broken-up concrete. No workers or detour signs were around and the plaintiff was in a hurry, so he backed up and drove down an entrance ramp that was clearly marked as such. As he came around a corner, his car was broadsided by a pickup truck. The plaintiff was seriously injured in the collision. A statute in the jurisdiction requires drivers to obey all traffic directional markings in both public and private parking lots and garages. The jurisdiction retains governmental immunity for municipalities. If the plaintiff brings a lawsuit against the city to recover for his injuries, which of the following facts will be LEAST helpful in the city’s defense? A The plaintiff was aware that another exit on the other side of the garage was open. B The construction workers responsible for blocking off the exit ramp were employees of an independent contractor rather than the city. C The city does not collect fees or make a profit in the operation of the garage. D The pickup truck driver could have avoided the plaintiff but recognized him as an old enemy and deliberately ran into him.

A

The fact least helpful to the city’s defense of the plaintiff’s lawsuit is the identity of the workers who blocked the exit ramp. Under vicarious liability rules, a principal will be liable for the tortious acts of an independent contractor if the duty is nondelegable on public policy grounds; included is the duty of a possessor of land to keep its premises safe for its invitees. If the workers were negligent in leaving the ramp blocked without providing another means of exiting, the fact that they were not city employees would not absolve the city of liability; hence, their identity would be of no help to the city’s defense. (A) is incorrect because if the plaintiff was aware of an alternate route, he may have been contributorily negligent in exiting down the entrance ramp. A plaintiff’s contributory negligence may be established by violation of an applicable statute. However, as with a statutory duty imposed on a defendant, the plaintiff’s violation of the statute may be excused if compliance was beyond the plaintiff’s control. If no other means of exiting the garage were known to the plaintiff, he may be excused for violating the traffic statute; however, if he knew of an alternative exit, the city will probably be able to establish contributory negligence on his part by his violation of the statute, reducing his potential recovery. (C) is incorrect because whether the city collects fees and makes a profit in operation of the garage will be considered by the court in determining whether the jurisdiction’s governmental immunity applies. Where municipal immunity still exists, courts have limited its scope by differentiating between “governmental” and “proprietary” functions of the municipality. If the municipality is performing a function that might as well have been provided by a private corporation, the function may be construed as a proprietary one and no immunity will attach. The inference that a function is proprietary will be strengthened where the city collects revenues by virtue of providing the service. Hence, the fact that the city is not collecting revenues or making a profit in operating the garage will make it less likely that the function will be deemed to be proprietary and more likely that it will be deemed to be governmental and thus immune; in other words, it will be more helpful rather than less helpful in the city’s defense. (D) is incorrect because the pickup truck driver’s conduct under these circumstances would be deemed a superseding force that breaks the causal connection between any negligence on the part of the city and the plaintiff’s injury. Assuming that the city workers were negligent, the fact that an independent intervening force caused the injury generally would not cut off the city’s liability, because its negligence created a foreseeable risk of that harm occurring. However, where this foreseeable harm is caused by an unforeseeable crime or intentional tort of a third party, most courts would not hold the city liable, treating the crime or tort as a superseding force. Here, while blocking the exit ramp created a foreseeable risk that someone might collide with the plaintiff, it was not foreseeable that his enemy would take that opportunity to commit an intentional tort against him. Because the pickup truck driver’s conduct was unforeseeable under the circumstances in choice (D), the city would be relieved of liability for any negligence in blocking the ramp.

90
Q

The owner of an apartment building contracted with a painter to paint the porches of the apartments for $5,000. The contract was specifically made subject to the owner’s good faith approval of the work. The painter finished painting the porches. The owner inspected the porches and believed in good faith that the painter had done a bad job. The painter demanded payment, but the owner told him that the paint job was poor and refused to pay. The painter pleaded that he was desperately in need of money. The owner told the painter that she would pay him $4,500, provided he repainted the porches. The painter reluctantly agreed, and the owner gave the painter a check in the amount of $4,500. The painter went to his bank, indorsed the check “under protest” and signed his name, then deposited the check in his account. He never returned to repaint the porches. The painter sues the owner for $500, which he believes is still owed to him on his contract to paint the porches. Will he prevail? A Yes, because he indorsed the check “under protest.” B Yes, but only if he repaints the porches. C Yes, because he performed the contract by painting the porches the first time. D No, even if he repaints the porches.

A

Highlight Answer Discussion - Correct The painter will be unable to recover the $500 because he did not satisfy the condition precedent to payment under the contract. A party does not have a duty to perform if a condition precedent to that performance has not been met. Here, the parties made the owner’s satisfaction with the painter’s paint job a condition precedent to the owner’s duty to pay the $5,000. Because the owner was not satisfied with the paint job, her duty to pay the painter never arose. The fact that the owner offered to give the painter $4,500 if he repainted the porches has no effect on this analysis, because the offer constituted a new contract, the owner having been excused from the old one. (A) is wrong because it does not matter whether the painter indorsed under protest. The indorsement will not change the result here because the new contract did not seek to discharge any contractual duty-the owner was already excused from her duties because the condition precedent was never met. (B) is wrong because the old contract, which provided for payment of $5,000, is considered to be at an end. Under the terms of the new contract, the painter is entitled to only $4,500, provided he repaints the porches. (C) is wrong because the condition precedent to the payment of $5,000, the owner’s satisfaction, was not met. The courts have held such conditions to be valid-not illusory promises-because of the promisor’s duty to exercise good faith in assessing satisfaction. Here, the facts state that the owner believed in good faith that the painter had done a bad job; thus, the painter is not entitled to payment under the original contract. Note that since he has not performed under the new contract, he is in breach and not entitled to the $4,500 already paid.

91
Q

The defendant was arrested, given Miranda warnings, and charged with burglary. At the police station, he telephoned his mother and asked her to come to the station to post bail. Instead, his mother immediately called the family attorney. In the meantime, the police had begun questioning the defendant. Although he never told the police to stop the questioning, his answers were at first vague or clearly unresponsive. During the course of the questioning, the family attorney phoned the station and told the police that she had been hired to represent the defendant and would be there in half an hour. The police did not inform the defendant of the attorney’s call. Ten minutes later, the defendant admitted to committing the burglary, and signed a statement to that effect prepared by the police. The attorney arrived a few minutes later and advised the defendant to remain silent, but he told her that he had already signed a confession. How should the court rule on the attorney’s pretrial motion to exclude the confession as evidence at trial? Press Enter or Space to submit the answer A Grant the motion, because the police had a duty to inform the defendant that an attorney was coming to represent him. B Grant the motion, because the defendant has been deprived of his Sixth Amendment right to counsel. C Deny the motion, because the defendant’s statement admitting the crime was voluntary. D Deny the motion, because the defendant waived his Miranda rights.

A

Answer Discussion - Incorrect The defendant’s confession should be admitted because he waived his Fifth Amendment privilege against compelled self-incrimination after receiving Miranda warnings. Miranda v. Arizona requires that a person in custody be informed of his right to remain silent and his right to the presence of an attorney during questioning. A suspect may subsequently waive his rights by making a confession, as long as the waiver was knowing and voluntary. In this case, the defendant received proper Miranda warnings, and there is no indication that he did not understand what his rights were. Although his answers during questioning were initially unresponsive, he never asked for an attorney or indicated that he wished to remain silent, and he voluntarily confessed after a relatively short period of interrogation. Hence, he validly waived his Miranda rights. (A) is incorrect because the police have no duty to inform the defendant that an attorney is attempting to see him. The defendant’s ignorance of his attorney’s efforts has no bearing on whether he made a knowing waiver of his Miranda rights. (B) is incorrect because the defendant’s right to counsel was not violated. Although the defendant does have a separate Sixth Amendment right to counsel under Escobedo v. Illinois because he has already been arrested and charged with the crime, this right would only be violated if the defendant, after being informed of his right to counsel, had requested an attorney or had been prevented from seeing his attorney. Here, he made no request to see an attorney-even when he called his mother-and his attorney was allowed to see him immediately upon her arrival. Thus, he has waived his Sixth Amendment right to counsel. (C) is incorrect even though it is true that the defendant made a voluntary statement. Due process requires that for confessions to be admissible, they must be “voluntary,” based on the totality of the circumstances, and here all of the circumstances indicate that the defendant’s confession was voluntary. However, even a voluntary confession will be inadmissible if it was obtained in violation of Miranda rights. (D) is therefore a better choice than (C).

92
Q

defendant was charged with the murder of a victim. During the course of the criminal trial, a witness testified on behalf of the defense that, at the time the murder took place, he saw someone who looked like the defendant dancing at a local nightclub. The defendant is eventually acquitted of the charge. Following the acquittal, the appropriate survivors of the victim bring a wrongful death action against the defendant. As part of her defense, the defendant wishes to introduce the testimony given at the criminal trial by the witness, who the defendant shows is now incarcerated in a prison in another state. Is the testimony of the witness admissible? A Yes, because the witness testified under oath at another hearing related to the same subject matter. B Yes, because the defendant is a party to both proceedings. C No, because the plaintiffs were not parties to the criminal proceeding. D No, because the witness can be subpoenaed to testify.

A

The witness’s testimony is inadmissible. Under Federal Rule 804(b)(1), the testimony of a witness who is unavailable, given at another hearing, is admissible in a subsequent trial if there is sufficient similarity of parties and issues so that the opportunity to develop testimony or cross-examination at the prior hearing was meaningful. The former testimony is admissible upon any trial of the same subject matter. The party against whom the testimony is offered or, in civil cases, the party’s predecessor in interest must have been a party in the former action. “Predecessor in interest” includes one in a privity relationship with the party, such as grantor-grantee, testator-executor, life tenant-remainderman, and joint tenants. These requirements are intended to ensure that the party against whom the testimony is offered (or a predecessor in interest in a civil case) had an adequate opportunity and motive to cross-examine the witness. In the civil suit here at issue, the survivors of the victim were not parties to the criminal case, nor were they in privity with any such party. (The parties to that case were the defendant and the government.) These survivors, who are the plaintiffs in the instant litigation, are the parties against whom the testimony of the witness is being offered. Because they were not parties to the action in which the witness testified, they had no opportunity to cross-examine him. Even if the government had a similar motive to cross-examine the witness as do the plaintiffs in the current action, that is not sufficient to make the government a predecessor in interest to the plaintiffs. Consequently, the testimony of the witness does not come within the former testimony exception to the hearsay rule, and the testimony is inadmissible hearsay. (A) and (B) incorrectly conclude that the testimony is admissible. Although it is true that the witness testified at an earlier hearing related to the same subject matter, and that the defendant is a party to both proceedings, what is missing is the requisite identity of parties against whom the testimony is being offered. (D) is incorrect because a witness incarcerated in another state is “unavailable” for purposes of civil proceedings. Under the Federal Rules, a witness is unavailable if he is absent from the hearing and the proponent of the statement is unable to procure the declarant’s attendance by process or other reasonable means. The Supreme Court has held that the Confrontation Clause requires a greater showing of “unavailability” in criminal cases than in civil cases. Because all states permit extradition of witnesses against the accused in criminal cases, a mere showing that a witness is incarcerated in a prison outside the state is insufficient to establish “unavailability.” In contrast, the reach of process in civil cases is more limited and the Confrontation Clause does not apply. A mere showing that the witness is incarcerated in a prison out of state will suffice to show unavailability in a civil case.

93
Q

A victim and his former business partner, the defendant, had a bitter falling out after the victim accused the defendant of embezzling company funds. The defendant threatened to get even. Shortly thereafter, while driving on the expressway, a car swerved suddenly in front of the victim’s car. Although the victim applied the brakes immediately, his car failed to stop. To avoid colliding with the car ahead of him, he swerved to the right and smashed into a concrete retaining wall. A passing motorist stopped and came to the aid of the victim. Bleeding profusely from a head wound, and rapidly losing consciousness, the victim said, “I don’t think I’m going to make it. I tried to slow down, but my brakes didn’t work. My former partner must have tampered with them to get back at me.” With that, the victim lapsed into unconsciousness, and has been in a coma and on life support ever since. A personal injury suit has been filed on his behalf by a court-appointed guardian against the defendant. At trial, can the motorist testify as to the statement made by the victim? A No, because the victim did not know that the defendant tampered with the brakes. B No, because the victim is still alive. C Yes, because the victim thought he was about to die. D Yes, because this is a civil case.

A

Testimony as to the statement made by the victim is inadmissible as a statement under belief of impending death, because the victim did not actually have firsthand knowledge that the defendant was responsible for the collision. The statement is hearsay because it is a statement made by the declarant (the victim), other than while testifying, offered to prove the truth of the matter asserted therein. Here, the plaintiff wants to present this testimony to prove the truth of the statement that the defendant was responsible for the brake failure, and will argue that the statement falls under the hearsay exception for dying declarations. In a civil case or a homicide prosecution, a statement made by a now unavailable declarant while believing his death to be imminent, that concerns the cause or circumstances of what he believed to be his impending death, is admissible. [Fed. R. Evid. 804(b)(2)] For this exception to apply, the declarant need not actually die. Rather, the declarant must be “unavailable” when the statement is offered. A declarant is unavailable if he: (i) is exempted from testifying on the ground of privilege, (ii) refuses to testify despite a court order, (iii) testifies to lack of memory of the subject matter of the statement, (iv) cannot be present or testify because of death or physical or mental illness, or (v) is beyond the reach of the court’s subpoena and the statement’s proponent has been unable to procure his attendance or testimony by process or other reasonable means. Regarding the statement at issue here, the victim certainly thought he was about to die from his injuries. In addition, he is unavailable, as his physical condition prevents him from testifying. However, the victim’s statement represents a mere suspicion that the defendant tampered with the brakes. As well-founded as such a suspicion may be (given the history between the victim and the defendant), a statement based on mere suspicion rather than actual knowledge does not constitute a statement concerning the cause or circumstances of an “impending death” for purposes of the dying declarations exception. Thus, (A) is the correct answer and (C) is incorrect. (B) is incorrect because the declarant’s death is no longer required; unavailability is sufficient. Thus, if the victim’s statement otherwise qualified under the dying declarations exception, the fact that he is not dead would not render the motorist’s testimony inadmissible. (D) is incorrect for the reasons stated above and also because it incorrectly implies that the dying declarations hearsay exception applies only in civil cases. As noted above, the exception also applies to homicide cases. (Note that the traditional view, still followed by some states, would only allow the declaration in a homicide prosecution.)

94
Q

A plaintiff brought a malpractice action against a law firm that had represented him in a personal injury suit. The plaintiff alleges that the firm was derelict in failing to interview a doctor he suggested as a prospective expert witness. The firm’s pleadings contend that the doctor was never brought to the attention of anyone at the firm and was never considered as a witness. The plaintiff wants to introduce a “proposed witness list” from his case file at the firm. After the name of the doctor is the notation, “the plaintiff wants us to check this guy out before trial.” The notation is in the handwriting of a paralegal with the firm who is responsible for updating various case files as part of his regular duties. The paralegal did no direct work on the plaintiff’s case and he cannot remember which attorney in the firm asked him to make the notation. The defense objects to the introduction of the proposed witness list containing the notation. Is the proposed witness list and notation admissible? A Yes, as past recollection recorded. B Yes, as a record of a regularly conducted activity. C No, as hearsay not within any recognized exception. D No, as hearsay within hearsay, and one level is not within an exception.

A

The witness list should be admitted as a record of a regularly conducted activity (business record). A writing or record made as a memorandum or record of any act, transaction, occurrence, or event is admissible as proof of such act, transaction, occurrence, or event if it was made in the course of a regularly conducted business activity and if it was customary to make the type of entry involved (i.e., the entrant must have had a duty to make the entry). The business record must consist of matters within the personal knowledge of the entrant or within the personal knowledge of someone with a business duty to transmit such matters to the entrant. The entry must have been made at or near the time of the transaction. The list of proposed witnesses and the notation constitute a statement that the law firm was alerted to the existence of the doctor as a potential expert witness. The plaintiff wants to introduce these documents to prove the truth of this statement (i.e., that he alerted the firm to the existence of the doctor). Thus, the documents present a hearsay problem. Making a list of proposed witnesses would be part of the regular course of business for a law firm, and it would be part of the duties of the paralegal responsible for updating case files to enter the handwritten notation regarding the doctor at the direction of one of the firm’s attorneys. The matters contained in the list and notation would be within the personal knowledge of the attorney, who was under a business duty to report the information accurately to the paralegal, who was under a business duty to properly record the information. Thus, all the requirements for a business record are present, and the list and notation, made as records of the firm’s having been alerted to the doctor as a potential expert witness, are admissible as proof of that fact. (A) is incorrect because past recollection recorded comes into play when a witness’s memory cannot be refreshed by looking at something. At that point, there may be an attempt to introduce a memorandum or other record made by the witness or under his direction at or near the time of the event. The record is characterized as past recollection recorded. Here, there is no indication that a witness who has an insufficient memory is testifying, and the list of proposed witnesses and notation are not being offered as a record of anyone’s past recollection. Rather, the evidence is offered as a record of the firm’s being informed of the doctor as a potential expert witness. Therefore, the evidence will not be admitted as past recollection recorded. (C) is incorrect because, as explained above, the proffered evidence does come within a recognized hearsay exception. (D) is incorrect because the facts do not present any problem of “levels” of hearsay. The list and notation are considered to be an out-of-court statement that the firm was alerted to the doctor as a potential expert witness, and are being offered as proof of that fact. If the notation had simply repeated an assertion made by one outside of the business (e.g., “the doctor says that he will be available to testify on the date of the trial”) and been offered to prove the truth of the assertion (that the doctor was available as a witness), a hearsay within hearsay problem would exist. Because the statement within the notation would be hearsay not within any exception, the notation itself, despite the fact that it is a business record, would not be admissible to prove the doctor’s availability.

95
Q

A resident being interviewed live by a television reporter stated that, “The biggest problem in this city is corruption in city government, particularly the mayor.” The mayor has now brought an action for defamation against the resident. At trial, the mayor has produced testimony as to his honesty and good character. As part of his defense, the defendant seeks to offer into evidence the fact that the mayor was convicted two years ago of taking a bribe to award a city contract for solid waste disposal. Is the evidence admissible? A No, because character evidence is not admissible in civil cases. B No, because character can be proved only by opinion or reputation testimony. C Yes, because the mayor’s character is directly in issue. D Yes, because there was an actual conviction for the crime.

A

The evidence is admissible because the mayor’s character is directly in issue. The general rule is that evidence of character to prove the conduct of a person in the litigated event is not admissible in a civil case. However, when a person’s character itself is one of the issues in the case, character evidence is not only admissible, but in fact is the best method of proving the issue. Where the plaintiff brings a defamation action for injury to reputation and the defendant pleads as an affirmative defense that his statements were true, the plaintiff’s character is directly at issue in the case. Under the Federal Rules, any of the types of evidence (reputation, opinion, or specific acts) may be used to prove character when character is directly in issue. [Fed. R. Evid. 405(b)] Here, the mayor’s character is at issue and the resident is offering character evidence to show that his assertion that the mayor is corrupt is a true statement. Thus, (C) is correct and (A) is incorrect. (B) is incorrect because, as stated above, any of the types of evidence can be used to prove character when it is directly in issue. (D) is incorrect because an actual conviction is required for impeachment purposes, but not for the purpose of establishing character-evidence of an arrest or indictment would have been equally admissible.

96
Q

A defendant was on trial for burglary, and he took the stand in his own defense. On direct examination, the defendant vigorously denied having committed the burglary. Also on direct examination, the defendant stated that his last regular employment was as a bookkeeper for a corporation. On cross-examination, the prosecutor asked the defendant if he had embezzled funds from the corporation. The defendant denied that he had embezzled from the corporation or from anyone else. The prosecutor then wanted to call a police officer to the stand to testify that when she arrested the defendant for embezzlement, the defendant admitted to the officer that he had embezzled money from the corporation. Assuming that the defendant has not yet been tried on the embezzlement charges, may the prosecutor call the officer to the stand? A Yes, but only for purposes of impeachment. B Yes, both for impeachment of the defendant and as substantive evidence. C No, because the defendant has not yet been convicted of embezzlement. Correct D No, because the evidence would be extrinsic.

A

The officer may not testify about the embezzlement because it constitutes impeachment by extrinsic evidence of a specific instance of misconduct. A specific act of misconduct offered to attack the witness’s character for truthfulness can be elicited only on cross-examination. If the witness denies the act, the cross-examiner cannot refute the answer by calling other witnesses or producing other evidence. Because the alleged embezzlement is admissible, if at all, only as impeachment evidence, when the defendant denied it the prosecutor could not call the officer to testify. (A) is wrong because extrinsic evidence, such as the officer’s testimony, of an instance of misconduct is not admissible. (B) is wrong because when a person is charged with one crime, extrinsic evidence of other crimes or misconduct is inadmissible to establish criminal disposition. Because nothing in the facts indicates that such evidence is being offered to prove something other than disposition (e.g., motive, identity, common plan or scheme), the officer’s testimony is not admissible as substantive evidence. As discussed above, for impeachment, the prosecutor is limited to inquiry on cross-examination regarding the embezzlement. (C) is wrong because it implies that a witness may not be impeached with a prior act of misconduct unless it resulted in a conviction, and this is not the case. A witness may be impeached on cross-examination with a prior act of misconduct that is probative of truthfulness even if the misconduct did not result in a conviction. Here the problem is that the prosecutor is attempting to prove the bad act with extrinsic evidence, which is not allowed.

97
Q

A defendant is on trial for the murder of the victim, who was found beaten to death in his home. Evidence already presented has shown that the victim was killed when no one was at home except for the victim and his dog. The prosecution wishes to call a neighbor to the stand who is prepared to testify that she went to the victim’s home the day after his murder and that when the defendant came by, the dog ran to a corner, where he cringed and whimpered. The neighbor is also prepared to testify that the dog is normally a very friendly dog, usually greeting visitors to the house, including the defendant, by approaching them with his tail wagging. The defense objects to the neighbor’s proposed testimony. How should the court rule on the neighbor’s testimony regarding the dog’s behavior? Press Enter or Space to submit the answer A Admissible, because the dog could be brought into court for a demonstration of his reaction to the defendant. B Admissible, as circumstantial evidence against the defendant. C Inadmissible, because the dog may have been reacting as he did for reasons other than those implied by the neighbor’s testimony. D Inadmissible, because even though the testimony has probative value, such value is outweighed by its prejudicial nature.

A

The court should admit the neighbor’s testimony because it is relevant circumstantial evidence. The Federal Rules of Evidence define relevant evidence as evidence having any tendency to prove or disprove a fact that is of consequence to the action. [Fed. R. Evid. 401] Generally, all relevant evidence is admissible unless it is barred by a specific exclusionary rule or by the general balancing test of Rule 403, which permits exclusion of relevant evidence if its probative value is substantially outweighed by the danger of unfair prejudice, confusion of the issues, etc. The neighbor’s testimony is relevant because the dog’s behavior when the defendant came by tends to prove circumstantially (i.e., indirectly) the prosecution’s contention that the defendant beat the victim to death (in the dog’s presence). The neighbor is competent to testify as to the dog’s behavior toward the defendant both before and after the murder, and no other competency rule warrants excluding the testimony; hence, it should be admitted. (A) is incorrect because the availability of other evidence that might demonstrate the dog’s reaction more clearly does not preclude the neighbor’s testimony on that issue. As long as she is competent to testify regarding the dog’s behavior, the dog’s availability is irrelevant. (C) is incorrect because it is up to the trier of fact to evaluate the inference for which the circumstantial evidence is being offered. The defense may attack the neighbor’s testimony on cross-examination by suggesting other reasons for the dog’s reaction, but it cannot exclude the neighbor’s testimony on this basis. (D) is incorrect because the balancing test of Rule 403 provides only that a court may exclude relevant evidence if its probative value is substantially outweighed by the danger of unfair prejudice. While all evidence is prejudicial to the opposing party, “unfair” prejudice refers to suggesting a decision on an emotional or otherwise improper basis. There is nothing in the neighbor’s testimony to justify excluding it on unfair prejudice grounds.

98
Q

A well-known actor sued a resort hotel for damages to his new limited edition sports car caused by the hotel’s parking valet while the actor was a guest at the hotel. His lawsuit, based on theories of respondeat superior and negligent hiring, alleged that after he gave the valet the keys, the valet, who had been working for the hotel for nine months, took the car for a drive without permission and negligently drove it into a tree, causing extensive damage to the car. At trial, the actor’s counsel offers evidence that six months before the accident, but three months after the valet was hired, the hotel instituted new hiring procedures for all potential employees, including parking valets. Included in the new rules was a requirement that all persons must pass a thorough background check before being hired. The valet had been required only to have a valid driver’s license when he was hired. In fact, he had an extensive record of traffic offenses at the time he was hired. Is the evidence regarding the new employment requirements admissible? A No, because it is irrelevant. B No, because it is evidence of remedial measures. C Yes, because it is evidence of the hotel’s negligence. D Yes, because it is evidence that the valet was incompetent.

A

he evidence is admissible because it tends to show that the hotel was not acting prudently when it hired the valet, an employee who damaged a guest’s car; thus (C) is correct and (A) is incorrect. (B) is not a good answer because only subsequent remedial measures (i.e., those taken after the injury to the plaintiff occurred) may not be proven as evidence of negligence; here the change in hiring procedures took place before the car was damaged, and so would be allowed. (D) is not accurate-the evidence does not show that the valet was incompetent, but rather that the hotel did not investigate his competence when he was hired, an issue related to the actor’s negligent hiring claim.

99
Q

During the course of their marriage, a husband told his wife that he stole a famous painting from a federal museum. Six months after the admission, the couple divorced. Shortly after the divorce, the husband was killed in an automobile accident. Later, the wife read in the paper that a man had been charged with the theft of the painting her husband admitted to stealing and was about to be tried in federal district court. She told her friend that the man was probably innocent because the husband told her that he had stolen the painting himself. The friend told several other people what the wife had told her, and eventually the story got back to the defense attorney. The attorney now wants the wife to testify in court to the husband’s statement. Can the wife be compelled to testify? A Yes, but only because the husband is dead and cannot invoke his privilege. B Yes, because there is no privilege when the defendant is not a spouse. C No, because the couple was still married at the time of the disclosure. D No, because her testimony is not essential to prevent a fraud on the court.

A

The wife can be compelled to testify because her husband is dead and cannot invoke the privilege. There are two separate privileges related to marriage. There is spousal immunity, under which: (i) a married person whose spouse is the defendant in a criminal case may not be called as a witness by the prosecution, and (ii) a married person may not be compelled to testify against her spouse in any criminal proceeding. In federal court, this privilege belongs to the witness-spouse so that she may not be compelled to testify, but neither may she be foreclosed from testifying. This privilege terminates upon divorce. There is also a privilege for confidential marital communications, under which either spouse, whether or not a party, has a privilege to refuse to disclose, and to prevent another from disclosing, a confidential communication made between the spouses while they were married. Both spouses jointly hold this privilege. Divorce does not terminate this privilege retroactively. Because the communication must be made in reliance upon the intimacy of the marital relationship, if the communication is made in the known presence of a stranger, it is not privileged. Similarly, if one spouse voluntarily reveals the contents of the communication to a stranger, that spouse waives the protection of the privilege as to herself (i.e., she cannot use the privilege to refuse to disclose, or to prevent another from disclosing, the communication), but the other spouse (i.e., the one who did not reveal the communication) retains this privilege. Here, the spousal immunity between the husband and the wife terminated upon their divorce. Thus, the only consideration is the applicability of the privilege for confidential marital communications. The husband’s statement to the wife came during their marriage and was made in reliance upon the intimacy of their relationship (marital communications are presumed to be confidential). Thus, the statement was covered by the privilege for confidential marital communications. Their subsequent divorce did not terminate this privilege. However, when the wife revealed to her friend what her husband had told her concerning the theft of the painting, the wife lost her privilege to refuse to disclose the matter. If the husband were alive, he would retain the privilege despite the wife’s disclosure and could prevent her from testifying to his statement concerning the theft of the painting. Because the husband is dead, he cannot invoke his privilege. Because the wife has waived her privilege and the husband is unable to foreclose her testimony, she can be compelled to testify. (B) is incorrect because the privilege for confidential marital communications applies to the disclosure of matters communicated during and in reliance on the intimacy of the marital relationship regardless of whether one of the spouses is a defendant in a criminal case. (C) is incorrect because it does not take into account the fact that the wife waived her privilege when she communicated her husband’s admission to her friend, as explained above. (D) incorrectly concludes that the wife cannot be compelled to testify. Due to her knowing and voluntary revelation of the husband’s statement to her friend, the wife has waived her privilege and may be compelled to testify. Note that, if the privilege were still applicable (i.e., if the wife had not waived it), she could not be compelled to testify as to the contents of the privileged communication simply on the ground that such testimony would be `essential to prevent a fraud on the court.

100
Q

A defendant is on trial in federal court for the armed robbery of a casino. The defendant claims that he was out of town at the time of the robbery. The defendant calls an alibi witness to the stand to testify that she was with him on the trip. When asked where she was and who she was with on the date in question, the witness stated that she could not recall. She said she recalls spending a weekend at a bed and breakfast this spring, but she does not recall the date or her traveling companion. The defendant’s attorney then showed the witness a letter written by her on stationery from the bed and breakfast, and asks her to look at it and try to answer the question again. The prosecution objects. How should the court proceed? A Overrule the objection, because this is a past recollection recorded. B Overrule the objection, but the witness cannot depend on the terms of the letter when answering. C Sustain the objection, because the letter is hearsay. D Sustain the objection, because the letter has not been properly authenticated.

A

The prosecution’s objection should be overruled. If a witness’s memory is incomplete, the examiner may seek to refresh her memory by allowing her to refer to a writing or anything else-provided she then testifies from present recollection and does not rely on the writing. (A) is incorrect because the past recollection recorded doctrine applies when a party is seeking to read into evidence the contents of a writing that was made or adopted by the witness. Here, the defendant is not seeking to introduce the writing into evidence; he merely wants the witness to look at it. Furthermore, the requisite foundation to introduce a past recollection recorded has not been laid. Thus, (A) is incorrect. (C) and (D) are incorrect because the letter is not being offered into evidence. Hence, the letter is not hearsay and does not need to be authenticated.

101
Q

A driver was driving north on a local road when his car went out of control, crossed the center line, and struck the vehicle of another driver who was driving south on the same road. Immediately after the accident, an off-duty officer came by and photographed the accident scene for the police report. In a suit between the drivers, the plaintiff seeks to introduce the photograph taken by the officer. The officer is present in court but has not been called as a witness. Is the photograph of the scene of the accident admissible? A Yes, because the photograph was taken by a police officer who took the photo for an official report. B Yes, because the officer is available to testify at trial. C No, because a proper foundation has not been laid. D No, because of the best evidence rule.

A

The photograph is not admissible because a proper foundation has not been laid. To be admissible, a photograph must be identified by a witness as a portrayal of certain facts relevant to the issue, and verified by the witness as a correct representation of those facts. It is sufficient if the identifying witness is familiar with the scene or object that is depicted. Here, the photograph taken by the officer must be verified by a witness who is familiar with the accident scene as an accurate representation of that scene. Absent such verification and identification (i.e., a proper foundation), the photograph is not admissible. (A) is incorrect because a photograph’s admissibility does not require that the photographer be a police officer or that the photograph be taken for an official report. The identity of the photographer and the purpose for which the photograph was taken are irrelevant to the issue of admissibility of the photograph. (B) is incorrect because the photographer need not be available to testify at trial. To authenticate a photograph, any person familiar with the scene may authenticate the photograph. (D) is incorrect because the best evidence rule (also known as the original document rule) is inapplicable to these facts. The best evidence rule states that in proving the terms of a writing (including a photograph), where the terms are material, the original writing must be produced. Secondary evidence of the writing, such as oral testimony regarding the writing’s contents, is permitted only after it has been shown that the original is unavailable for some reason other than the serious misconduct of the proponent. [Fed. R. Evid. 1002] Here, the admissibility of the original photograph is in issue. A copy of the photograph is not being offered. Thus, no problem arises under the best evidence rule.

102
Q

A landowner’s will left his ranch to a rancher, his heirs, and assigns, so long as the property was used exclusively for ranch purposes, then to the landowner’s grandson. The remainder of the landowner’s property passed through the residuary clause of his will to the grandson. Seven years after the landowner’s death, the rancher began strip mining operations on the ranch. The grandson brought an action to quiet title to the ranch against the rancher, and the rancher counterclaimed on the same theory. Who should prevail? A The rancher, because the condition imposed on his interest under the will is void as violating the Rule Against Perpetuities. B The rancher, because the condition imposed is a restraint against alienation. C The grandson, pursuant to the residuary clause. D The grandson, because the condition imposed is valid and he takes according to the subsequent provision.

A

The grandson prevails because the ranch passed through the residuary clause. Under the Rule Against Perpetuities, the attempt to give the grandson an executory interest is void, so (D) is incorrect. However, the courts would simply read the conveyance without the language of the executory gift, leaving a possibility of reverter in the grantor, the landowner. Thus, (A) is incorrect. Because the grandson succeeded to the landowner’s interest as grantor via the residuary clause of the will, he will prevail. (B) is incorrect because there is no restraint on alienation contained in the will.

103
Q

An accountant employed by the Federal Communications Commission was offended by various jokes and cartoons that employees would post in the office cafeteria. The Commission did not have any rules regarding what employees could post in the cafeteria, and none of the cartoons were pornographic or harassing. Nevertheless, the accountant lodged a number of complaints with his supervisor that went unheeded. The accountant’s actions prompted a great deal of debate among employees and a great deal of displeasure on the part of the accountant’s supervisor. A labor contract between the agency and the clerical workers’ union contained a policy for providing for termination of union employees only for certain specified grounds, but the accountant was not a member of the union and was not covered by the policy or any other employment agreement. Which of the following statements is most accurate regarding the agency’s right to dismiss the accountant? A The accountant has a liberty interest in the exercise of his First Amendment rights that entitles him to a hearing to contest the grounds of his dismissal. B The accountant has a property interest as a public employee that precludes him from being fired without notice and an opportunity to respond. C The accountant has no right to a hearing because his statements were not an expression of views on public issues. D The accountant has both a liberty interest and a property interest that entitles him to a pretermination evidentiary hearing.

A

Answer : A = The accountant is entitled to a hearing because he has a liberty interest in the exercise of his First Amendment rights. If the accountant is fired, he has a right to a hearing to determine whether his First Amendment rights were violated by his dismissal. Under the Due Process Clause of the Fifth Amendment, a person has a liberty interest in the exercise of specific rights provided by the Constitution, including freedom of speech. A government employee may not be fired for expressing his views regarding public issues, but can be fired for speech that disrupts the employer’s policies or undermines the employer’s authority. Under the Court’s expansive interpretation of what a public issue is in this context [see Rankin v. McPherson (1987)], the accountant’s statement would probably qualify. At the very least, he can make enough of a showing that his termination violates his free speech rights to be entitled to a hearing on the issue under procedural due process principles. [See Givhan v. Western Line Consolidated School District (1979)] (B) is wrong because the accountant does not appear to have a property interest in his job. A public employee who is subject to removal only for “cause” has a property interest in his job and must be given notice of the charges against him that are to be the basis for his job termination, and a pretermination opportunity to respond to those charges. Here, however, the accountant did not have a property interest in his job. He could have been dismissed for no reason at all. He was not covered by the labor contract between the agency and its clerical workers, and there appears to be no other basis for him to claim an entitlement to continued employment. (C) is wrong because the accountant is entitled to a hearing as long as he can raise a prima facie claim that his speech, which was regarding an issue important to the perception of his agency, was on a public issue and therefore protected by the First Amendment. (D) is wrong for two reasons: As discussed above, the accountant does not have a property interest in his job. Also, due process does not necessarily entitle him to a pretermination evidentiary hearing; a post-termination evidentiary hearing is probably sufficient. [See Cleveland Board of Education v. Loudermill (1985)]

104
Q

A print cartridge company sent a letter to a business office offering to supply a free premium printer if the business office would agree to purchase all the print cartridges the office would need from the print cartridge company. This letter arrived in the hands of the business owner on the same day the office printer failed. The business office had experienced a slow month and the business owner was trying to decide whether to pay the office rent for the month or fix the printer. Based on the offer, the owner paid the rent. A week after she put the rent check in the mail, the owner received a second letter from the print cartridge company indicating that the printer program was being canceled due to a lack of printers. The next day, before she read the second letter, the owner mailed her acceptance letter to the print cartridge company. The print cartridge company refused to supply the business owner with a printer. If the business owner brings a breach of contract action against the print cartridge company, how should the court rule? A For the print cartridge company, because its offer to the business office was not sufficiently definite. B For the print cartridge company, because its revocation letter was received by the business owner before she dispatched the acceptance letter. C For the business owner, because she mailed her acceptance letter without being aware that the print cartridge company had revoked its offer. D For the business owner, because the owner used funds to pay the office rent in reliance on the print cartridge company’s offer.

A

The print cartridge company will prevail because the business owner could no longer accept the offer. A revocation generally is effective when received by the offeree. A written communication is considered to have been “received” when (i) it comes to a person’s attention, or (ii) it is delivered at a place of business through which the contract was made. The communication need not be read by the recipient to be effective. Hence, the business owner received the revocation when the revocation letter arrived. Her receipt of the revocation letter before she dispatched her acceptance letter effectively revoked the offer (even though the business owner was unaware of its contents when she mailed the acceptance). (A) is wrong because an offer to make a requirements contract (i.e., the buyer promises to buy from a certain seller all the goods it requires and the seller agrees to sell that amount to the buyer) is sufficiently definite because the quantity is capable of being made certain by reference to objective, extrinsic facts (i.e., the buyer’s actual requirements). (C) is wrong because, as stated above, the revocation was effective when it was received (i.e., when it came into the offeree’s possession), not when the offeree becomes aware of the revocation. (D) is wrong because the business owner’s response to the offer was not reasonably foreseeable. As an exception to the general rule that a revocation is effective on receipt, an offer cannot be revoked and will be treated as an option contract for a reasonable length of time where the offeror could reasonably expect that the offeree would rely to her detriment on the offer. However, this exception usually is applied in only two circumstances: an offer for a unilateral contract and a subcontractor’s bid to a general contractor. If the offeror is seeking a bilateral contract in a circumstance other than that of a subcontractor, it would be extremely rare for the offer to be irrevocable due to detrimental reliance. Generally, an offeree must accept the offer before relying on it. Here, it is not even clear that the business owner suffered a detriment (unless she could have skipped paying the rent that month). She was already obligated to pay rent. In any case, nothing suggests that the cartridge company reasonably should have foreseen that the business owner would rely on the offer the way she did (without having accepted it yet).

105
Q

A large wholesale dealer in produce had never done business with a certain greengrocer who operated a small chain of markets in the Midwest. They entered into a written agreement whereby the wholesale dealer agreed to supply to the greengrocer the “fuzzy” variety of peaches at $35 per 50-pound lot. The agreement contained a provision stating that the greengrocer will buy “as many 50-pound lots of fuzzy peaches as the greengrocer chooses to order.” Assuming that the greengrocer has not yet placed any orders for peaches with the wholesale dealer, is this agreement between the parties enforceable? A Yes, because it is a valid requirements contract and, as such, is enforceable under the Uniform Commercial Code. B Yes, because the Uniform Commercial Code will imply reasonable terms. C No, because the total quantity of the contract is not specified. D No, because there is no manifestation of commitment on the greengrocer’s part.

A

The agreement is not enforceable because the greengrocer’s promise is illusory. For a contract to be enforceable, consideration must exist on both sides, i.e., each party’s promise must create a binding obligation. If one party has become bound but the other has not, the agreement lacks mutuality because one of the promises is illusory. Here, the wholesale dealer has promised to supply the greengrocer with fuzzy peaches at a fixed price. The greengrocer, however, has not promised to order any peaches from the wholesale dealer. Even if the greengrocer decides to sell fuzzy peaches, it has not bound itself to order them from this particular wholesale dealer. The illusory nature of the greengrocer’s promise makes the agreement unenforceable on consideration grounds. (A) is incorrect because in a valid requirements contract, both parties’ promises create binding obligations: The promisor binds itself to buy from the supplier all that it requires, and the supplier binds itself to sell to the promisor that same amount. Consideration exists because the promisor is suffering a legal detriment; it has parted with the legal right to buy the goods it may need from another source. Under the UCC, which governs in this case because a contract for the sale of goods is involved, a good faith term is implied: The buyer’s requirements means such actual requirements as may occur in good faith. Thus, if the provision had stated instead that the greengrocer will buy “as many 50-pound lots of fuzzy peaches as the greengrocer shall require,” it would be a valid requirements contract under the UCC because it requires the greengrocer to buy fuzzy peaches only from the wholesale dealer and to act in good faith in setting its requirements. (B) is incorrect even though the UCC will imply reasonable terms under certain circumstances. Such terms as price and time for performance need not be spelled out in the contract; the terms will be supplied by a “reasonableness” standard if that is otherwise consistent with the parties’ intent. However, supplying reasonable terms will not change the express terms of the contract. The provision that the greengrocer will buy as many peaches as it chooses to order is not sufficiently obligatory to be saved by the court supplying reasonable terms. (C) is incorrect because if the agreement were otherwise a valid requirements contract, the absence of a total quantity term would not matter. As a general rule in sale of goods contracts, the quantity being offered must be certain or capable of being made certain. The UCC provides that an agreement to buy all of one’s requirements is sufficiently certain because requirements usually can be objectively determined. Furthermore, the quantity ultimately required in good faith must not be unreasonably disproportionate to any stated estimate or any normal requirements (in the absence of a stated estimate). Hence, if the greengrocer had contracted to buy all of its requirements from the wholesale dealer, the absence of a term specifying total quantity would not have made the agreement unenforceable.

106
Q

The owner of a house put the property up for sale. A surgeon entered into negotiations with the owner to purchase the house, and the parties agreed upon a sale price of $200,000 and a closing date. The owner told the surgeon that she would drop a contract in the mail and have her attorney draw up a deed. The owner signed a land sale contract, which included the property’s address but did not contain a metes and bounds legal description. She mailed the contract to the surgeon that afternoon, although it was mailed too late for the last mail pickup of the day. The owner’s attorney promptly drew up a deed and dropped it in the mail to his client. The surgeon received the contract the next day. After she mailed the contract, the owner received an offer of $250,000 for her property from her next-door neighbor, who wanted to expand beyond his own property line. The owner called her attorney and told him to inform the surgeon that the deal was off. The attorney e-mailed the surgeon, stating that his client had found another purchaser for the property, and that all matters regarding the surgeon’s offer for the property were rescinded. The surgeon signed the contract anyway and returned it to the homeowner by registered mail. If the surgeon brings an action to compel the owner to convey the property to him for $200,000, is he likely to prevail? A Yes, because the owner signed the land sale contract. B Yes, because the contract was effective when the owner placed the document in the mail. C No, because the surgeon did not mail the signed contract until after he received the revocation by e-mail. D No, because the land sale contract does not contain the complete legal description of the property. .

A

The surgeon is entitled to specific performance because the owner signed the land sale contract. A contract was formed here when the parties orally agreed to the sale of the property. However, the contract was unenforceable at that time because, under the Statute of Frauds, a contract for the sale of land is unenforceable unless a memorandum containing the contract’s essential terms is signed by the party to be charged. Here, the party to be charged is the owner, and she signed the land sale contract, a writing sufficient to satisfy the Statute of Frauds (a memorandum for the sale of land is sufficient if it contains the price, a description of the property-which need not be a “legal” description-and a designation of the parties). Thus, the contract was enforceable. Specific performance is allowed when the legal remedy (damages) would be inadequate, such as with contracts to purchase land, which is unique. Therefore, the surgeon is entitled to specific performance (assuming the property has not already been sold to a bona fide purchaser); under the facts the neighbor had made an offer but nothing indicates that the owner accepted the offer yet. (B) is incorrect because the contract was effective when the parties orally agreed to it. The contract became enforceable when the owner signed it, not when she placed it in the mail. (C) is incorrect because this is not a mailbox rule issue. The contract was already formed before the attorney attempted to revoke the offer. Thus, his attempt at revocation was ineffective. It did not matter when the surgeon mailed the contract. (D) is incorrect because, to satisfy the Statute of Frauds, a description need not be a complete legal description, but need merely be sufficient to reasonably identify the subject of the contract. It is sufficient that the property was identified by its address

107
Q

A sailing enthusiast went to a boat builder and told him that he wanted a yacht built to his specifications. They agreed that the price would be $400,000, and that the sailing enthusiast was to make payment in full within 30 days after he had accepted delivery of the yacht. They further agreed that the boat builder would not subcontract any of the work. The boat builder, however, contacted a master sail maker and subcontracted the sails for the yacht to him. They agreed orally that the boat builder would pay the sail maker $25,000 for the sails within 20 days of receiving them. The boat builder did not tell the sail maker of his agreement with the sailing enthusiast regarding subcontracting. The sail maker made the sails and delivered them to the boat builder, who then completed the yacht and delivered the boat to the sailing enthusiast. Although the yacht was built to his specifications, the sailing enthusiast refused to accept it after he learned that the boat builder had subcontracted for the sails. When the 20-day payment period for the sails had expired, the sail maker went to the boat builder and demanded the $25,000. The boat builder told the sail maker that he could not pay the $25,000 unless the sailing enthusiast paid him for the yacht. If the sail maker brings an action against the boat builder for breach of contract, is he likely to prevail? Press Enter or Space to submit the answer A Yes, because both parties are merchants. B Yes, because the sail maker fully performed. C No, because the boat builder had agreed not to subcontract. D No, because their agreement was oral.

A

Answer Discussion - Correct The contract between the sail maker and the boat builder is enforceable despite the Statute of Frauds because the sail maker fully performed. The contract here was for the sale of goods (sails) for the price of $500 or more; thus, the contract is within the Statute of Frauds. A contract within the Statute of Frauds is generally unenforceable absent written evidence of a contract signed by the party to be charged. However, there is an exception to the general rule for goods received and accepted. Here, although the contract was oral, the boat builder accepted the sails, and so he is bound despite the Statute. Note that the boat builder might also be bound under another exception to the Statute-for specially manufactured goods if the sails were made specially for the yacht and were not suitable for sale to others. (A) is incorrect because the UCC does not exempt merchants from the Statute of Frauds. Although it provides a special confirmatory memo rule by which a merchant may be bound even if his signature does not appear on the writing evidencing the contract, here there is no writing at all. (C) is incorrect because it is irrelevant. The fact that the boat builder agreed not to subcontract is relevant to whether he breached his contract with the sailing enthusiast, but it does not affect his contract with the sail maker. (D) is incorrect because, as stated above, the contract here falls within an exception to the Statute of Frauds because the boat builder accepted the sails.

108
Q

The owner of a condominium hired a cleaning and junk removal service to clean his condominium after he moved. The parties agreed in writing that the company was to completely empty out the condominium, wash the walls and floors, and clean the appliances in exchange for $1,500. Shortly after beginning performance, the company assigned to a creditor its right to all monies due under the contract (i.e., $1,500), and the creditor promptly notified the condominium owner of the assignment. The condominium owner acknowledged the assignment. The company continued working, completely emptying out the condominium, washing the walls and floors, and cleaning all of the appliances except for the oven before quitting the job. It would cost $150 to hire a substitute to clean the oven. The condominium owner refuses to pay the creditor anything because of the cleaning service’s breach. If the creditor sues the condominium owner, how much, if anything, is the creditor entitled to recover? A $1,500, the amount assigned, and the condominium owner may look to the company to recover for the minor breach. B The reasonable value of the labor and materials expended by the company on the portion of the job it did complete. C $1,350, on a theory of substantial performance. D Nothing, because the condominium owner’s duty to pay is subject to a constructive condition precedent, and the assignee takes subject to the defense that the condition has not been satisfied. Answer Discussion - Incorrect

A

The creditor will be able to recover the contract price less damages for the company’s minor breach. Generally, an assignee has whatever rights his assignor would have against the obligor. Similarly, the assignee is subject to any contract-related defenses that the obligor has against the assignor. Thus, the creditor will have whatever rights the company would have against the condominium owner. Here, the company completed all of the tasks that needed to be done except for one (i.e., cleaning the oven). If the work remaining on the contract is minor, the company will be seen as substantially performing its contract, and substantial performance will discharge its duty to perform and obligate payment by the condo owner. Because the facts state that the cost of finishing the job was relatively small (10% of the cost of the contract), it will probably be seen as a minor breach. Thus, the condominium owner cannot avoid payment of the contract price. However, despite the substantial performance, the other party to the contract may recover damages for the less than complete performance. Thus, the condominium owner will be able to offset his damages from the breach. The creditor then will be able to recover $1,350 (the contract price less the damages). (A) is incorrect because, as stated above, the obligor may offset damages directly against the assignee; he does not have to pay the full contract price and then seek damages from the assignor. (B) is incorrect because this suggests a restitutionary remedy, but, as stated above, the creditor, as assignee of the company, would be able to recover the contract price less damages because the company substantially performed. (D) is incorrect because the constructive condition precedent to the condominium owner’s duty to pay (the company’s performance) has been satisfied here by substantial performance.

109
Q

A building contractor entered into a contract with the local college to remodel a residence hall during the summer. As specified by the contract, the work had to be completed before the fall semester began at the beginning of September. Because the contractor received a great deal of other maintenance business from the college, his price of $400,000 was significantly lower than other contractors and he was not going to demand payment until the work was completed. By the end of the first week in August, the contractor had completed 75% of the project and had expended $350,000 in labor and materials. At that time, however, a labor dispute between the contractor and his employees prompted most of the workers to walk off the job. Because prospects for a quick settlement of the dispute were doubtful, the contractor informed the college that he would not be able to meet the completion deadline. A week later, the college obtained another contractor who was able to finish the project by the end of August. The college paid him $150,000, which included a substantial amount of overtime for his workers. The increase in value of the residence hall due to the remodeling was $425,000. The original contractor, who had not been paid, files suit against the college, which files a counterclaim against him. What should the contractor recover from the college? A Nothing, because the contractor breached the contract. B $200,000 in restitutionary damages, which is the difference between its expenditures and the amount the college paid the other contractor to complete the work. C $250,000 in restitutionary damages, which is the contract price minus the amount the college paid the other contractor to complete the work. D $275,000 in restitutionary damages, which is the difference between the value of the completed remodeling and the amount the college paid the other contractor to complete the work.

A

Answer Discussion - Incorrect The contractor should be able to recover $250,000 in restitutionary damages. Where a builder in a construction contract breaches during the construction, the nonbreaching party is entitled to the cost of completion plus compensation for any damages caused by the delay in completing the building. Most courts, however, will allow the builder to offset or recover for work performed to date to avoid the unjust enrichment of the owner. This restitutionary recovery is usually based on the benefit received by the unjustly enriched party. If substitute performance is readily obtainable, damages are measured by the unpaid contract price minus the cost of completion (up to the value of the benefit received by the defendant). Here, the contractor’s duty to complete the project was not discharged by impossibility; he could have hired another contractor to take his place or yielded to his employees’ demands. Hence, the contractor’s failure to complete the remodeling constituted a breach of contract and resulted in the college having to expend $150,000 to have the building completed on time. However, the contractor did not receive any payments for the work that he did before breaching; the college would be unjustly enriched if it does not have to pay for any of this work. The benefit of the completed remodeling is measured by the contract price, $400,000, because a restitutionary recovery here would be based on the failed contract between the parties and substitute performance is readily obtainable. This amount is reduced by the $150,000 cost of completion that the college can recover from the contractor, leaving a net recovery of $250,000 for him. (A) is incorrect. Most modern courts would permit the contractor to recover in restitution to prevent the college’s unjust enrichment from the work that he did. (B) is incorrect because recovery measured by the claimant’s detriment (i.e., his reliance interest) is an appropriate alternative only where the standard “benefit” measure would achieve an unfair result; it is not applied where the party seeking restitutionary recovery was the breaching party. (D) is incorrect because courts will always limit relief to the contract price where the claimant is the breaching party. Measuring the benefit to the college in terms of the value of the improvements rather than the contract price will deny to the college the benefit of the bargain that it became entitled to when the contractor breached.

110
Q

On March 1, a health food store owner and a health food distributor entered into a written agreement providing that the distributor would supply the store owner with his natural foods requirements over the next 12 months. In return, the store owner agreed to purchase only from the distributor. The agreement also provided that payment for any purchases made under the agreement during the month of April would be turned over to a nonprofit food research corporation, “to carry on its good works.” On April 1, the store owner ordered and the distributor shipped 20 cases of health food products, at a wholesale price of $4,000, which remains unpaid. On April 15, the store owner sold his store, inventory, and accounts receivable to a chain operation that also sells health food products. As part of the sale, the store owner assigned to the chain operation the contract with the distributor. The chain operation promptly notified the distributor of the sale and assignment. The $4,000 for the April 1 transaction remained unpaid and, on May 31, the food research corporation commenced suit to collect the money. Against whom may the action be maintained? Press Enter or Space to submit the answer A The store owner only. B Both the store owner and the chain operation. C The chain operation only. D Neither the store owner nor the chain operation

A

nswer Discussion - Incorrect The store owner (the promisor) remains liable based on his contract with the distributor (the promisee), and the chain operation is liable based on its assumption of the contract with the distributor. The food research corporation is expressly designated in the contract between the store owner and the distributor as a party to whom payment for any April purchases is to be directly made. Thus, the portion of the contract providing for payment is primarily for its benefit. Consequently, the food research corporation is an intended third-party beneficiary of the store owner’s promise to make the April payment. As the third-party beneficiary, it has a right of action against the promisor (the store owner) for enforcement of the promise to pay. When the store owner assigned to the chain operation his contract with the distributor, the chain operation was deemed to have assumed the store owner’s duties under that contract as well as being assigned the rights thereunder. Thus, the chain operation can be held to the duty to pay for the health food products that were ordered in April. Although the chain operation is deemed to have assumed the duties of the store owner under the contract, the store owner (the delegator) remains liable on the contract. Consequently, the food research corporation has a right of action for the April payment against both the store owner and the chain operation (although it will be limited to only one recovery, see below). (A) and (C) are incorrect because they conclude that, as between the store owner and the chain operation, only one of them has the duty to tender the required payment. As explained above, both the store owner and the chain operation are under this duty. (D) is incorrect because, as discussed above, both the store owner and the chain operation are liable. Of course, there will be only one recovery, so as not to cause a windfall as a result of the breach. Generally, the purpose of contract damages is to put the nonbreaching party in the position it would have been in had the contract been performed.

111
Q

A small computer company contracted with a community college to supply computer tech support for all of the college’s faculty and staff beginning September 1. Two weeks before the date that performance was to begin, the computer company received a more lucrative offer at a large university. In order to fulfill its contract obligations to the community college, the company delegated its duties under the contract to another computer company for $1,000 above the contract price. The community college agreed to allow the replacement computer company to perform the services, but on September 1, the replacement company refused to perform. If the community college sues the replacement company, what will the result be? A The replacement company will prevail, because the community college’s only remedy is against the computer company. B The replacement company will prevail, because $1,000 is grossly inadequate consideration. C The community college will prevail, because there has been a novation. D The community college will prevail, because it is an intended third-party beneficiary of the computer company-replacement company contract, and the replacement company assumed the duty of performance under the contract. Answer Discussion - Incorrect

A

The community college will prevail in its suit against the replacement company because there was a delegation of duties plus an assumption of duty on the part of the replacement company, supported by consideration, creating a third-party beneficiary situation in which the obligee can compel performance or bring suit for nonperformance. Because the replacement company has promised that it will perform the duty delegated and that promise is supported by consideration, an assumption of duty has occurred and a third-party beneficiary situation is created in which the community college can compel performance or bring suit for nonperformance. Thus, (D) is correct and (A) is incorrect. (B) is incorrect because a court would probably not even inquire into adequacy of consideration when the value of the consideration was satisfactory to the parties involved. (C) is incorrect because the facts do not clearly indicate a novation. A novation occurs where a new contract substitutes a new party to receive benefits and assume duties that had originally belonged to one of the original parties under the terms of the old contract. A novation discharges the old contract. A novation will be found when there is (i) a previous valid contract; (ii) an agreement among the parties, including the new party to the new contract; (iii) the immediate extinguishment of contractual duties as between the original contracting parties; and (iv) a valid and enforceable new contract. Here, the facts do not state or otherwise indicate that a new contract was created and the duties under the old contract were discharged. Thus, this is not a novation situation.

112
Q

A homeowner borrowed $50,000 from a bank, secured by a mortgage on his home. Shortly thereafter, the homeowner sold his home to a buyer for $70,000 by a deed containing a recital signed by both parties that title passed “subject to” the bank’s mortgage, “which obligation grantee expressly assumes.” The buyer paid the homeowner $20,000, took possession of the house, and began making monthly payments of principal and interest to the bank. A few years later, a chemical manufacturing firm built a huge sulfur processing plant just down the road from the home, which caused the house to immediately decline in value to $35,000. Subsequently, the buyer stopped making the monthly payments to the bank. The bank exercised its contractual right of nonjudicial foreclosure and sold the house at a public auction for $34,000. The bank then brought suit against the homeowner and the buyer for $14,000, the difference between the proceeds of the foreclosure sale and the $48,000 principal remaining due on the original loan to the homeowner. The jurisdiction does not bar deficiency judgments. Against whom should the bank be granted a judgment for $14,000? A Both the homeowner and the buyer. B Only the homeowner. C Only the buyer. D No one.

A

Both the homeowner and the buyer are liable for the deficiency. If a sale of foreclosed property does not bring enough to satisfy the mortgage debt, the mortgagee/lender can bring a personal action against the mortgagor/debtor for the deficiency (as long as the jurisdiction does not bar deficiency judgments). When the mortgagor sells the mortgaged property and gives a deed, the grantee takes subject to the mortgage, which remains on the land. If the grantee does not sign an agreement to assume the mortgage, he does not become personally liable on the loan, and the original mortgagor remains personally liable. If the grantee does sign an assumption agreement, however, the lender is considered a third-party beneficiary of the agreement, and hence may recover from the assuming grantee, who is primarily liable, or the original mortgagor, who is secondarily liable. Here, the buyer signed the recital providing for the assumption, so she will be personally liable on the loan. Therefore, (A) is correct, and (B) is incorrect. (C) is incorrect because the homeowner, the original mortgagor, did not extinguish his own personal liability on the loan by obtaining the assumption agreement from the buyer. He remains secondarily liable as a surety. Thus, the bank may sue the homeowner on the original mortgage agreement. (Note that while the bank may obtain a judgment against both of them, its maximum recovery will be the $14,000 deficiency.) (D) is incorrect because the facts indicate that the jurisdiction does not bar deficiency judgments

113
Q

A college student was the sole lifetime beneficiary under a large trust administered by a banker. The student received a large monthly distribution from the trust, and whenever he ran short, he simply called the banker for extra funds, because the trust provided that the student was to receive whatever he needed from income or principal. The student’s roommate found out about the trust arrangement and decided to see if he could make it pay off for him. The roommate sent an email to the banker, which appeared to be from the student, and which asked for several thousand dollars to cover medical expenses. The email further stated that, since he was in the hospital, the student would send his roommate to pick up the cash. The next day, the roommate showed up at the banker’s office and obtained the money on the promise that he would take it to the student. The roommate absconded with the funds. When the roommate obtained the cash from the banker, what crime did the roommate commit? A False pretenses. B Embezzlement. C Larceny by trick. D Larceny.

A

The roommate committed larceny by trick because the banker’s consent to the roommate’s taking the money was induced by the misrepresentation that the roommate would take the money to the student/beneficiary. Larceny consists of a taking and carrying away of tangible personal property of another by trespass, with intent to permanently (or for an unreasonable time) deprive the person of his interest in the property. If the person in possession of property has not consented to the taking of it by the defendant, the taking is trespassory. However, if the victim consents to the defendant’s taking possession of the property, but such consent has been induced by a misrepresentation, the consent is not valid. Under such circumstances, the larceny is called larceny by trick. Here, the roommate obtained the money from the banker on the promise that he would take it to the student/beneficiary. This misrepresentation induced the banker to give possession of the money to the roommate. The roommate then proceeded to take the money and carry it away, intending all the while to permanently deprive one who had a possessory interest superior to the roommate’s of his interest in the money. Thus, all the elements of larceny are present. Because the original wrongful taking resulted from consent induced by misrepresentation, the specific larceny committed by the roommate is more precisely characterized as larceny by trick. Consequently, although the roommate has in fact committed larceny, (C) is a better answer than (D). Regarding (A), false pretenses consists of obtaining title to the property of another by an intentional (or knowing) false statement of past or existing fact, with intent to defraud the other. If a victim intends to convey only possession of the property to the defendant, the offense is larceny by trick. However, if the victim intends to convey title, the offense is false pretenses. Here, the banker intended to convey possession of the money to the roommate so that he could give the money to the student/beneficiary. The banker did not intend to convey title to the roommate. Because the roommate did not obtain title by means of his misrepresentation but simply obtained possession, the offense of false pretenses was not committed. (B) is incorrect because embezzlement is the fraudulent conversion of property of another by a person in lawful possession of that property. In embezzlement, the misappropriation of the property occurs while the defendant has lawful possession of it. In larceny, the misappropriation occurs generally at the time the defendant obtains wrongful possession of the property. The roommate did not have lawful possession of the money because his possession of the money resulted from his misrepresentation to the banker. Thus, the roommate’s taking of the money was wrongful from the outset. Because the roommate had wrongful, rather than lawful, possession of the money, there was no embezzlement

114
Q

Acting with probable cause, the police arrested a man in connection with the armed robbery of a liquor store. After being given Miranda warnings, the man confessed to the robbery but denied his involvement with several other recent armed robberies of businesses in the area. He was formally charged with the one robbery and put into a cell with a paid informant working undercover for the police. The informant had been instructed to find out what he could about the other robberies but not to ask any questions. The informant began talking about a convenience store robbery in which a bystander was shot and seriously injured by the robber, and he deliberately misstated how it happened. The man, unaware that his cellmate was an informant, interrupted to correct him, bragging that he knew what really happened because he was there, and proceeded to make incriminating statements about the robbery. The man was subsequently charged with armed robbery and attempted murder in the convenience store robbery. At a motion-to-suppress hearing on that charge, if the man’s attorney moves to exclude the statements made to the informant, should the motion be granted? A Yes, because the informant deliberately elicited incriminating statements in violation of the man’s Sixth Amendment right to counsel. B Yes, because the informant’s conduct constituted custodial interrogation in violation of the man’s Fifth Amendment privilege against self-incrimination. C No, because the man had not yet been charged with the robbery of the convenience store when he made the statements to the informant. D No, because the informant’s conduct did not constitute interrogation.

A

The man’s motion should be denied because neither his Fifth nor Sixth Amendment rights were violated by the informant’s conduct. The Sixth Amendment right to counsel applies to all critical stages of a criminal prosecution after formal proceedings have begun, but does not apply in precharge custodial interrogations. Because this right is “offense specific,” the fact that the right to counsel has attached for one charge does not bar questioning without counsel for an unrelated charge. Because the man has not been charged with the convenience store robbery, his Sixth Amendment right to counsel has not been violated. The Fifth Amendment privilege against self-incrimination requires Miranda warnings and a valid waiver before any statement made by the accused during custodial interrogation can be admitted. However, this requirement does not apply where interrogation is by an informant who the defendant does not know is working for the police, because the coercive atmosphere of police-dominated interrogation is not present. [Illinois v. Perkins (1990)] Because the man was not aware of the informant’s status, the informant’s conduct did not constitute a police interrogation. (A) is wrong despite the fact that the informant’s conduct may have been deliberately designed to elicit incriminating remarks. As discussed above, the man’s right to counsel did not attach for purposes of the convenience store robbery. (B) is incorrect because, as discussed above, the Miranda warnings need not be given before questioning by a cellmate working covertly for the police. (D) is incorrect because interrogation refers not only to express questioning, but also to any words or actions on the part of the police that the police should know are reasonably likely to elicit an incriminating response from the suspect. Here, the informant, working for the police, made statements about the convenience store robbery that were intended to, and reasonably likely to, prompt a response from his cellmate. Hence, it is not the absence of “interrogation” that avoids the Miranda problem, but the fact that the man did not know that his cellmate was working for the police.

115
Q

A driver was operating her car on a city street when she was stopped by a police officer for speeding. As the police officer reached the driver’s car, he saw her put something into her purse. The officer told the driver, “Ma’am, you were speeding; that’s why I stopped you. I’d like your driver’s license, and, by the way, what did you just put into your purse?” The driver responded, “It’s just a marijuana cigarette, but don’t worry, I’ve only had two and my driving judgment hasn’t been impaired.” The officer took her purse, removed the “joint,” and charged the driver with possession of marijuana as well as speeding. At the driver’s trial for marijuana possession, the prosecution seeks to introduce the marijuana cigarette into evidence. The driver’s attorney moves to suppress the evidence. Should the defense motion be granted? A Yes, because the cigarette is fruit of the poisonous tree. B Yes, because the police officer did not have a valid search warrant. C No, because the police officer’s asking about the contents of the driver’s purse did not constitute custodial interrogation. D No, provided the police officer had a reasonable suspicion of criminal activity.

A

The defense motion should be denied because the driver was not in custody when she made the statement. Persons temporarily detained for routine traffic stops are not in custody for Miranda purposes. Therefore, the driver was not entitled to Miranda warnings, and her statement about the marijuana was not tainted. Her statement thus properly provided the probable cause for the search of her purse. (A) is therefore wrong. (B) is wrong because this case falls within the automobile exception to the warrant requirement. Under that exception, if the police have probable cause to believe that a vehicle contains contraband or fruits, instrumentalities, or evidence of a crime, they may search the vehicle, including the driver’s belongings, without a warrant. Here, the driver’s response established probable cause to search her purse. (D) states the test for a stop, not a search. An automobile search requires probable cause.

116
Q

The defendant was arrested, given Miranda warnings, and charged with burglary. At the police station, he telephoned his mother and asked her to come to the station to post bail. Instead, his mother immediately called the family attorney. In the meantime, the police had begun questioning the defendant. Although he never told the police to stop the questioning, his answers were at first vague or clearly unresponsive. During the course of the questioning, the family attorney phoned the station and told the police that she had been hired to represent the defendant and would be there in half an hour. The police did not inform the defendant of the attorney’s call. Ten minutes later, the defendant admitted to committing the burglary, and signed a statement to that effect prepared by the police. The attorney arrived a few minutes later and advised the defendant to remain silent, but he told her that he had already signed a confession. How should the court rule on the attorney’s pretrial motion to exclude the confession as evidence at trial? A Grant the motion, because the police had a duty to inform the defendant that an attorney was coming to represent him. B Grant the motion, because the defendant has been deprived of his Sixth Amendment right to counsel. C Deny the motion, because the defendant’s statement admitting the crime was voluntary. D Deny the motion, because the defendant waived his Miranda rights.

A

The defendant’s confession should be admitted because he waived his Fifth Amendment privilege against compelled self-incrimination after receiving Miranda warnings. Miranda v. Arizona requires that a person in custody be informed of his right to remain silent and his right to the presence of an attorney during questioning. A suspect may subsequently waive his rights by making a confession, as long as the waiver was knowing and voluntary. In this case, the defendant received proper Miranda warnings, and there is no indication that he did not understand what his rights were. Although his answers during questioning were initially unresponsive, he never asked for an attorney or indicated that he wished to remain silent, and he voluntarily confessed after a relatively short period of interrogation. Hence, he validly waived his Miranda rights. (A) is incorrect because the police have no duty to inform the defendant that an attorney is attempting to see him. The defendant’s ignorance of his attorney’s efforts has no bearing on whether he made a knowing waiver of his Miranda rights. (B) is incorrect because the defendant’s right to counsel was not violated. Although the defendant does have a separate Sixth Amendment right to counsel under Escobedo v. Illinois because he has already been arrested and charged with the crime, this right would only be violated if the defendant, after being informed of his right to counsel, had requested an attorney or had been prevented from seeing his attorney. Here, he made no request to see an attorney-even when he called his mother-and his attorney was allowed to see him immediately upon her arrival. Thus, he has waived his Sixth Amendment right to counsel. (C) is incorrect even though it is true that the defendant made a voluntary statement. Due process requires that for confessions to be admissible, they must be “voluntary,” based on the totality of the circumstances, and here all of the circumstances indicate that the defendant’s confession was voluntary. However, even a voluntary confession will be inadmissible if it was obtained in violation of Miranda rights. (D) is therefore a better choice than (C).

117
Q

A man and a woman were arrested and charged with conspiring to blow up a federal government building. After being given Miranda warnings, they were questioned separately and each of them gave a written confession. The confessions interlocked with each other, implicating both of the defendants as being involved in every stage of the conspiracy. Subsequently, the woman attempted to retract her confession, claiming that it was false. At a preliminary hearing, the judge rejected her claim. Both defendants were tried together, and the prosecutor introduced both confessions into evidence. At trial, the woman testified that she was not involved in any conspiracy and that her confession was fabricated. Both defendants were found guilty by the jury. The woman challenges her conviction on appeal because of the admission of the man’s confession. If the woman succeeds, what is the likely reason? Press Enter or Space to submit the answer A The man’s confession was more incriminatory to her than her own confession. B The jury was not instructed to consider the man’s confession as evidence only of his guilt and not of the woman’s. C The man refused to testify at trial and therefore was not subject to cross-examination regarding his confession. D The man testified at trial and was subject to cross-examination but denied making the confession attributed to him.

A

Answer Discussion - Correct If the woman prevails in her challenge to the admission of the man’s confession, it will be because the man could not be cross-examined regarding his confession. Under the Sixth Amendment, a defendant in a criminal prosecution has the right to confront adverse witnesses at trial. If two persons are tried together and one has given a confession that implicates the other, the right of confrontation generally prohibits the use of that statement because the other defendant cannot compel the confessing co-defendant to take the stand for cross-examination. A co-defendant’s confession is inadmissible even when it interlocks with the defendant’s own confession, which is admitted. If the man refused to take the stand and subject himself to cross-examination, his confession was not properly admitted because it violated the woman’s Confrontation Clause rights. (A) is incorrect because the fact that the man’s confession incriminates the woman more than her own confession is not relevant. Just the interlocking nature of the man’s confession with the woman’s confession makes it more damaging by making it harder for the woman to claim that her confession was false. (B) is incorrect because the Supreme Court has held that instructing the jury to consider the confession only as going to the guilt of the confessing defendant is inadequate to avoid Confrontation Clause problems, because the risk that the jury will not follow the limiting instructions is too great in this context. (D) is incorrect. Confessions of a co-defendant may be admitted if (i) all portions referring to the other defendant can be eliminated (so that there is no indication of that defendant’s involvement), (ii) the confessing defendant takes the stand and subjects himself to cross-examination regarding the truth or falsity of the statement, or (iii) the confession of the nontestifying co-defendant is being used to rebut the defendant’s claim that his confession was obtained coercively, and the jury is instructed as to that purpose. Even if the co-defendant denies ever having made the confession, as stated in choice (D), the opportunity at trial to cross-examine the co-defendant satisfies the Confrontation Clause

118
Q

A woman was arrested, given Miranda warnings, and questioned about an armed robbery. After she asked to speak with an attorney, the police stopped questioning her about the robbery. Several hours later, the police gave the woman a fresh set of Miranda warnings and began to question her about a different robbery. She did not repeat her request for an attorney and instead made several incriminating statements about the robbery. At the woman’s trial for the robbery for which she made incriminating statements, the prosecution seeks to have her statements introduced into evidence. If the woman’s attorney objects on appropriate grounds, how should the court rule? A Overrule the objection, because the police did not badger the woman into confessing. B Overrule the objection, because the woman did not renew her request for an attorney after receiving fresh Miranda warnings. C Sustain the objection, because the police did not honor the woman’s request. D Sustain the objection, because a confession obtained in violation of a defendant’s Miranda rights but otherwise voluntary may be used against the defendant.

A

The court should sustain the objection because the police did not honor the woman’s request for an attorney. At any time prior to or during a custodial interrogation, the accused may invoke a Miranda (Fifth Amendment) right to counsel. If the accused invokes this right, all questioning must cease until the accused is provided with an attorney or initiates further questioning himself. Thus, the police questioning of the woman about the robbery was improper, and she can have her statements excluded. (A) is incorrect. After receiving Miranda warnings, if an accused invokes the right to remain silent, the police cannot badger the accused. However, courts have ruled that if the police scrupulously honor the request, they can rewarn the accused and later resume questioning, at least about a different crime. Here, however, the accused did not simply invoke the right to remain silent, but rather requested an attorney. After such a request, as indicated above, all questioning must cease. (B) is incorrect because the accused does not need to reassert the right to an attorney; all questioning must stop until the accused is provided an attorney or resumes the questioning herself. (D) is incorrect. It is stating the rule for impeachment-a confession obtained in violation of a defendant’s Miranda rights but otherwise voluntary may be used against the defendant for purposes of impeachment, but there is no such rule for use of the confession for other purposes.

119
Q

A husband and wife were charged with stealing credit cards and charging expensive items on the misappropriated cards. An attorney was appointed by the court to represent the couple jointly. At the preliminary hearing, the judge found that the attorney would have no conflict representing both defendants in the joint trial. Halfway through the trial, however, a conflict arose between the defenses of the husband and wife. At the wife’s request, the attorney moved that another attorney be appointed to represent the wife and that a mistrial be declared. The trial judge moved favorably on the attorney’s motion. Another attorney was appointed to represent the wife, and as soon as the wife’s trial began, her attorney moved to dismiss the case on the ground that jeopardy had attached during the wife’s first trial and that she was being retried in violation of the United States Constitution. Should the judge grant the wife’s attorney’s motion? Press Enter or Space to submit the answer A Yes, because jeopardy attached when the jury began to hear evidence in the first trial. B Yes, because the judge incorrectly ruled that there would be no conflict of interest from the joint representation. C No, because the wife requested the mistrial. D No, because it is premature to move for a dismissal based on double jeopardy until the defendant is convicted.

A

Although jeopardy attached in the wife’s first trial, her retrial is not barred because she initiated the grant of the mistrial in her first trial. As a general rule, the right to be free of double jeopardy for the same offense bars a retrial for the same offense once jeopardy has attached in the first trial. However, one of the exceptions permitting retrial even if jeopardy has attached is when a mistrial is granted in the first trial at the request of the defendant on any ground not constituting an acquittal on the merits. Here, the wife requested the mistrial because a conflict arose between the defenses of her and her co-defendant in the joint trial, and the judge granted the mistrial solely to allow the wife to obtain another attorney. Thus, no acquittal on the merits occurred and the double jeopardy rule does not apply. (A) is incorrect for several reasons. Merely because jeopardy attaches does not mean that the double jeopardy rule will apply; retrial will be permitted under certain exceptions, one of which is applicable here. Furthermore, (A) is not a correct statement of law. Jeopardy attaches in a jury trial when the jury is impaneled and sworn in, even if it has not yet heard any evidence. (B) is incorrect because the judge’s finding at the preliminary hearing stage appears to be an honest error rather than bad faith conduct. In the absence of bad faith conduct by the judge or prosecutor designed to force the defendant to seek a mistrial, the defendant’s securing of a mistrial does not preclude a retrial. (D) is incorrect because the right to be free of double jeopardy creates a bar as soon as the defendant is retried for the same offense, rather than on her conviction.

120
Q

A grand jury was investigating a bank robbery. The only information known to the prosecutor was a rumor that a certain ex-convict might have been involved. The grand jury subpoenaed the ex-convict. He refused to answer questions about the robbery and was granted use and derivative use immunity. He then testified that he and a friend had robbed the bank. The grand jury indicted both the ex-convict and his friend for the bank robbery. The prosecutor permitted the friend to enter a plea to a lesser offense in exchange for the friend’s agreement to testify against the ex-convict. The prosecutor had no evidence as to the identity of the robbers except the testimony of the friend and the ex-convict. At the ex-convict’s trial, should his objection to his friend’s being permitted to testify be sustained? Press Enter or Space to submit the answer A Yes, because the prosecutor may not bargain away the rights of one co-defendant in a deal with another. B Yes, because the friend’s testimony was acquired as a result of the ex-convict’s grand jury testimony. C No, because the police suspected the ex-convict even before he testified in the grand jury hearing. D No, because a witness cannot be precluded from testifying if his testimony is given voluntarily.

A

Answer Discussion - Correct Testimony obtained by a promise of immunity is by definition coerced and therefore involuntary. Thus, immunized testimony may not be used for impeachment of the defendant’s testimony at trial. The friend’s testimony will not be permitted to be used against the ex-convict because it resulted from the ex-convict’s immunized testimony, and use and derivative use immunity prevents, as the term states, use of the immunized testimony and use of any evidence derived from the immunized testimony. (A) is wrong because it is an inaccurate statement of the law. Prosecutors can bargain away the rights of co-defendants. (C) is wrong because police suspicion is not the equivalent of actual testimony. (D) is wrong. Even though a witness wants to testify, various privileges such as lawyer-client, doctor-patient, etc., may bar the testimony. Here, the grant of immunity to the ex-convict is a bar to the friend’s derived testimony because use immunity bars use of one’s testimony or anything derived from it.QUESTION ID: MR014

121
Q

A restaurant owner in State A bought two large freezers from a manufacturer of commercial refrigeration equipment with its principal place of business in State B. Within one week and after being fully stocked with meat, one of the freezers broke down. The restaurant owner filed a state-based products liability action against the manufacturer in federal court in State A, and included a demand for a jury trial. Under the law in State A, jury verdicts do not need to be unanimous, but the Federal Rules of Civil Procedure require jury verdicts to be unanimous. At trial, the restaurant owner makes a motion asking the court to apply the State A law. How should the court rule on the motion? Press Enter or Space to submit the answer A Grant the motion, because applying the federal rule may change the outcome of the case. B Grant the motion, because, when a federal court has diversity jurisdiction, it is required to apply the substantive law of the state in which it is sitting. Correct C Deny the motion, because the Federal Rules of Civil Procedure apply in federal court as long as they are consistent with the Rules Enabling Act and not unconstitutional. D Deny the motion, because the Supreme Court’s balancing factors indicate that federal law should apply.

A

The court should deny the motion. Under the Erie doctrine, when a state law-based claim is brought in federal court based on diversity of citizenship, the federal court generally applies the substantive law of the state in which it is sitting. However, where a specific federal statute or the Federal Rules of Civil Procedure are on point, the federal court must apply federal procedural law as long as the federal rule is valid. Under the Rules Enabling Act, a Federal Rule is valid if it deals with “practice or procedure” and does not “abridge, enlarge, or modify” a substantive right. Here, there is a specific federal procedural rule that is on point [Fed. R. Civ. P. 48], which requires jury verdicts to be unanimous, unless the parties agree otherwise. Since there is no evidence of agreement, the federal procedural rule will apply, and the motion should be denied. (A) is wrong because it states the wrong conclusion, and it incorrectly cites the “outcome determination” test. This is the test that may be applied when there is no federal procedure law on point, and it is unclear whether the state matter is substantive or procedural. As stated above, there is a federal procedural rule on point. Therefore, this test does not apply. Similarly, (D) is wrong because it is referring to the balance of interests test, which is another test that may be applied when there is no federal procedural law on point, and it is unclear whether the state matter is substantive or procedural. This test is not applicable here, however, because there is a federal procedural rule on point. (B) is wrong because, although it is a true statement of law that a federal court with diversity jurisdiction is required to apply the substantive law of the state in which it is sitting, the requirements for a jury verdict is a procedural rule, not a substantive rule. Therefore, as set forth above, the federal procedural rule governs.

122
Q

A restaurant franchise properly sued the franchise owner of one of its restaurants in federal court for breach of contract for refusing to display materials for the new marketing campaign it launched. The court scheduled a pretrial conference for the purpose of “pressuring” the parties to settle the case. Both parties were given proper notice of the scheduled hearing, but only the owner of the restaurant and her attorney appeared at the conference. The judge immediately issued an order requiring the restaurant franchise to pay the restaurant owner travel expenses and attorneys’ fees for failing to attend. Is this a valid order of the court? A No, because the judge does not have the power to issue such an order. B No, because the court does not have the discretion to order a pretrial conference solely to pressure parties into settling a case. Correct C Yes, because the Federal Rules of Civil Procedure give the federal court discretion to order a pretrial conference for virtually any matter that is relevant, including settlement. D Yes, because a pretrial order binds all parties to the case for the remainder of the case.

A

This order by the court is valid. The Federal Rules give the court the power to call one or more pretrial conferences for a variety of reasons as necessary to expedite trial and foster settlement. Moreover, this conference is to be attended by at least one of the lawyers for each side who will actually be conducting the trial, and by any unrepresented parties. A party or counsel may be sanctioned for failure to attend a conference or obey an order entered pursuant to the conference. Additionally, the court must require the disobedient party or counsel to pay expenses incurred (including attorneys’ fees) by other parties unless the court finds that circumstances make such an award unjust. Here, the judge does have the power to not only order a pretrial conference to expedite the trial and encourage settlement, but also to sanction the restaurant franchise to pay the restaurant owner’s travel expenses and attorneys’ fees for failing to appear. (A) is wrong because it is a misstatement of the law. The judge does have the power under the rules to order the party who fails to appear to pay the opposing side’s attorneys’ fees. (B) is wrong because it also misstates the law. The Federal Rules permit courts to call pretrial conferences to expedite trial and foster settlement. (D) is wrong because, although it is a correct statement of the law (that pretrial orders bind all parties to the case for the remainder of the case), it does not directly answer the call of the question. The question asks if the order is valid. It does not ask if it is binding on the parties.

123
Q

A restaurant owner in State A bought two large freezers from a manufacturer of commercial refrigeration equipment with its principal place of business in State B. Within one week and after being fully stocked with meat, one of the freezers broke down. The restaurant owner filed a state-based products liability action against the manufacturer in federal court in State A, and included a demand for a jury trial. Under the law in State A, jury verdicts do not need to be unanimous, but the Federal Rules of Civil Procedure require jury verdicts to be unanimous. At trial, the restaurant owner makes a motion asking the court to apply the State A law. How should the court rule on the motion? A Grant the motion, because applying the federal rule may change the outcome of the case. B Grant the motion, because, when a federal court has diversity jurisdiction, it is required to apply the substantive law of the state in which it is sitting. C Deny the motion, because the Federal Rules of Civil Procedure apply in federal court as long as they are consistent with the Rules Enabling Act and not unconstitutional. D Deny the motion, because the Supreme Court’s balancing factors indicate that federal law should apply.

A

Answer Discussion - Incorrect The court should deny the motion. Under the Erie doctrine, when a state law-based claim is brought in federal court based on diversity of citizenship, the federal court generally applies the substantive law of the state in which it is sitting. However, where a specific federal statute or the Federal Rules of Civil Procedure are on point, the federal court must apply federal procedural law as long as the federal rule is valid. Under the Rules Enabling Act, a Federal Rule is valid if it deals with “practice or procedure” and does not “abridge, enlarge, or modify” a substantive right. Here, there is a specific federal procedural rule that is on point [Fed. R. Civ. P. 48], which requires jury verdicts to be unanimous, unless the parties agree otherwise. Since there is no evidence of agreement, the federal procedural rule will apply, and the motion should be denied. (A) is wrong because it states the wrong conclusion, and it incorrectly cites the “outcome determination” test. This is the test that may be applied when there is no federal procedure law on point, and it is unclear whether the state matter is substantive or procedural. As stated above, there is a federal procedural rule on point. Therefore, this test does not apply. Similarly, (D) is wrong because it is referring to the balance of interests test, which is another test that may be applied when there is no federal procedural law on point, and it is unclear whether the state matter is substantive or procedural. This test is not applicable here, however, because there is a federal procedural rule on point. (B) is wrong because, although it is a true statement of law that a federal court with diversity jurisdiction is required to apply the substantive law of the state in which it is sitting, the requirements for a jury verdict is a procedural rule, not a substantive rule. Therefore, as set forth above, the federal procedural rule governs.

124
Q

A car driver was hit by a pickup truck driver and suffered severe injuries. The car driver properly sued the truck driver in federal court for negligence, based on diversity jurisdiction, claiming the truck driver ran a red light. The truck driver defended on the grounds that the car driver was contributorily negligent for speeding, which under the applicable law would completely bar the car driver’s claim. The case went to trial, where all issues were vigorously litigated by the parties. The jury returned a verdict for the truck driver, specifically finding that the car driver was negligent for speeding. Thereafter, a passenger who was riding with the truck driver filed a diversity claim for negligence in federal court against the car driver for personal injuries from the same accident. The passenger wants to preclude the car driver from relitigating the question of whether the car driver was negligent. How will the court likely rule? A Not preclude the car driver from relitigating the question of whether she was negligent, because that finding was not essential to the judgment in the initial claim. B Not preclude the car driver from relitigating the question of whether she was negligent, because to do so would violate the car driver’s due process rights. C Preclude the car driver from relitigating the question of whether she was negligent due to issue preclusion. D Preclude the car driver from relitigating the question of whether she was negligent due to claim preclusion.

A

nswer Discussion - Incorrect The court will preclude the car driver from relitigating the question of whether she was negligent. Under issue preclusion (collateral estoppel), a judgment binds the plaintiff or defendant (or their privies) in subsequent actions on different causes of action between them (or their privies) as to issues actually litigated and essential to the judgment in the first action. Issue preclusion focuses on something relatively narrow-an issue that was litigated and determined in the first case, and that is relevant in a second case. With issue preclusion, the issue is deemed established in the second case without need to proffer evidence on it. Here, the car driver was a party to the first lawsuit and, after both parties “vigorously litigated” the issues of negligence, the jury determined that the car driver was negligent in the first case. Since the car driver filed a negligence claim against the truck driver claiming that he ran the red light in the first case, and the truck driver defended on the grounds that the car driver was contributorily negligent for speeding, the issue of who was negligent was essential to the judgment in the first action. Therefore, the court will preclude the car driver from relitigating whether she was negligent in the claim asserted against her by the passenger. For these same reasons, (A) is wrong. (B) is wrong because the car driver’s due process rights are not violated. Due process requires that issue preclusion may be asserted only against someone who was a party (or in privity with a party) to the previous case (the case in which the issue was actually litigated and determined). Here, due process is satisfied because the car driver was a party in the previous claim and vigorously litigated the claim. Note that the car driver may be able to assert the traditional “mutuality” rule to prevent the passenger from asserting issue preclusion, but this is not what was asserted in choice (B); due process was. Under the traditional “mutuality” rule, only someone who was a party (or in privity with a party) in the previous case can use issue preclusion. However, this requirement is not imposed by due process and has been subject to modification in certain circumstances to allow nonparties to take advantage of a prior judgment. In fact, although courts have been very reluctant to permit a nonparty to use issue preclusion to aid him offensively (as a plaintiff) to obtain relief, the Supreme Court has upheld its use offensively by a nonparty where it was fair and equitable to do so. [Parklane Hosiery Co. v. Shore, 439 U.S. 322 (1979)] (D) is wrong because it incorrectly states that the claim will be precluded by claim preclusion (res judicata). Claim preclusion focuses on something relatively large-the scope of a “cause of action.” If it applies, the result is usually to bar the same party in the first case from asserting a second case against the other party in the first case. Issue preclusion, in contrast, focuses on something relatively narrow- an issue that was litigated and determined in the first case and that is relevant in a second case.

125
Q

A pedestrian sued a driver for personal injuries in federal court, properly invoking diversity of citizenship jurisdiction. In the complaint, the pedestrian alleged that the driver ran a red light and struck the pedestrian while the pedestrian was in the crosswalk. Concurrent with the accident, a police report was prepared on which the name and address of a witness to the accident was listed, but neither party requested a copy of the report from the police department. Thus, when the pedestrian submitted an interrogatory to the driver for the names and addresses of persons with knowledge of the accident known to the driver, the driver truthfully omitted the name of the witness. When asked during his deposition whether he knew of any witnesses, the driver again truthfully answered “I don’t know of any.” At trial, the jury found for the driver. In one of the special interrogatories answered by the jury, the jury found that the driver had the green light and that the pedestrian was crossing against the light. Six months and a day after a final judgment was rendered in favor of the driver, the pedestrian’s attorney was contacted by the witness, who stated that the driver ran a red light and that the pedestrian had the “walk” sign when he attempted to cross the road. The pedestrian’s attorney immediately moved for relief from judgment based on newly discovered evidence, and the trial judge granted the motion. On appeal, what should the court do? A Affirm the trial judge’s decision, because whether to deny or grant a motion for relief from judgment is strictly a matter within the trial judge’s discretion. B Affirm the trial judge’s decision, because the driver should have discovered the identity of the witness. C Reverse the trial judge’s decision, because more than six months had passed since the judgment was rendered when the trial judge granted the motion. D Reverse the trial judge’s decision, because the pedestrian could have discovered the witness’s identity with reasonable diligence.

A

Answer Discussion - Correct The court should reverse the trial judge’s decision. On motion and just terms, the court may relieve a party from a final judgment or order on the following grounds: (i) mistake, inadvertence, surprise, or excusable neglect; (ii) newly discovered evidence that by due diligence could not have been discovered in time to move for a new trial; (iii) fraud, misrepresentation, or other misconduct of an adverse party; (iv) the judgment is void; (v) the judgment has been satisfied, released, or discharged; a prior judgment on which it is based has been reversed or otherwise vacated; or it is no longer equitable that the judgment should have prospective application; or (vi) any other reason justifying relief from the operation of the judgment. For grounds (i), (ii), and (iii), the motion must be made within a reasonable time not to exceed one year from the judgment; for the other grounds, the motion must be made within a reasonable time. (But remember that a lack of subject matter jurisdiction may be raised at any time.) Such a motion is left to the trial judge’s discretion, and, on appeal, her decision will be reviewed on an “abuse of discretion” standard. Here, the motion would be based on newly discovered evidence. When deciding whether to grant or deny such a motion, the trial judge must determine whether the evidence could have been discovered with any amount of due diligence. Here, the name and address of the witness could have been easily discovered had the pedestrian’s attorney simply requested a copy of the police report from the police department. This is such an obvious case of lack of due diligence on the part of a represented party that it would be an abuse of discretion to grant relief based on newly discovered evidence. (C) is incorrect because the time period for bringing a motion for relief from judgment based on newly discovered evidence is within a reasonable time, not to exceed one year from the judgment, not six months. (A) is incorrect because, although the decision is left to the trial judge’s discretion, the exercise of discretion would be reviewed on appeal on an abuse of discretion standard, and here, the newly discovered evidence could have easily been discovered in time for trial with minimal preparation. Thus, the judge very likely abused discretion by granting the motion. (B) is incorrect. Although a party is required to disclose information within his possession upon a proper request (and after a proper search of his records), and to truthfully answer questions at a deposition, those requirements do not relieve the opposing party from conducting an investigation of her own case. In other words, the fact that the driver’s attorney may have been grossly negligent in preparing his case does not relieve the failure of the pedestrian’s attorney to discover the identity of the witness

126
Q

The passenger in a bus was injured when the bus hit a concrete divider on the highway. The passenger sued the bus company in federal court for negligence. . The case went to trial, and the jury found that the bus company was in fact negligent, awarding the plaintiff $500,000. Judgment was entered on August 5. On August 14, the defendant filed a motion for a new trial. On August 22, the plaintiff filed to enforce the judgment. The court has not issued any orders since the final judgment. May the plaintiff enforce the judgment? A No, because judgments cannot be enforced while a post-trial motion is pending. B No, because judgments are not enforceable until 30 days after entry. C Yes, because judgments are enforceable as soon as they are entered. D Yes, because judgments are enforceable during the pendency of post-trial motions unless the court otherwise orders.

A

Answer Discussion - Incorrect Absent a court order, no execution on judgments is allowed for 30 days after entry except for injunctions or receiverships, which are not held up unless otherwise ordered by a court. (A) is incorrect because the fact that a post-trial motion has been filed is irrelevant to the automatic stay provision. Even if no motion were filed, the automatic stay provision would apply absent a court order. (C) and (D) are incorrect; the automatic stay provision would prevent a judgment from being enforced.

127
Q

A patient properly filed a medical malpractice claim against a doctor in federal district court. The complaint simply asserted negligence as the grounds for relief without any facts supporting the claim. The doctor filed a pretrial motion for a more definite statement, which the court denied. Immediately thereafter, and without submitting an answer, the doctor filed a motion to dismiss, asserting that the court lacked personal jurisdiction. Will the court grant this motion to dismiss? Press Enter or Space to submit the answer A Yes, because a defendant may file a motion to dismiss based on lack of personal jurisdiction at the time he files a motion or at the time he files his answer, whichever occurs later. B Yes, because the lack of personal jurisdiction defense can be raised at any time, even for the first time on appeal. C No, because a defendant must file a motion to dismiss based on lack of personal jurisdiction at the time he files a motion or at the time he files his answer, whichever occurs first. D No, because a lack of personal jurisdiction defense can only be raised in an answer or it is waived, and the defendant did not submit an answer.

A

Answer Discussion - Correct A defendant must file a motion to dismiss based on lack of personal jurisdiction at the time he files a motion or at the time he files his answer, whichever occurs first. If he does not, the defendant waives the defense. Here, the defendant first filed a pretrial motion for a more definite statement, which the court denied. Since he did not assert the lack of personal jurisdiction in this first motion, he waived the defense in the subsequent motion. (A) is wrong because it misstates the rule. This affirmative defense must be asserted in either a motion or an answer, whichever is first, not later, or it is waived. (B) is wrong because it also misstates the rule for personal jurisdiction. This would be the correct answer if the motion to dismiss were based on lack of subject matter jurisdiction, but this motion is based on lack of personal jurisdiction. (D) is wrong because it also misstates the law, as set forth above

128
Q

The driver of a car filed a civil action in federal district court against the owner and operator of a truck that collided with her vehicle. The driver has reason to believe that the truck was serviced at a particular auto repair shop shortly before the accident. The driver wants to see any records the repair shop has regarding the truck’s servicing because a failure to properly service the truck could help prove the truck owner’s liability. The driver requested the service records from the truck owner, but the truck owner has no service records in his possession. Can the driver obtain the truck’s service records from the repair shop? Press Enter or Space to submit the answer A Yes, by serving on the repair shop a request for production of the designated documents. B Yes, by serving on the repair shop a subpoena requiring that the shop produce the documents for inspection. C No, because, while the driver can depose the repair shop owner or mechanics, she cannot force the repair shop to permit inspection of the records unless the shop voluntarily agrees. D No, because the mechanics at the repair shop are experts, so discovery from the repair shop is limited.

A

Answer Discussion - Incorrect The driver can obtain the truck’s service records from the repair shop by serving a subpoena requiring the shop to produce the documents. A party may serve on a nonparty a subpoena that compels the nonparty to produce physical material, including documents and electronically stored information, relevant to the pending action. (A) is incorrect because a request for production is used to request documents from parties. Nonparties require subpoenas. (C) is incorrect because a party can force a nonparty to permit the inspection of records. (D) is incorrect because the mechanics are not experts who have been retained in anticipation of litigation or trial.

129
Q

While traveling on a commercial bus line, a passenger was injured when some luggage fell on him. As required by applicable state law, the bus company’s in-house attorney conducted an investigation and filed the required report with the state transportation department. The passenger subsequently filed a civil action against the bus company in federal district court, seeking compensatory damages for the injuries he suffered. During discovery, the passenger’s lawyer served on the bus company a request for production of documents, including a request for the report that the bus company filed with the state. The bus company objected to the request for the report and refused to produce it on the grounds that the report was privileged and protected from discovery under the work product doctrine. It did, however, produce other documents that were requested. The passenger then filed a motion to compel production of the report. If the court finds that the bus company’s claims of privilege and work product were not substantially justified, what orders must the court make relating to the passenger’s request for production of the report? Press Enter or Space to submit the answer A The court must sanction the bus company for improperly obstructing discovery by entering a default judgment against the bus company. B If the passenger’s attorney has attempted in good faith to resolve the dispute, the court must order the bus company to produce the report and to pay the passenger’s reasonable costs in making the motion. C Regardless of whether the passenger’s attorney has attempted in good faith to resolve the dispute, the court must order the bus company to produce the report and to pay the passenger’s reasonable costs in making the motion. D If the passenger’s attorney has attempted in good faith to resolve the dispute, the court must order the bus company to produce the report, but need not order the company to pay the passenger’s reasonable costs in making the motion.

A

Answer Discussion - Incorrect If the passenger’s attorney has attempted in good faith to resolve the dispute, the court must order the bus company to produce the report and to pay the passenger’s reasonable costs in making the motion, including attorney’s fees. Costs will be awarded unless: (i) the movant filed the motion before attempting in good faith to obtain disclosure or discovery without court action; (ii) the opposing party’s nondisclosure, response, or objection was substantially justified; or (iii) other circumstances exist that make an award of expenses unjustified. [Fed R. Civ. P. 37(a)] None of these exceptions apply. (A) is incorrect because default judgments are possible sanctions for a party who fails to comply with an order to provide discovery. (C) is incorrect because an attempt to resolve the discovery dispute without court intervention is required before an award of reasonable costs may be made. (D) is incorrect because reasonable costs must be awarded absent the exceptions noted above, which do not appear applicable here. [Fed. R. Civ. P. 37(a)(5)(A)]

130
Q

A plaintiff sued a defendant over a claimed debt. At the trial, the plaintiff established the existence of the debt and testified that he never received payment. In response, the defendant presents evidence sufficient to establish that she took her check to the post office and sent it to the plaintiff’s proper address by certified mail. The defendant offers a certified mail receipt with an illegible signature, which she claims is the plaintiff’s signature. The defendant also presents evidence that her basement flooded on March 28, and she claims that she cannot produce a canceled check because her box of canceled checks was destroyed from the water damage. Evidence is also presented that, due to a computer glitch, the defendant’s bank cannot reproduce her checking account records for the months of February and March. After the defendant’s testimony, which of the following is correct? Correct A The burden of persuasion and the burden of going forward with the evidence are on the plaintiff. B The burden of persuasion is on the plaintiff, but he has no burden of going forward with the evidence. C The plaintiff has satisfied his burden of persuasion, but he has a burden of going forward with the evidence. D The plaintiff has satisfied both his burden of persuasion and his burden of going forward with the evidence.

A

Highlight Answer Discussion - Correct The burden of persuasion and the burden of going forward with the evidence are on the plaintiff because the defendant’s testimony raises a rebuttable presumption that the check had been delivered in the mail. The burden of persuasion is the burden of a party to persuade the jury to decide an issue in its favor. If, after all the proof is in, the issue is equally balanced in the mind of the jury, then the party with the burden of persuasion must lose. The burden of persuasion does not shift from party to party during the course of a trial. Because the plaintiff sued the defendant for the debt, the plaintiff has the burden of persuasion when the time for the jury to make a decision arrives. The burden of going forward with the evidence is the burden of producing sufficient evidence to create a fact question of the issue involved. If a plaintiff makes out a prima facie case, he has met his burden of going forward with the evidence and the burden shifts to the defendant. Here, when the plaintiff made out a prima facie case of the defendant’s debt, the burden of going forward with the evidence shifted to the defendant. The defendant met this burden through the use of a presumption. Federal Rule 301 provides that a presumption imposes on the party against whom it was directed the burden of going forward with the evidence to rebut the presumption. The defendant’s evidence regarding the proper posting of the check raises a rebuttable presumption that the check was delivered to the plaintiff because a letter shown to have been properly addressed, stamped, and mailed is presumed to have been delivered in the due course of mail. Therefore, the burden of going forward with the evidence has shifted back again to the plaintiff, who must now produce evidence to rebut the presumption (i.e., evidence that he did not receive the check). (B) is incorrect because, as discussed above, the defendant’s testimony raised a rebuttable presumption that the check was delivered in the mail, which shifted the burden of going forward with the evidence to the plaintiff. The fact that the plaintiff met his burden of going forward with the evidence of the debt once, when he made out his prima facie case, does not mean the burden cannot shift back to him. (C) is incorrect because the plaintiff has not satisfied his burden of persuasion. As discussed above, the burden of persuasion does not shift from party to party and is only a crucial factor when all the evidence is in. This burden is satisfied when the jury finds a party has been more persuasive in arguing his side of the issue than the other party. Because the defendant’s testimony raises a rebuttable presumption that the check was delivered to the plaintiff, the plaintiff’s burden of persuasion cannot be met until he offers evidence to prove that the check was not received (a necessary element of his case). (D) is incorrect because, as discussed above, the defendant’s testimony raised a rebuttable presumption of delivery of the check in the mail to the plaintiff, which shifted the burden of going forward with evidence of nondelivery back to the plaintiff. The plaintiff’s burden of persuasion cannot be satisfied until he comes forward with this evidence because a necessary element of his case is that the defendant never paid him.

131
Q

An acquaintance asked the defendant to give him a lift downtown because he did not have bus fare. While riding on the defendant’s motorcycle, the acquaintance asked to stop at a convenience store to get a bottle of wine, showing the defendant a tire iron in his backpack that he was going to use. The defendant nodded in acknowledgment of what the acquaintance was planning to do. The defendant stopped at the store and waited in the parking lot while the acquaintance went in. He demanded money from the clerk, brandishing the tire iron. The clerk tried to grab a gun under the counter while he was filling a bag with money, and a struggle ensued. The gun discharged, killing the clerk. The defendant heard the gunshot and raced off, but was eventually apprehended. The jurisdiction’s criminal code provides that a death caused during the commission of certain felonies, including robbery, is first degree felony murder, for which the death penalty is permitted. The code also permits cumulative penalties for first degree felony murder and for the underlying felony. The defendant was charged and convicted of both robbery and felony murder. After appropriate consideration of all relevant circumstances, the jury imposed the death penalty. On appeal, the defendant challenged both the convictions and the sentence. Assuming that the above facts were properly admitted into evidence, how should the appellate court rule? A The defendant’s conviction for both offenses should be upheld, but imposition of the death penalty was not proper. B The defendant’s conviction for both offenses should be upheld, and imposition of the death penalty was proper. C The defendant’s conviction should be overturned under double jeopardy principles because robbery is a lesser included offense of felony murder. D The defendant’s conviction for felony murder should be overturned because the circumstances do not establish the necessary degree of culpability. Answer Discussion - Correct

A

The defendant can be found guilty of robbery and felony murder, but the death penalty cannot be imposed. The defendant can be found guilty of robbery as an accomplice. The Supreme Court has held that, under the Eighth Amendment, the death penalty may not be imposed for felony murder where the defendant, as an accomplice, did not take or attempt or intend to take life, or intend that lethal force be employed. [Enmund v. Florida (1982)] Here, because the defendant’s involvement in the crime was only to provide transportation, it cannot be said that he participated in such a major way that he acted with reckless indifference to human life; hence, the death penalty cannot constitutionally be imposed against him. (B) is therefore incorrect. (C) is incorrect because the defendant’s conviction of both robbery and felony murder does not raise double jeopardy problems under these facts. Under the rule that lesser included offenses “merge” into greater offenses, a person may not be convicted of both the greater offense and a lesser included offense. While the Supreme Court has held that a subsequent prosecution for robbery is not permitted against a defendant who has been tried for felony murder where the robbery is the underlying felony, this situation is different. Imposition of cumulative punishments for two statutorily defined offenses arising from the same transaction and constituting the same crime does not violate double jeopardy when the punishments are imposed at a single trial, as long as the two offenses were specifically intended by the legislature to carry separate punishments. [Missouri v. Hunter (1983)] Here, the legislature did specifically provide for cumulative penalties for first degree felony murder and for the underlying felony. Thus, the defendant can be convicted of both robbery and felony murder. (D) is incorrect because the jury could properly find the defendant guilty of felony murder. When the felony murder rule is combined with accomplice liability rules, the scope of liability becomes very broad. The felony murder rule provides that a killing-even an accidental one-committed during the course of a felony is murder. All parties to the felony are liable for the murder as long as (i) it was committed during the commission of the felony or in fleeing from the scene, and (ii) it was a foreseeable result of commission of the felony. Courts have been willing to find most deaths committed during a felony to be foreseeable. Here, the jury could reasonably find the shooting death of a store clerk by the acquaintance during a struggle for a gun to be a foreseeable result of the commission of a robbery and impose felony murder liability on the defendant as an accomplice because he knew that the acquaintance was going to commit a robbery.

132
Q

A buyer for a chain of shoe stores ordered 1,000 pairs of shoes from a shoe manufacturer. The shoes cost $50 per pair, so the total contract price was $50,000. It happened that the manufacturer owed $50,000 to a trucking company. The manufacturer assigned, in writing, “all proceeds from the contract with the buyer” to the trucking company. The manufacturer notified the buyer that he had assigned the proceeds of the contract to the trucking company and then shipped the 1,000 pairs of shoes to the buyer. Upon receipt of the shoes, the buyer discovered that 10% of the shoes were defective. He sent a check for 90% of the contract price ($45,000) to the manufacturer, who deposited the check. Shortly thereafter, the manufacturer closed down its business and disappeared without a trace. The trucking company, meanwhile, demanded payment from the buyer, to no avail. If the trucking company sues the buyer for the $45,000 that the buyer paid on the contract, will the trucking company prevail? Press Enter or Space to submit the answer A Yes, because the buyer had notice from the manufacturer that the contract had been assigned to the trucking company. B No, because the manufacturer wrongfully took the money that was assigned to the trucking company and is solely liable to the trucking company. C No, because the buyer fulfilled his obligations under the contract by paying the manufacturer. D No, because the trucking company could not have performed the other side of the contract by furnishing the shoes.

A

Answer Discussion - Incorrect The trucking company will be able to recover the $45,000 from the buyer because the buyer had notice of the assignment. Most contract rights may be assigned, and the right assigned here (to receive money) falls within the general rule. Once the assignment is effective, the assignee (the trucking company) becomes the real party in interest, and he alone is entitled to performance under the contract. (The assignor has been replaced by the assignee.) Once the obligor (the buyer) has knowledge of the assignment, he is bound to render performance to the assignee. Here, the assignment was effective as soon as the assignor (the manufacturer) manifested his intent that the right should be assigned (i.e., in his written assignment to the trucking company). The buyer was given notice of the assignment and, thus, was bound to pay the trucking company. The buyer breached his duty by paying the manufacturer instead of the trucking company. Thus, the trucking company may recover from the buyer for his failure to perform. (B) is wrong because the assignee, as the real party in interest, may enforce its rights against the obligor directly. (C) is wrong because, as stated above, once the buyer had notice of the assignment, he owed the duty to pay to the trucking company (the assignee), and payment to any third party, even the manufacturer (the assignor), does not discharge this duty. (D) is wrong because it is irrelevant whether the assignee could perform under the contract; the relevant question is whether the assignor could and did properly perform.

133
Q

A landowner conveyed his land “to my son for life, then to my son’s widow for her life, then to my son’s children.” At the time of the conveyance, the son was 20 years old and unmarried. The son eventually married and had two children, the landowner’s grandson and granddaughter. Many years later, the landowner and the grandson were involved in a train accident. The landowner was killed instantly. The grandson died a short time later of his injuries. The landowner left his entire estate by will to his friend. The grandson’s will devised his entire estate to the city zoo. The son’s wife was so grief-stricken that she became ill and died the next year, leaving her entire estate to her husband. Eventually the son met and married a 21-year-old. Ten years later, the son died, leaving everything to his second wife. When the second wife moved onto the land, the granddaughter filed suit to quiet title to the land, joining all of the appropriate parties. If the jurisdiction recognizes the common law Rule Against Perpetuities, unmodified by statute, in whom will the court most likely find that title to the land is held? A One-half in the granddaughter and one-half in the city zoo, subject to the second wife’s life estate. B One-half in the granddaughter and one-half in the second wife, because the second wife took the son’s interest. C Entirely in the friend, subject to the second wife’s life estate, because the gift to the son’s children violates the Rule Against Perpetuities. D Entirely in the granddaughter, subject to the second wife’s life estate, because the grandson did not survive the son.

A

Highlight Answer Discussion - Incorrect The granddaughter and the city zoo each own one-half of the land, subject to the second wife’s life estate. At the time of the conveyance by the landowner, the son had a life estate, the son’s widow had a contingent interest (because the son’s “widow” cannot be ascertained until the son’s death), and the son’s children had a contingent remainder (because they have not yet been born). When the grandson and the granddaughter were born, however, their interests became vested subject to open (i.e., if the son had more children). Thus, when the grandson died, he had a vested remainder subject to open that he was free to devise by will; the city zoo took his vested remainder subject to open. At the son’s death, the class of his “children” closed (because the son could not have any more children), and the granddaughter’s and the zoo’s vested remainders subject to open became indefeasibly vested. Also at the son’s death, his widow was ascertained and her interest vested in possession. Because the second wife was the son’s widow, she is entitled to the valid life estate. Thus, the granddaughter and the city zoo hold one-half interests, subject to the second wife’s life estate. (B) is wrong because the son had no interest in the land when he died. He merely had a life estate, which ended at his death. He did not inherit any interest in the property from anyone else. The only person he inherited from in these facts was his first wife, and she had no interest in the land. Furthermore, this choice overlooks the city zoo’s interest, which was inherited from the grandson. (C) is wrong because the son’s children’s interest does not violate the Rule Against Perpetuities. To be valid under the Rule, an interest must vest if at all within a life in being at its creation plus 21 years. The son is a life in being. At the son’s death, his children’s interest is certain to vest or fail: If the son had any children, at his death, the children’s interest would become indefeasibly vested (i.e., the class would close and the children’s interest would no longer be subject to open). Note that the children need not come into possession within the perpetuities period; the only requirement is that their interests vest within the period. Likewise, if the son had no children, the gift to them was certain to fail at his death. Thus, the children’s interest does not violate the Rule. Because the son had children and their interest was valid, there was no interest to revert to the landowner and to be devised to the friend. Note that the unborn widow aspect of this question is a red herring. The fact would be relevant only if the children’s gift were conditioned on their surviving the widow, in which case the takers would remain unascertained and their interest would remain contingent until that time. But because the children’s interest vested at the son’s death, it is irrelevant that the son’s “widow” was not a life in being at the creation of the interest. (D) is wrong because the gift to the son’s children was not conditioned on their survival of the son. The law does not imply such a condition. The grandson’s interest was vested subject to open and could be disposed of by his will.

134
Q

An owner and a builder executed a contract providing that the builder was to construct a residence on a specified lot according to plans and specifications. The total contract price was $800,000. No date was included in the contract for completion of the home. After the builder completed 60% of the residence, a flash flood from a nearby river partially eroded the lot but left the construction undamaged. The builder determined that it would cost an additional $1.7 million to repair the lot so that the residence can be constructed according to the plans. Without the additional lot repair work, the residence cannot be constructed at all. Which of the following states the probable legal consequences of the lot erosion? A The builder may avoid the contract if the increased costs of construction would bankrupt him. B The contract is void because of mutual mistake. C The builder is discharged from his duties under the contract because of impracticability. D The builder remains obligated to perform under the contract, but he may bring an action against the owner for the increased costs of construction.

A

Highlight Answer Discussion - Incorrect The builder will be discharged from his duties under the contract. Modern courts recognize that impracticability due to excessive and unreasonable difficulty or expense is a defense to breach of contract for nonperformance. Since the cost to the builder to perform under the original contract would exceed more than double what he would be paid, he likely would be excused from performance by commercial impracticability. (D) is therefore incorrect. Unlike destruction of the building itself before completion, which will not discharge a contractor’s duty, the erosion of the lot, which destroys the means of performing the contract, will generally not be one of the risks that a builder will be deemed to have assumed. (A) is wrong because the builder’s bankruptcy is irrelevant for this purpose. (B) is wrong because there is no mutual mistake here.

135
Q

wo neighbors owned summer homes adjacent to each other on the lake. After a week-long stay by the son of one of the property owners, the neighbor called the owner and said that his boat dock had been badly damaged and was told by another resident that the owner’s son and some friends had gotten drunk and accidentally crashed their boat into his dock. The owner was surprised at the accusation because he was sure that if his son had caused the damages, he would have told him. However, he did not want to get into a dispute with his neighbor, so he told his neighbor that he would have the dock repaired and pay for the repairs if the neighbor agreed not to bring a claim against his son for the damage to the dock. The neighbor agreed, and the owner hired a local carpenter to do the work. Later, however, the owner discovered that his son did not damage the dock because the damages occurred after his son had returned to college. Is the owner obligated to pay for the repairs? A No, because the owner never really believed that his son caused the damage. B No, because his son in fact did not cause the damage. C No, because the neighbor was wrong when he accused his son of causing the damage and it would be unfair to enforce an agreement when there was a mutual mistake of fact. Correct D Yes.

A

Answer Discussion - Correct Modern courts would hold that a promise to forbear suit on a claim that the promisor honestly and reasonably believes to be valid is good consideration to support an agreement, even if the claim ultimately turns out not to be valid. Hence, (A) and (B) are wrong. (C) is wrong because mutual mistake is not a defense when the adversely affected party bore the risk that the parties’ assumption was mistaken. To be a defense, it must be a true mistake, not merely an uncertainty. Here, the owner always had the right to investigate the truth of the facts before he agreed to pay for the dock.

136
Q

A three-year-old girl attending nursery school punched a boy in the face because he was teasing her about wearing glasses. The blow knocked out the boy’s newly acquired front teeth. If the boy’s parents sue the girl’s parents for the injury, what is their best defense? A The boy was the initial aggressor. B The girl is too young to be responsible for her actions. C A parent cannot be liable for damages due to the child’s conduct. D The parents were unaware of any potentially violent behavior by the girl.

A

The best defense of the girl’s parents is that they were unaware of any potentially violent behavior by the girl. At common law, parents are not vicariously liable for the torts of their child. (Statutes in most states allow for limited liability for intentional torts, but there is no indication of such a statute here.) Parents can be liable, however, for their own negligence, i.e., in not exercising due care under the circumstances. Thus, if the parents know their child may be violent, they could be negligent if they do not take precautions to prevent that behavior or injury from that behavior. However, if the parents have no reason to know their child could be violent, they have no duty to protect against such behavior. Here, if the girl had never done anything like this before, and her parents had no idea that she would be violent, they were not negligent in allowing her to attend nursery school. (A) is wrong because, although the boy’s teasing may have provoked the girl, he did not initiate the violence. He did nothing to allow the girl a right of self-defense, and so his actions would not provide the girl’s parents with a good defense. (B) is wrong because there is no general tort immunity for children. As long as the child is old enough to intend the act, she can be held liable. Here it seems that the girl intended to cause a battery. She either intended or knew with substantial certainty that swinging her fist would strike the boy in the face, i.e., would cause a harmful or offensive contact. Thus, this choice does not present the best defense for the defendants. (C) is wrong because parents can be liable for damages due to their child’s conduct. As explained above, although the parents are not vicariously liable at common law, they can be liable based on their own negligence (e.g., for negligent supervision).

137
Q

While the defendant was committing a robbery, he shot and killed the victim. The defendant is charged with first degree murder in a state that defines first degree murder as murders committed with premeditation or deliberation or during the commission of burglary, arson, rape, or robbery, and defines second degree murder as all other murders. The state also defines voluntary manslaughter as the unlawful killing of a human being with malice upon a sudden quarrel or heat of passion, and it defines involuntary manslaughter as the unlawful killing of a human being without malice in the commission of an unlawful act, not amounting to an enumerated felony, or in the commission of a lawful act that might produce death in an unlawful manner or without due caution and circumspection. Assuming evidence to support, what explanation for the shooting would best help the defendant in avoiding conviction for first degree murder? A In an act of resistance, the victim suddenly attacked the defendant and knocked him down, so the defendant pulled the trigger because he was afraid the victim was going to hit him again. B The defendant had the gun for many years, it was old and rusty, and he did not think it would fire. C The defendant had taken “angel dust” before the incident and does not remember getting a gun or holding up the victim. D When the defendant tried to hold up the victim, the victim said, “Get out of here, you dirty bum, or I’ll kill you,” and the defendant became so upset that he did not know what he was doing.

A

Because the defendant was charged with first degree murder, the theory of the case is most likely felony murder, and (C) is the only choice that sets out a theory to avoid a felony murder conviction. If the defendant was so intoxicated that he could not form the intent to steal, then he is not guilty of robbery, and there would be no “felony” from which the felony murder rule is to arise. (A) and (B) are incorrect because even though the defendant could argue that no premeditation or deliberation was present, he would still be guilty of first degree murder under the felony murder rule because the felony was robbery. (D) is wrong because insulting someone is not adequate provocation that would mitigate a homicide to voluntary manslaughter; neither would this “threat” suffice, in all likelihood. At most, the circumstances might produce the sort of unreasonable anger that would negate premeditation and deliberation. However, because the defendant clearly caused the homicide while committing the felony of robbery, his crime remains first degree murder.

138
Q

Acting with probable cause, the police arrested a man in connection with the armed robbery of a liquor store. After being given Miranda warnings, the man confessed to the robbery but denied his involvement with several other recent armed robberies of businesses in the area. He was formally charged with the one robbery and put into a cell with a paid informant working undercover for the police. The informant had been instructed to find out what he could about the other robberies but not to ask any questions. The informant began talking about a convenience store robbery in which a bystander was shot and seriously injured by the robber, and he deliberately misstated how it happened. The man, unaware that his cellmate was an informant, interrupted to correct him, bragging that he knew what really happened because he was there, and proceeded to make incriminating statements about the robbery. The man was subsequently charged with armed robbery and attempted murder in the convenience store robbery. At a motion-to-suppress hearing on that charge, if the man’s attorney moves to exclude the statements made to the informant, should the motion be granted? A Yes, because the informant deliberately elicited incriminating statements in violation of the man’s Sixth Amendment right to counsel. B Yes, because the informant’s conduct constituted custodial interrogation in violation of the man’s Fifth Amendment privilege against self-incrimination. C No, because the man had not yet been charged with the robbery of the convenience store when he made the statements to the informant. D No, because the informant’s conduct did not constitute interrogation.

A

Highlight Answer Discussion - Incorrect The man’s motion should be denied because neither his Fifth nor Sixth Amendment rights were violated by the informant’s conduct. The Sixth Amendment right to counsel applies to all critical stages of a criminal prosecution after formal proceedings have begun, but does not apply in precharge custodial interrogations. Because this right is “offense specific,” the fact that the right to counsel has attached for one charge does not bar questioning without counsel for an unrelated charge. Because the man has not been charged with the convenience store robbery, his Sixth Amendment right to counsel has not been violated. The Fifth Amendment privilege against self-incrimination requires Miranda warnings and a valid waiver before any statement made by the accused during custodial interrogation can be admitted. However, this requirement does not apply where interrogation is by an informant who the defendant does not know is working for the police, because the coercive atmosphere of police-dominated interrogation is not present. [Illinois v. Perkins (1990)] Because the man was not aware of the informant’s status, the informant’s conduct did not constitute a police interrogation. (A) is wrong despite the fact that the informant’s conduct may have been deliberately designed to elicit incriminating remarks. As discussed above, the man’s right to counsel did not attach for purposes of the convenience store robbery. (B) is incorrect because, as discussed above, the Miranda warnings need not be given before questioning by a cellmate working covertly for the police. (D) is incorrect because interrogation refers not only to express questioning, but also to any words or actions on the part of the police that the police should know are reasonably likely to elicit an incriminating response from the suspect. Here, the informant, working for the police, made statements about the convenience store robbery that were intended to, and reasonably likely to, prompt a response from his cellmate. Hence, it is not the absence of “interrogation” that avoids the Miranda problem, but the fact that the man did not know that his cellmate was working for the police.

139
Q

A trainer of homing pigeons brought several of them to a park that he often used for training. He had trained this group of pigeons carefully and was confident that they would readily find their way home. When they were released, one of the pigeons inexplicably turned in the opposite direction from home. Several blocks away at the other end of the park, it collided with a radio-controlled model airplane that its owner had just purchased and was trying out for the first time. The collision sent the airplane out of control; it dipped low across a highway and was struck and run over by a truck. The airplane owner sued the pigeon trainer for the destruction of his airplane. The parties stipulated to the above facts and the airplane owner presented evidence of his damages. The trainer then moved for a directed verdict. Should it be granted? A No, because the trainer’s pigeon caused the destruction of the airplane. B No, because the jury could find negligence on the trainer’s part under the doctrine of res ipsa loquitur. C Yes, because the truck, rather than the pigeon, was the direct cause of the airplane’s destruction. D Yes, because the trainer took reasonable care in training his pigeons

A

Answer Discussion - D The court should grant a directed verdict for the trainer because the airplane owner has not shown that the trainer breached any duty that he owed to him. A prima facie case of negligence requires plaintiff to show the following elements: (i) the existence of a duty on the part of the defendant to conform to a specific standard of conduct for the protection of the plaintiff against unreasonable risk of injury, (ii) breach of that duty by the defendant, (iii) that the breach of duty was the actual and proximate cause of plaintiff’s injury, and (iv) damage to plaintiff’s person or property. Here, it is doubtful that the trainer’s releasing his pigeons created any duty to other users of the park. To the extent that it did, the fact that he had taken great care to train them to return directly to their roosts indicates that he did not breach his duty to the airplane owner. Because the airplane owner has offered no other evidence of negligence, nor any reason to impose strict liability on the trainer (as discussed below), the trainer’s motion for a directed verdict should be granted. (A) is incorrect because that choice suggests the imposition of a strict liability standard on the trainer. The owner of a domestic or inherently nondangerous animal is not strictly liable for the injuries it causes. The conduct of the trainer’s homing pigeon would not make the trainer liable in the absence of some negligence on his part. And, as discussed above, there is no evidence of negligence here. While this choice establishes the causation element, the breach of duty element is not established. (B) is incorrect because the doctrine of res ipsa loquitur applies only to situations where the fact that a particular injury occurred itself establishes that defendant breached a duty. If the doctrine is applicable, no directed verdict may be given for defendant because plaintiff has established a prima facie case. However, the accident must be the type that would not normally occur unless someone was negligent. The collision between the trainer’s homing pigeon and the model airplane is not that type of accident; by itself, it provides no suggestion that anyone was negligent. (C) is incorrect because the truck is not a superseding force that breaks “the causal connection” between the action of the trainer’s pigeon and the airplane’s destruction. In indirect cause cases, where a force came into motion after defendant’s act and combined with it to cause injury to plaintiff, defendant will still be potentially liable for foreseeable intervening forces that are within the increased risk caused by his acts. Even if the intervening force is independent (i.e., not a natural response or reaction to the situation), it will be foreseeable where defendant’s negligence increased the risk that the independent force would cause harm. Hence, if the trainer were negligent in releasing his pigeon, the fact that the destruction of the airplane was directly caused by the truck would not relieve the trainer from liability, because the initial collision with the pigeon caused the airplane to go out of control and created a substantial risk that it would be damaged by an intervening force. QUESTION ID: MT137

140
Q

In an accountant’s trial for filing fraudulent tax reports, the prosecution calls a former colleague of the accountant, and she testifies that the accountant’s reputation in the community is for frequently participating in very questionable reporting, often resulting in unnecessary risk for his clients. She testifies further that she thinks the accountant is dishonest. Should the trial court admit this evidence over the accountant’s objection? Press Enter or Space to submit the answer Correct A No, because the prosecution cannot initiate evidence of the defendant’s character. B No, because use of the colleague’s opinion is improper. C Yes, because it is evidence of the defendant’s character for dishonesty. D Yes, because it is evidence of habit.

A

Answer Discussion - Correct The court should not admit this evidence because the prosecution cannot initiate evidence of the defendant’s bad character. The prosecution may offer such evidence only after the accused has put his character in issue by either taking the stand (thus placing his credibility in issue) or offering evidence of his good character. Thus, (C) is incorrect. (B) is incorrect because, under the Federal Rules, character may be proven by opinion evidence. (D) is incorrect because this does not constitute a regular response to a specific set of circumstances; it is merely reputation and opinion evidence.

141
Q

A town with a population of 30,000 merged with a city of 60,000. To protect voting rights of the citizens of the former town, a proposal was made that for a period of 20 years, beginning at the date of the merger, the city council of the merged city would consist of six persons. Each formerly separate municipality would be divided into three council districts. Each district from the former town would have approximately 10,000 residents, and each district from the former city would have 20,000 residents. A mayor would be elected at large. Before this proposal was placed on the ballot, the state attorney general issued an advisory opinion stating that the proposal was not in violation of any state statutory or constitutional provision. The proposal was placed on the ballot and was carried by large majorities in both the town and the city, and the districts were carved out. Three taxpayers filed suit to enjoin the holding of an election with council districts of such disparate proportions. The suit reached the state supreme court, which ruled that the governmental formula was constitutional under both the state and United States Constitutions. The plaintiffs wish to take the case to the United States Supreme Court. How should the Supreme Court proceed? Press Enter or Space to submit the answer A Rely on the attorney general’s opinion and not hear the case on its merits. Incorrect B Not hear the case, because it was decided below on an independent state ground. C Not hear the case, but remand it to federal district court. Correct D Hear the federal issues involved, but decline to rule on state issues.

A

Answer Discussion - Incorrect The Supreme Court may grant certiorari to review a case from the highest court in a state that can render an opinion on the matter if a state statute’s validity is called into question under the federal Constitution. [28 U.S.C. §1257] The Court may decide the federal issues, but cannot rule on the state law issues. (A) is incorrect for several reasons: (i) the attorney general evaluated only the proposal’s validity under the state constitution; and (ii) even if her opinion had addressed the proposal’s federal constitutional validity, the Supreme Court is not bound by advisory opinions of state attorneys general. (B) is incorrect even though the state supreme court may have had an independent state ground for finding the law constitutional under its state constitution. The Supreme Court will refuse to hear the case only if the state ground is adequate by itself to support the decision as well as independent, so that the Court’s review of the federal ground for the decision would have no effect on the outcome of the case (such as if the state court had found the law invalid under both the state and federal Constitutions). Here, the Supreme Court’s review of the state court opinion on the law’s federal constitutional status may have an outcome on the case regardless of the state court’s decision on the state constitutional issue; the Court therefore will hear the federal issues involved. (C) is incorrect because 28 U.S.C. section 1257 provides that appellate review of a matter from a state’s highest court is to the Supreme Court by petition for a writ of certiorari, rather than to a federal district court.

142
Q

A law student was sued by a student loan provider in federal court for failing to pay back her student loans. At the close of a federal civil trial, the court held in favor of the student loan provider for the full amount plus post-verdict interest at 8%. When the court clerk typed up the final order to send to all parties, she accidentally typed in 9% as the post-verdict interest rate. The law student did not notice the error until 14 months after the judgment was entered. She immediately filed a motion for relief from judgment based on a clerical mistake to correct the order to reflect the accurate interest rate. Will the court likely grant this motion? Incorrect A No, because motions for relief from judgment to correct clerical mistakes must be made within a reasonable time, not to exceed one year. B No, because motions for relief from judgment to correct clerical mistakes must be made within 28 days of the order. C Yes, because motions for relief from judgment to correct clerical mistakes must be made within a reasonable time. Correct D Yes, because motions for relief from judgment to correct clerical mistakes can be made at any time.

A

Highlight Answer Discussion - Incorrect The court will likely grant the motion. There is no time limit to file a motion for relief from judgment to correct clerical mistakes, and the court order correcting the error dates back to the time judgment was entered. [Fed. R. Civ. P. 60(a)] A clerical error is one arising from oversight or omission and may occur in judgments, orders, or other parts of the record. Since there is no time limit, the only issue is whether the error constitutes a “clerical mistake.” Here, the clerk clearly made a clerical error in recording the incorrect interest rate at 9% instead of the 8% interest rate actually ordered by the court. Therefore, the motion will likely be granted. (A) is wrong because it is an incorrect statement of law. There are several different grounds for obtaining relief from judgment, with different time limits. Other grounds besides correcting clerical mistakes include: (i) mistake, inadvertence, surprise, or excusable neglect; (ii) newly discovered evidence that by due diligence could not have been discovered in time to move for a new trial; (iii) fraud, misrepresentation, or other misconduct of an adverse party; (iv) the judgment is void; (v) the judgment has been satisfied, released, or discharged; a prior judgment on which it is based has been reversed or otherwise vacated; or it is no longer equitable that the judgment should have prospective application; or (vi) any other reason justifying relief from the operation of the judgment. For grounds (i), (ii), and (iii), the motion must be made within a reasonable time not to exceed one year; for the other grounds, the motion must be made within a reasonable time. [Fed. R. Civ. P. 60(b)] (B) and (C) are wrong because they incorrectly state the time frame for a motion to correct clerical mistakes, which can be filed at any time. Choice (B) states the time frame to file a motion to reconsider a final order under Rule 59(e). Choice (C), as stated above, states the time frame for a motion for relief from judgment on the grounds that the judgment is void, which are not the grounds asserted in the question. QUESTION ID: MJ262

143
Q

A law student was sued by a student loan provider in federal court for failing to pay back her student loans. At the close of a federal civil trial, the court held in favor of the student loan provider for the full amount plus post-verdict interest at 8%. When the court clerk typed up the final order to send to all parties, she accidentally typed in 9% as the post-verdict interest rate. The law student did not notice the error until 14 months after the judgment was entered. She immediately filed a motion for relief from judgment based on a clerical mistake to correct the order to reflect the accurate interest rate. Will the court likely grant this motion? Incorrect A No, because motions for relief from judgment to correct clerical mistakes must be made within a reasonable time, not to exceed one year. B No, because motions for relief from judgment to correct clerical mistakes must be made within 28 days of the order. C Yes, because motions for relief from judgment to correct clerical mistakes must be made within a reasonable time. Correct D Yes, because motions for relief from judgment to correct clerical mistakes can be made at any time.

A

Highlight Answer Discussion - Incorrect The court will likely grant the motion. There is no time limit to file a motion for relief from judgment to correct clerical mistakes, and the court order correcting the error dates back to the time judgment was entered. [Fed. R. Civ. P. 60(a)] A clerical error is one arising from oversight or omission and may occur in judgments, orders, or other parts of the record. Since there is no time limit, the only issue is whether the error constitutes a “clerical mistake.” Here, the clerk clearly made a clerical error in recording the incorrect interest rate at 9% instead of the 8% interest rate actually ordered by the court. Therefore, the motion will likely be granted. (A) is wrong because it is an incorrect statement of law. There are several different grounds for obtaining relief from judgment, with different time limits. Other grounds besides correcting clerical mistakes include: (i) mistake, inadvertence, surprise, or excusable neglect; (ii) newly discovered evidence that by due diligence could not have been discovered in time to move for a new trial; (iii) fraud, misrepresentation, or other misconduct of an adverse party; (iv) the judgment is void; (v) the judgment has been satisfied, released, or discharged; a prior judgment on which it is based has been reversed or otherwise vacated; or it is no longer equitable that the judgment should have prospective application; or (vi) any other reason justifying relief from the operation of the judgment. For grounds (i), (ii), and (iii), the motion must be made within a reasonable time not to exceed one year; for the other grounds, the motion must be made within a reasonable time. [Fed. R. Civ. P. 60(b)] (B) and (C) are wrong because they incorrectly state the time frame for a motion to correct clerical mistakes, which can be filed at any time. Choice (B) states the time frame to file a motion to reconsider a final order under Rule 59(e). Choice (C), as stated above, states the time frame for a motion for relief from judgment on the grounds that the judgment is void, which are not the grounds asserted in the question. QUESTION ID: MJ262

144
Q

A landowner owned a large parcel of land that he divided into two equal parcels. Thirty years ago, the landowner deeded the eastern parcel to a purchaser by warranty deed, including an easement over the south 25 feet of the western parcel for access to the navigable river that ran along the westerly boundary of the western parcel. The landowner acknowledged the deed and easement, and the purchaser recorded the document. The recording officer maintains an alphabetical grantor-grantee index, but no tract index. The purchaser made no use of the easement until five years ago, one year after her neighbor had purchased the western parcel from the landowner. The neighbor had paid at least market value for the western parcel and was not aware of the purchaser’s easement. The neighbor objected to the purchaser’s use of the easement shortly after she began using it, but the purchaser paid no attention. The neighbor sues the purchaser to quiet his title and to restrain the purchaser from using the easement over the western parcel. The purchaser has reasonable access to a public highway on the easterly boundary of the eastern parcel. If the purchaser is successful, what is the likely reason? A The absence of a tract index requires that the neighbor make inquiry regarding the riparian rights of owners abutting his property. B The neighbor and the purchaser trace their title to a common grantor, the landowner, whose covenants for title run with the land and estop the neighbor from denying the purchaser’s title. Incorrect C An easement is a legal and incorporeal interest that is not just attached to an estate in the land, but runs with the land itself and therefore binds successive owners of the servient estate regardless of notice. Correct D The easement is a legal interest in the neighbor’s chain of title even though there is no tract index.

A

If the purchaser prevails, it will be because the easement is a legal interest in the neighbor’s chain of title. This is a recording act problem. Even though the neighbor had no actual or inquiry notice, the recorded easement by his grantor, the landowner, would give him constructive notice of the purchaser’s interest in the western parcel, regardless of the absence of a tract index. (A) is a misstatement of the law. The absence of a tract index does not require a purchaser to inquire about the riparian rights of abutting landowners. (B) is incorrect because covenants for title are those contained in a general warranty deed. Although the future covenants for quiet enjoyment, warranty, and further assurances run to successive grantees, the present covenants of seisin, right to convey, and against encumbrances are breached, if at all, at the time of conveyance. (C) is incorrect because a bona fide purchaser of the servient parcel with no notice of the easement takes free of the easement.

145
Q

After being notified by a doctor that her employment with his office was terminated, a nurse applied for a position with a hospital. In her application, the nurse listed her former employment with the doctor, with the understanding that the doctor might be contacted. The doctor, in response to a telephone inquiry from the hospital, stated that the nurse “lacked professional competence.” Although the doctor reasonably believed that to be a fair assessment of the nurse, his adverse rating was based on an episode of malpractice for which he blamed the nurse but which in fact was chargeable to another doctor. Because of the doctor’s adverse comment on her qualifications, the nurse was not employed by the hospital. If the nurse asserts a claim based on defamation against the doctor, will the nurse prevail? A Yes, because the doctor was mistaken in the facts on which he based his opinion of the nurse’s competence. Incorrect B Yes, because the doctor’s statement reflected adversely on the nurse’s professional competence. C No, because the nurse authorized the hospital to make inquiry of her former employer. Correct D No, because the doctor had reasonable grounds for his belief that the nurse was not competent.

A

Highlight Answer Discussion - Incorrect (D) is correct. The nurse will not prevail because the doctor had reasonable grounds for his statement. As a former employer responding to queries of a prospective employer about a job applicant, the doctor has a qualified privilege. Such a privilege is not absolute; it exists only if exercised in a reasonable manner and for a proper purpose. The privilege may be lost if the speaker made a statement not within the scope of the privilege or if the speaker acted with actual malice (i.e., knowledge that the statement was untrue or with reckless disregard as to its truth or falsity). Because the doctor had reasonable grounds for his belief, he was not acting with actual malice. (A) is incorrect because of the reasons stated in the analysis above. A statement of opinion may be actionable if it appears to be based on specific facts which, if expressly stated, would be defamatory. However, because of the qualified privilege, the doctor will not be liable for his mistake as long as his belief was reasonable. (B) is incorrect because the fact that the statement was in a category that is slander per se (i.e., adversely reflecting on the nurse’s abilities to practice her profession) goes to whether the nurse must plead special damages. It does not, however, undermine the qualified privilege. (C) is incorrect because permission to make inquiry is not tantamount to consent to be defamed.

146
Q

An attorney was employed by the United States Department of Health and Human Services in a regional office located in a tobacco-growing state. A labor contract between the agency and the clerical workers union contained a policy providing for termination of union employees only for certain specified grounds. The attorney, however, was not a member of the union and not covered by such a policy. The attorney was angered by the regional director’s refusal to adopt a no-smoking policy for employees and visitors in the office. She posted a notice in the employee cafeteria ridiculing what she called the hypocrisy of an agency promoting health issues and nonsmoking programs while refusing to provide its employees with those same opportunities. The notice prompted a great deal of debate among the employees and was brought to the attention of the regional director, who was very displeased. Which of the following statements is most accurate regarding the director’s right to dismiss the attorney? A The attorney has a liberty interest in the exercise of her First Amendment rights that entitles her to a hearing to contest the grounds of her dismissal. B The attorney has a property interest as a public employee that precludes her from being fired without notice and an opportunity to respond. C The attorney has no right to a hearing because her statements were not an expression of views on public issues. D The attorney has both a liberty interest and a property interest that entitles her to a pre-termination evidentiary hearing.

A

Highlight Answer Discussion - Incorrect If the attorney is fired, she has a right to a hearing to determine whether her First Amendment rights were violated by her dismissal. Under the Due Process Clause of the Fifth Amendment, a person has a liberty interest in the exercise of specific rights provided by the Constitution, including freedom of speech. If a government employer seeks to fire an employee for speech-related conduct when the speech involved a matter of public concern but is not made pursuant to her official duties, the courts must carefully balance the employee’s rights as a citizen to comment on a matter of public concern against the government’s interest as an employer in the efficient performance of public service. Under the Court’s expansive interpretation of what is a public issue in this context [see Rankin v. McPherson (1987)], the attorney’s statement would probably qualify. At the very least, she can make a sufficient showing that her termination violates her free speech rights to be entitled to a hearing on the issue under procedural due process principles. [See Givhan v. Western Line Consolidated School District (1979)] (B) is wrong because the attorney does not appear to have a property interest in her job. A public employee who is subject to removal only for “cause” has a property interest in her job and generally must be given notice of the charges against her that are to be the basis for her job termination, and a pre-termination opportunity to respond to those charges. Here, however, the attorney did not have a property interest in her job; she could have been dismissed for no reason at all. She was not covered by the labor contract between the agency and its clerical workers, and there appears to be no other basis for her to claim an entitlement to continued employment. (C) is wrong because the attorney is entitled to a hearing as long as she can raise a prima facie claim that her speech, which was regarding an important health issue and the perception of her agency, was on a public issue and therefore protected by the First Amendment. (D) is wrong for two reasons. As discussed above, the attorney does not have a property interest in her job. Also, due process does not necessarily entitle her to a pre-termination evidentiary hearing; a post-termination evidentiary hearing is probably sufficient. [See Cleveland Board of Education v. Loudermill (1985)]

147
Q

A doctor and a lawyer owned adjoining parcels of land. Ten years ago, the doctor installed a swimming pool on her land. The doctor obtained the lawyer’s oral consent to run plumbing from the pool across part of the lawyer’s land. Last year, the lawyer sold his land to a buyer. The buyer wants to plant a garden on the land under which the doctor’s plumbing runs, and wants to eject the doctor and quiet title. The statute of limitations for ejectment is seven years. With respect to the land under which the plumbing was laid what, if anything, has the doctor acquired? A The doctor has acquired title by adverse possession. B The doctor has acquired a prescriptive easement. C The doctor has acquired both title by adverse possession and a prescriptive easement. D The doctor has acquired neither title by adverse possession nor a prescriptive easement.

A

The doctor has acquired neither title by adverse possession nor a prescriptive easement in the land under which the plumbing was laid. To establish title by adverse possession, the possession must be (i) actual and exclusive, (ii) open and notorious, (iii) adverse (hostile), and (iv) continuous throughout the statutory period. Exclusive possession generally means not sharing possession with the true owner or the general public. Possession is open and notorious when it is such as the usual owner would make of the land and is sufficient to put the true owner on notice of the fact of possession. Possession is hostile when it is without the owner’s consent. Continuous possession is possession that the average owner would make of the property under the circumstances. Acquiring a prescriptive easement is analogous to acquiring property by adverse possession; however, the use need not be exclusive. Here, although the doctor’s use and possession of the lawyer’s land was actual, exclusive, open, notorious, and continuous throughout the seven-year statutory period, the doctor used the land with the lawyer’s consent, and there are no facts to indicate that the doctor in any way manifested an intention to claim the land as her own. Therefore, the “hostile” requirement is not met and (A), (B), and (C) are wrong.

148
Q

A trial judge presiding over a lawsuit plans to call a witness to the stand and question her. May the judge do so?

A Yes, but only if the witness has already been called and examined by one of the parties.

B Yes, but only if the witness is not testifying as an expert.

C Yes, but the parties are entitled to cross-examine the witness.

D No, because only parties may call and examine witnesses.

A

Highlight Answer Discussion - Incorrect The judge may call and examine the witness, but the parties are entitled to cross-examine the witness. A court is entitled to examine any witness called by any party, and may also call a witness on its own or at a party’s request. Therefore, (D) is wrong. (A) is wrong because the court may call a witness on its own, even if the witness has not been called by any of the parties. (B) is wrong because the court may call any witness, even an expert. Although the court has wide discretion to call and examine witnesses, each party is entitled to cross-examine the court’s witness. A party may also object to the court’s examining or calling a witness either at that time or at the next opportunity when the jury is not present (to spare counsel the potential embarrassment of objecting to the judge’s questions in front of the jury).

149
Q

During the trial of a personal injury case, the plaintiff calls a witness to testify that he saw the defendant spill a slippery substance in the roadway. Following the testimony of the witness, the defendant calls the witness’s neighbor, who testifies that the witness has a poor reputation for truthfulness in the community. The plaintiff’s attorney then cross-examines the neighbor, asking her, in good faith, if she committed the crime of false pretenses last year. Last year, the neighbor had in fact been charged with and convicted of the crime of false pretenses. The defendant’s attorney objects to this question. Should the objection be sustained? Incorrect A No, because the neighbor was convicted of the crime of false pretenses. Correct B No, because the plaintiff’s attorney asked the question in good faith. C Yes, because an impeaching witness cannot be impeached on collateral matters. D Yes, because such an inquiry is not proper on cross-examination.

A

Highlight Answer Discussion - Incorrect The question by the plaintiff’s attorney should be allowed because he was acting in good faith. A witness may be impeached by means of being interrogated upon cross-examination, in the discretion of the court, with respect to any act of misconduct that is probative of truthfulness (i.e., an act of deceit or lying). The cross-examiner must act in good faith with some reasonable basis for believing that the witness may have committed the bad act inquired about, but it is not required that the witness have been convicted of a crime. Here, the plaintiff’s attorney is attempting to cast an adverse reflection on the truthfulness of the neighbor. The commission of the crime of false pretenses involves the making of a false representation and is therefore an act of misconduct that is probative of the actor’s truthfulness. Thus, because the plaintiff’s attorney inquired as to this matter in good faith, his question is a permissible method of impeachment, and the objection of the defendant’s attorney should be overruled. (A) is incorrect because it implies that the objection could be sustained if the neighbor was not convicted of the crime. As noted above, such an inquiry can be conducted regardless of whether the witness was convicted. Therefore, the objection to the plaintiff’s attorney’s good faith inquiry would be overruled even if the neighbor was not convicted of false pretenses. (C) is incorrect. Although impeaching witnesses who testify to a witness’s reputation for truth and veracity are often impeached by asking the “Have you heard” and “Do you know” questions, that is not the only method of impeachment available. Any witness who takes the stand puts her character for honesty and veracity in issue and may be impeached by evidence that might show her to be unworthy of belief. Instances of misconduct may properly be inquired into only if they are probative of truthfulness. By taking the stand, the neighbor has put her character for honesty in issue. The crime of false pretenses is probative of truthfulness and is a proper subject for impeachment. (D) is incorrect because a specific act of misconduct offered to attack the witness’s character for truthfulness can be elicited only on cross-examination of the witness. Extrinsic evidence is not permitted. Thus, (D) states the opposite of the correct rule.

150
Q

A state set up an intrastate message routing system to carry messages to and from the various state agency offices located throughout the state. This proved to be cheaper and more efficient than the United States Postal Service. The message service worked so well that the state offered the messenger service to its employees as a fringe benefit. Moreover, it expanded delivery options beyond state offices to any address in the state and permitted the employees to use the service for personal correspondence as well as for official business. Are the state’s actions constitutional? A Yes, because the messenger service operates entirely within the state borders. B Yes, because the Commerce Clause does not prohibit states from acting as a market participant. C No, because the Equal Protection Clause prohibits this singling out of state employees for special benefits. Correct D No, because it violates the federal postal monopoly.

A

Highlight Answer Discussion - Correct

The legislation is unconstitutional because it violates the federal postal monopoly. Article I, Section 8, Clause 7 of the Constitution grants Congress the power to establish post offices and post roads.

This power grants Congress a monopoly over the delivery of mail. No other system for delivering mail-public or private-can be established absent Congress’s consent. Congress has delegated to the Postal Service the power to decide whether others may compete with it, and the Postal Service has carved out an exception to its monopoly for extremely urgent letters. However, this exception would not apply to the state messenger service here since the state service extends to every letter or package of an employee deliverable within the state. (A) is irrelevant because the postal monopoly applies even to wholly intrastate competing systems. The rationale is that the Postal Service must be protected from companies that would deliver only on profitable routes at a low cost, leaving the Postal Service only expensive, money-losing routes. (B) also is irrelevant. While it is true that there is a market participant exception to the Commerce Clause, the Commerce Clause is not the controlling law here; the controlling law is the federal postal power. (C) is incorrect because the Equal Protection Clause would not prohibit the special treatment here. Because no suspect class or fundamental right is involved, the program would be judged under the rational basis standard. Under this standard, a law is upheld if it is rationally related to any legitimate government interest. Here, the law would be upheld because there is a conceivable rational basis for the program (e.g., to make government employment more attractive), and the law is rationally related to that interest.

151
Q

After being notified by a doctor that her employment with his office was terminated, a nurse applied for a position with a hospital. In her application, the nurse listed her former employment with the doctor, with the understanding that the doctor might be contacted. The doctor, in response to a telephone inquiry from the hospital, stated that the nurse “lacked professional competence.” Although the doctor reasonably believed that to be a fair assessment of the nurse, his adverse rating was based on an episode of malpractice for which he blamed the nurse but which in fact was chargeable to another doctor. Because of the doctor’s adverse comment on her qualifications, the nurse was not employed by the hospital. If the nurse asserts a claim based on defamation against the doctor, will the nurse prevail?

A Yes, because the doctor was mistaken in the facts on which he based his opinion of the nurse’s competence.

B Yes, because the doctor’s statement reflected adversely on the nurse’s professional competence.

C No, because the nurse authorized the hospital to make inquiry of her former employer.

D No, because the doctor had reasonable grounds for his belief that the nurse was not competent.

A

Highlight Answer Discussion - Incorrect (D) is correct.

The nurse will not prevail because the doctor had reasonable grounds for his statement. As a former employer responding to queries of a prospective employer about a job applicant, the doctor has a qualified privilege. Such a privilege is not absolute; it exists only if exercised in a reasonable manner and for a proper purpose. The privilege may be lost if the speaker made a statement not within the scope of the privilege or if the speaker acted with actual malice (i.e., knowledge that the statement was untrue or with reckless disregard as to its truth or falsity). Because the doctor had reasonable grounds for his belief, he was not acting with actual malice.

(A) is incorrect because of the reasons stated in the analysis above. A statement of opinion may be actionable if it appears to be based on specific facts which, if expressly stated, would be defamatory. However, because of the qualified privilege, the doctor will not be liable for his mistake as long as his belief was reasonable. (B) is incorrect because the fact that the statement was in a category that is slander per se (i.e., adversely reflecting on the nurse’s abilities to practice her profession) goes to whether the nurse must plead special damages. It does not, however, undermine the qualified privilege. (C) is incorrect because permission to make inquiry is not tantamount to consent to be defamed.

152
Q

A homeowner contracted for construction of a custom-built, elevated deck in his backyard. The deck’s designer supervised the construction, which was carried out by several employees of a local building company. The homeowner was pleased with the appearance of the deck, but the first time he stepped on it, a support on one side of the deck gave way, causing the homeowner to fall and be injured. The homeowner brought an action joining the building company and the deck’s designer as defendants, alleging negligence. In his complaint, he alleged that he does not know which of the defendants is responsible for the damages. Which of the following doctrines would be most helpful against the designer? Incorrect A Respondeat superior. Correct B Res ipsa loquitur. C Contribution. D Indemnity.

A

Res ipsa loquitur will be most helpful against the designer. Res ipsa loquitur means the thing speaks for itself. It is appropriate in situations where an injury does not usually occur unless someone was negligent and the plaintiff does not know which of the defendants caused the injury. While res ipsa loquitur is sometimes not available where more than one person may have been in control of the instrumentality causing the injury, it is available in a case where a particular defendant had the power of control over the site of the injury. Even if the homeowner does not know why the deck collapsed, the deck’s designer would be responsible because he designed the deck and was supervising the construction. Hence, res ipsa loquitur likely could be used. (A) is not correct because the doctrine of respondeat superior imposes vicarious liability on an employer for the tortious conduct of its employee. That doctrine would be helpful against the building company for any negligence by its employees but not against the designer, because the workers were not the designer’s employees. (C) and (D) are incorrect because the doctrines of contribution and indemnity pertain to how the responsibility of the loss is apportioned or shifted among the defendants after the plaintiff has recovered his judgment. They are not relevant to the homeowner’s right to recover.

153
Q

The owner of a small fleet of taxicabs had his cabs serviced by a national chain of auto service centers. One of his cabs went through a stop sign when its brakes failed without warning. The ensuing collision seriously injured the passenger. An investigation revealed that brake repairs had been made on the cab a week before, but the service center’s mechanic had used the wrong parts and had made numerous errors in reassembling the brakes.

If the passenger sues the cab company owner for her injuries, who should prevail?

A The passenger should prevail, unless the jury determines that the owner exercised a high degree of care in selecting the service center for maintenance of his cabs.

B. The passenger should prevail, because the owner breached his duty to her to provide a safe vehicle in which to ride.

C. The owner should prevail, because he had no reason to know that the service center’s mechanic would be negligent.

D. The owner should prevail, because he is not vicariously liable for the negligence of an independent contractor.

A

The passenger will recover against the owner for her injuries because the owner, a common carrier, owed her a nondelegable duty to provide a safe vehicle in which to ride. The general rule is that a principal will not be liable for tortious acts of his agent if the agent is an independent contractor.

However, a major exception to this rule applies when the duty, because of public policy considerations, is nondelegable. In these cases, the principal is vicariously liable for the agent’s negligence despite the principal’s own exercise of due care.

A common example of these types of duties is the duty of a business to keep its premises and instrumentalities safe for its customers. This includes the duty of a common carrier, such as a taxi company, to keep its vehicles in safe working order. Thus, the owner’s duty to the passenger, a passenger in his cab, was nondelegable. The negligent conduct of the mechanic is deemed to be that of the owner. The negligent conduct was the actual and proximate cause of the passenger’s injuries. Thus, the owner is vicariously liable to the passenger for those injuries.

(A) and (C) are wrong because the passenger will prevail regardless of how careful the owner was in selecting the service center to maintain his cabs. As a common carrier, the owner owes his passenger, the passenger, a very high degree of care; i.e., he will be liable for slight negligence. However, because his duty to provide a safe taxicab is not delegable, the fact that he was careful in selecting the mechanic is irrelevant. Even though the owner had no reason to know that a service center mechanic would be negligent, he is vicariously liable for that negligence because it caused injury to the passenger. (D) is wrong because, as stated above, the situation here falls within an exception to the general rule of no liability for the torts of an independent contractor.